Categories
Exam Questions Harvard

Harvard. Exam questions for European countries’ resources and economies. Ripley and Meyer, 1901-02

This post adds to the expanding collection of Harvard economics examinations. European economic geography was the subject of a year-long course co-taught in 1901-1902 by William Zebina Ripley and Hugo Richard Meyer.

_____________________________

Last minute staff replacements for 1901-1902

It will be recalled that the Harvard economics department was faced with enormous staffing problems for the 1901-1902 academic year. Frank Taussig was on sick-leave and Edward Cummings and William Ashley had left the department for other pastures.

In an earlier post we saw that Charles W. Mixter substituted for William Ashley in a history of economic thought course.

Thomas Nixon Carver had to fill-in for Frank Taussig for the advanced theory course.  He also took over Taussig’s methods course.

Thomas Nixon Carver and Frederick Bush were needed to cover the “Socialism and Communism” course previously taught by Edward Cummings.

In this post Economics in the Rear-view Mirror adds the transcriptions of exams for a course on European resources and organization that had been previously taught by William Ashley but was taught by William Z. Ripley and Hugo R. Meyer in 1901-1902.

Examinations for Professor William J. Ashley’s course in 1900-01  posted earlier.

Biographical information for Professor William Z. Ripley also posted earlier.

Economics in the Rear-view Mirror has also posted some life and career information for Hugo Richard Meyer.

_____________________________

ECONOMICS 17
Course Description
1901-1902

For Undergraduates and Graduates

[Economics] 17. The Economic Organization and Resources of European Countries. Tu., Th., and (at the pleasure of the instructor) Sat., at 12. Professor Ashley.

This course will begin with a survey of the physical geography and of the distribution of population and wealth in Europe as a whole, in order to explain (1) the relative position at present of agriculture in its various forms on the one side and mining and manufacture on the other, and (2) the presence in their several localities of the chief industries. The great countries — Great Britain and Ireland, France, Germany, Austria-Hungary, Russia, and Italy — will then be taken in order; and a statement will be made (1) of their natural resources, and (2) of the manner in which these are utilized. Under the latter head will be considered such topics as the following: the investment of capital, the forms of business organization, the means of transportation, the relations between private enterprise and governmental stimulus and control, and the character and supply of labor. Returning, then, to Europe as a whole, attention will be directed to the points at which the interests of the several countries appear to conflict, and to the attempts to remove or mitigate the antagonism by international agreements — as, for instance, in the matters of customs tariffs, bounties, and labor legislation. Finally, a survey will be made of the main lines of transportation for commodities between one country and another and between Europe and the rest of the world, and of the economic effects of recent changes in this regard.

Course 17 is open to those who have passed satisfactorily either in History 1 or in Economics 1.

Source: Harvard University Archives. Official Register of Harvard University 1901-1902. Box 1. Faculty of Arts and Sciences, Division of History and Political Science (June 21, 1901), University Publications, New Series, No. 16, p. 39.

_____________________________

Enrollment
1901-02
ECONOMICS 17

Economics 17. Professor Ripley and Mr. Meyer. — The Economic Organization and Resources of European Countries.

Total 24: 2 Graduates, 13 Seniors, 6 Juniors, 2 Sophomores, 1 Other.

Source: Harvard University. Report of the President of Harvard College, 1901-1902, p. 77.

_____________________________

ECONOMICS 17
Mid-year Examination
1901-1902

  1. English farming and German farming from 1879 to 1900: a study of the respective effects of the policy of free-trade in grains and meats and the policy of protection to agriculture.
    Alternative:—
    The arguments for and against the small farm and the peasant farm. Take your facts from the history of England; Belgium or France; and Germany.
  2. The German customs duties on grains and meats, and Germany’s position in the “struggle for the export markets.”
    Alternative:—
    Give an account, descriptive and critical, of the beet-sugar policy of the countries of continental Europe from 1870 to 1900
  3. What facts and what kinds of statistics were used in the discussion of the failure of the Prussian (German) State Railways to contribute materially to the industrial development effected in Germany in the years 1880 to 1900?
  4. What kinds of statistics were used in discussing the mineral resources of England, Germany, France, and the United States?
  5. The birth-rate in France in the last twenty-five years.

Source: Harvard University Archives. Harvard University Mid-Year Examination Papers, 1852-1943. Box 6, Bound volume, Mid-year Examination Papers, 1901-02. Sub-volume Papers Set for Final Examinations in History, Government, Economics, … in Harvard College (January 1902).

_____________________________

ECONOMICS 17
Year-end Examination
1901-1902

  1. State four economic reasons for the imperial federation movement in Great Britain.
  2. Analyze the principal characteristics of British foreign trade, comparing it with Germany.
  3. When and why did Germany change from a free trade to a protectionist country, and what had Prince Bismarck to do with the movement?
  4. How do the Agrarian demands in the present tariff discussion in Germany compare with those of a generation ago?
  5. Outline the present status of France, in respect of population, industry, and foreign trade.
  6. What are some of the international industrial combinations?
  7. What are the main features of the E. J. Smith type of industrial combinations in Great Britain?
  8. Compare Germany and Austro-Hungary in respect of the industrial combination movement.

Source: Harvard University Archives. Harvard University Examination Papers, 1873-1915. Box 6, Bound volume, Examination Papers, 1902-03. Sub-volume Papers Set for Final Examinations in History, Government, Economics, … in Harvard College (June 1902).

Image Source: Political Map of Europe, ca. 1890 in Wikimedia Commons.

Categories
Exam Questions Harvard History of Economics

Harvard. Final exams for history of economics up through Ricardo. Mixter, 1901-1902

 

With Edward Cummings and William J. Ashley gone and Frank W. Taussig on a medical leave-of-absence, the Harvard economics department had to scramble to cover its course offerings in 1901-02. The course on the history of economics up through the early nineteenth century was then taught by Harvard economics Ph.D. alumnus, Charles W. Mixter. His semester final examinations questions have been transcribed below.

In an earlier post we find the exams from 1900-01 when William J. Ashley last taught the course at Harvard.

The immediately preceding post provides us with a student’s POV of University of Vermont Professor Charles W. Mixter in the classroom. 

*  *  *  *  *  *  *  *  *  *  *  *  *  *

Charles Whitney Mixter
(b. Sept. 23, 1869 in Chelsea, MA;
d. Oct. 21, 1936 in Washington, D.C.)

A.B. Johns Hopkins University (Md.), 1892; A.M. Harvard University, 1893; 1897 Harvard Ph.D.

Thesis title: Overproduction and overaccumulation: a study in the history of economic theory.

Edited Work

John Rae. The Sociological Theory of Capital, being a complete reprint of the New Principles of Political Economy, 1834Edited with biographical sketch and notes by Charles Whitney Mixter, Ph.D., Professor of Political Economy in the University of Vermont. New York: Macmillan, 1905.

OBITUARY
The Burlington Free Press (Oct. 22, 1936), p. 14

Charles Whitney Mixter, for nine years a member of the University of Vermont faculty, died at a hospital in Washington, D. C., on Tuesday evening. [October 20]

Dr. Mixter was born in Chelsea, Mass., in 1867. He received his early education at Thayer Academy and Williston Seminary, and received his A.B. degree from John Hopkins University in 1892.

This was followed by graduate studies at Berlin, Goettingen and Harvard, from which he received his doctorate in 1897. Then followed a series of teaching positions: Assistant in economics at Harvard, 1897-98; Trinity College, Hartford, Conn., 1899-1900; instructor in economics, Harvard, 1901-1903; professor of economics, University of Vermont, 1903-1912.

Then Dr. Mixter served as efficiency expert for Towne and Yale at New Haven, Conn., and later for several manufacturing concerns in New Hampshire. For a year he was professor of economics at Clark University, and for a brief period he was an investigator in the service of the United States Chamber of Commerce.

For the last 13 years he had been connected with the tariff commission in Washington.

Professor Mixter had an unusually fertile mind, was an accomplished scholar in his special field, and widely read in related subjects. he became an enthusiastic student of scientific management introduced by the late Frederick W. Taylor and an active exponent of the system. He was a member of the leading economic organizations and a frequent contributor to economic journals.

He was a strong advocate of free trade. Interment was made in Plymouth, Mass.

_____________________________

ECONOMICS 15
Course Description
1901-02

Primarily for Graduates

[Economics] 15. The History and Literature of Economies, to the opening of the Nineteenth Century.
Mon., Wed., and (at the pleasure of the instructor) Fri., at 12. Professor Ashley.

The course of economic speculation will here be followed, in its relation alike to the general movement of contemporary thought and to contemporary social conditions. The lectures will consider the economic theories of Plato and Aristotle; the economic ideas underlying Roman law; the medieval church and the canonist doctrine; mercantilism in its diverse forms; “political arithmetic”; the origin of the belief in natural rights and its influence on economic thought; the Physiocratic doctrine; the beginnings of academic instruction in economics; the work and influence of Adam Smith; the doctrine of population as presented by Malthus; and the Ricardian doctrine of distribution.

The lectures will be interrupted from time to time for the examination of selected portions of particular authors; and careful study will be given to portions of Plato’s Republic and Aristotle’s Politics (in translation), to Mun’s England’s Treasure, Locke’s Consideration of the Consequences of the Lowering of Interest, certain Essays of Hume, Turgot’s Réflexions, and specified chapters of Adam Smith’s Wealth of Nations, Malthus’ Essay, and Ricardo’s Principles. Students taking the course are expected to procure the texts of the chief authors considered, and to consult the following critical works: Ingram, History of Political Economy; Cossa, Introduction to the Study of Political Economy; Cannan, History of the Theories of Production and Distribution; Bonar, Philosophy and Political Economy; Böhm-Bawerk, Capital and Interest; Taussig, Wages and Capital.

Course 15 is open to those who have passed satisfactorily in Course 1. It is taken to advantage after Course 2, or contemporaneously with that Course.

Source: Harvard University Archives. Official Register of Harvard University 1901-1902. Box 1. Faculty of Arts and Sciences, Division of History and Political Science (June 21, 1901), University Publications, New Series, No. 16, p. 45.

_____________________________

ECONOMICS 15
Enrollment
1901-02

Economics 15. Dr. Mixter. — The History and Literature of Economics to the opening of the Nineteenth Century.

Total 5: 3 Graduates, 2 Seniors.

Source: Harvard University. Report of the President of Harvard College, 1901-1902, p. 78.

_____________________________

ECONOMICS 15
Mid-year Examination
1901-02

  1. Give an account of Aristotle on “the art of money-making”(χρηματιστική) as contrasted with “household management” (οἰκονομική); on the institution of private property.
  2. Why was economics little cultivated in classical times: in Ingram’s opinion; in your opinion?
  3. Where, in economic literature, do the following expressions occur, and what was meant by them: “City of pigs”; “Private Vices, Public Benefits”; “led by an invisible hand”?
  4. The chief distinction between man and the inferior animals consists in this: They are moved only by the immediate impressions of sense, and, as its impulses prompt, seek to gratify them from the objects before them, scarce regarding the future, or endeavoring from the experience of the past to provide against what is to come. Man, as he is endowed with reason,…” Who first expressed this thought? What use was made of it by a later writer?
  5. What passage in the Wealth of Nations has frequently been quoted as giving a concise statement of the author’s theory of the law of profits? What is the usual criticism of this passage? What your own criticism?
  6. Many writers have held that the increase of capital lessens at the same time the demand for the products of capital, since savings are made by curtailing one’s consumption. Show the fallacy of this contention?
  7. State the doctrine of wages in the Wealth of Nations, bringing out the contrast with the pre-Smithian doctrine.
  8. What are Adam Smith’s four “maxims” or canons of taxation, and what his position on “Taxes upon Profit, or upon the Revenue arising from Stock”?
  9. Comment on the leading arguments of the chapter, “Of Restraints upon the Importation from foreign Countries of such Goods as can be produced at Home.”
  10. What are the “Effects of the Progress of Improvement upon the real Price of Manufactures”? What is the significance of this doctrine in the history of economic opinion?

Source: Harvard University Archives. Harvard University Mid-Year Examination Papers, 1852-1943. Box 6, Bound volume, Mid-year Examination Papers, 1901-02. Sub-volume Papers Set for Final Examinations in History, Government, Economics, … in Harvard College (January 1902).

_____________________________

ECONOMICS 15
Year-end Examination
1901-02

  1. Who were they and what do they stand for: Nicholas Oresme, Acquinas, Thomas Mun, Boisguillebert, Turgot, Gournay?
  2. What was the general advance in economic thought during the century preceding the publication of the Wealth of Nations?
  3. Comment upon Jones’ „Primitive Political Economy in England” and Schmoller’s Mercantile System.
  4. Give a critical account of the history of opinion on the subjet of lending money at interest.
  5. Sketch in outline the history of the theory of “natural law” and indicate the way in which it came in contact with economics.
  6. What part of the teaching of the Physiocrats do you consider to have helped forward economic science, and what part to have been of little or no use?
  7. What was “Political Arithmetic”?
  8. Give a brief account of :–
    1. Speculation on the subject of population before Malthus.
    2. The Malthusian doctrine, its purpose, its content, the argument put forward in its support.
    3. The bearing of Rae’s principle of “the effective desire of offspring” upon the Malthusian doctrine.

Source: Harvard University Archives. Harvard University Examination Papers, 1873-1915. Box 6, Bound volume, Examination Papers, 1902-03. Sub-volume Papers Set for Final Examinations in History, Government, Economics, … in Harvard College (June 1902).

Image Source: Harvard University Archives. Hollis Images. College Yard, ca. 1900.

Categories
Exam Questions Harvard Statistics

Harvard. Statistics. Course description and semester exams. Ripley, 1901-1902

William Zebina Ripley first came on board the Harvard economics department as a visiting professor from M.I.T. for the academic year 1901-02. There was a screaming need for someone to cover the statistics course that had been taught by John Cummings through 1899-1900 but had been bracketed (i.e. not offered) for 1900-01. From the June announcement of course offerings for 1901-02 we see that the department’s economic historian, William J. Ashley, was originally planned for teaching the statistics course. That plan needed to be scrapped once Ashley announced his resignation to go the University of Birmingham. Down the Charles River at M.I.T. William Z. Ripley, assistant professor of Sociology and Economics, turned out to be a good fit for the department’s instructional program. 

_________________________

Related post in the Economics in the Rear-view Mirror archive:

Harvard. Short Bibliography on Social Statistics for “Serious-minded Students”, Ripley, 1910

_________________________

Original Course Announcement
(June 1901)

For Undergraduates and Graduates

4 hf. Statistics. — Theory, methods, and practice. Half-course. Fri., at 11. Professor [William J.] Ashley.

After a brief history of statistics, this course will proceed to an exposition of the statistical methods most commonly employed, and a statement of the theoretical considerations most deserving of attention in practical investigation. An account, with running comment, will then be given of the work of government offices; and the latter part of the year will be employed in the disentangling and comparison of the main results of the recent industrial censuses of Germany and France. Two reports on assigned topics will be required during the year, from every student in the course.

Course 4 is open to students who have taken Economics 1; and it is also open to Juniors and Seniors who are taking Economies 1.

Source: Harvard University Archives. Annual Announcement of the Faculty of Arts and Sciences, Division of History and Political Science comprising the Departments of History and Government and Economics (June 21, 1901).  Official Register of Harvard University 1901-1902. Box 1. Bound volume: Univ. Pub. N.S. 16. History, etc. p. 41.

_________________________

Revised Statement
Concerning Course 4 hf.

4 hf. Statistics. — Theory, methods, and practice. Half-course. Tu., at 11. Professor [William Zebina] Ripley (Mass. Inst. Technology).

This course is intended to serve rather as an analysis of methods of research and sources of information than as a description of mere results. A brief history of statistics will be followed by an account of modes of collecting and tabulating census and other statistical material in the United States and abroad; the scientific use and interpretation of results by the mean, the average, seriation, the theory of probability, etc. The main divisions of vital statistics, relating to birth, marriage, morbidity, and mortality, life tables, etc.; the statistics of trade and commerce, such as price indexes, etc.; industrial statistics relating to labor and employment; statistics of agriculture and manufacture; of transportation by means of ton, car, train-mileage, and revenue; will be then considered in order. The principal methods of graphic representation will be comprehended, and laboratory practice in the preparation of charts, maps, and diagrams from original material will be required.

Course 4 is open to students who have taken Economics 1; and it is also open to Juniors and Seniors who are taking Economies 1.

Source: Harvard University Archives. Syllabi, course outlines and reading lists in Economics, 1895-2003. Box 1, Folder “Economics, 1901-1902.”

_________________________

Course Enrollment

For Undergraduates and Graduates:—

[Economics] 4 hf. Professor [William Zebina] Ripley. — Statistics. Theory, method, and practice.

Total 5:  2 Graduates, 2 Seniors, 1 Junior.

Source: Harvard University. Report of the President of Harvard College, 1901-1902, p. 77.

_________________________

Mid-Year Exam, 1902
ECONOMICS 4

  1. Outline the different tests to be applied to determine in any given population,
    1. the existence of migration,—
    2. its amount and character.
  2. How do still-born and illegitimate births compare with normal and legal ones, in character and frequency?
  3. Compare urban and rural populations from the point of view of births and marriages; giving suitable explanations.
  4. What are the principal errors to be guarded against or corrected in a census enumeration.
  5. Describe three methods of estimating population at intercensal periods, critically comparing their merits.
  6. How might the vital statistics of two American states differ, on account of a wide difference in the proportions of male and female adults?
  7. What is
    1. the expectation of life?
    2. the refined birth rate?
  8. What does a mortality table show. Construct an hypothetical one.

Source: Harvard University Archives. Harvard University Mid-year Examinations, 1852-1943. Box 6, Bound volume: Examination Papers, Mid-Years, 1901-02.

Year-end Examination, 1902
ECONOMICS 4

  1. Explain the construction of a life table and show its uses.
  2. What are the main errors to be avoided in comparison of mortality rates of different countries or groups of population?
  3. Why are index numbers of prices apt to be misleading? Show by an example.
  4. Criticize the Senate Finance Committee Report on Prices and Wages, as to its methods.
  5. In what way may movements of wages be most accurately measured?
  6. To what classes of phenomena may the modulus be best applied, and what is its advantage over the use of averages?
  7. What are the principal defects in statistics as to the movement of imports and exports?
  8. What is the best method of “smoothing curves” in a graphic diagram? Give an example.

Source: Harvard University Archives. Harvard University, Examination Papers, 1873-1915. Box 6, Bound volume: Examination Papers, 1902-03. Papers Set for Final Examinations in History, Government, Economics, Philosophy, Education, Fine Arts, Architecture, Landscape Architecture, Music in Harvard College (June, 1902), p. 22.

Image Source: M.I.T. yearbook, Technique 1901, p. 26.

Categories
Faculty Regulations Harvard Teaching Undergraduate

Harvard. Observations on the organization of academic life at Harvard. Ashley, 1897

 

The economic historian William J. Ashley taught at Harvard from 1892 to 1901. His observations regarding the tension between professors and junior staff’s desire to work for the advancement of science and scholarship and the core educational mission of universities have not lost their relevance 125 years after he shared his Harvard experience.

But there are many wonderful obiter dicta you will want to savor. 

________________________

C.V. from Harvard’s records

William James Ashley, B.A. Oxford 1881; M.A. Oxford 1885; M.Com. Birmingham (Eng.) 1902; Ph.D. Berlin 1910; Fellow Lincoln Coll. Oxford 1885; Prof. of Political Economy and Constitutional History, Toronto 1888-1892; Prof. of Economic History 1892-1901; Prof. of Commerce, Birmingham; Dean (Faculty of Commerce) Birmingham 1901-1902; Cor. Memb. Mass. Hist. Soc; Cor. Memb. Am. Acad.

Harvard University. Quinquennial catalogue of the officers and graduates 1636-1930, p. 42.
________________________

Prof. William J. Ashley on academic administration and procedures at Harvard (1897)

[…] The peculiarity in the position of Harvard is that while the professorial ideal has definitely triumphed among the teaching body, the tutorial ideal is still cherished by the ‘constituency.’ Most of the professors care first of all for the advancement of science and scholarship; they prefer lectures to large audiences to the catechetical instruction of multiplied ‘sections,’ and they would leave students free to attend lectures or neglect them, at their own peril; they would pick out the abler men, and initiate them into the processes of investigation in small ‘research courses’ or ‘seminaries;’ and, to be perfectly frank, they are not greatly interested in the ordinary undergraduate. On the other hand the university constituency — represented, as I am told, by the Overseers — insists that the ordinary undergraduate shall be ‘looked after;’ that he shall not be allowed to ‘waste his time;’ that he shall be ‘pulled up’ by frequent examinations, and forced to do a certain minimum of work, whether he wants to or not. The result of this pressure has been the establishment of an elaborate machinery of periodical examination, the carrying on of a vaster bookkeeping for the registration of attendance and of grades than was ever before seen at any university, and the appointment of a legion of junior ‘instructors’ and assistants, to whom is assigned the drudgery of reading examination — books and conducting ‘conferences.’

So far as the professors are concerned, the arrangement is as favourable as can reasonably be expected. Of course they are all bound to lecture, and to lecture several times a week; they exercise a general supervision over the labours of their assistants; they guide the studies of advanced students; they conduct the examinations for honours and for higher degrees; they carry on a ceaseless correspondence; and each of them sits upon a couple of committees. But they are not absolutely compelled to undertake much drudging work in the way of instruction, and if they are careful of their time they can manage to find leisure for their own researches. As soon as a ‘course’ gets large, a benevolent Corporation will provide an assistant. The day is past when they were obliged, in the phrase of Lowell, ‘to double the parts of professor and tutor.’

But the soil of America is not as propitious as one could wish for the plant of academic leisure. It is a bustling atmosphere; and a professor needs some strength of mind to resist the temptation to be everlastingly doing something obvious. The sacred reserves of time and energy need to be jealously guarded, and there is more than one direction from which they are threatened. University administration occupies what would seem an unduly large number of men and an unduly large amount of time; it is worth while considering whether more executive authority should not be given to the deans. Then there is the never — ending stream of legislation, or rather of legislative discussion. I must confess that when I have listened, week after week, to Faculty debates, the phrase of Mark Pattison about Oxford has some times rung in my ears: ‘the tone as of a lively municipal borough.’ It would be unjust to apply it; for, after all, the measures under debate have been of far-reaching importance. Yet if any means could be devised to hasten the progress of business, it would be a welcome saving of time. Still another danger is the pecuniary temptation — hardly resistible by weak human nature — to repeat college lectures to the women students of Radcliffe. That some amount of repetition will do no harm to teachers of certain temperaments and in certain subjects may well be allowed, but that it is sometimes likely to exhaust the nervous energy which might better be devoted to other things can hardly be denied. The present Radcliffe system, to be sure, is but a makeshift, and an unsatisfactory one.

The ‘instructors’ and assistants, on their part, have little to grumble at, if they, in their turn, are wise in the use of their time. It is with them, usually, but a few years of drudgery, on the way to higher positions in Harvard or elsewhere; and it is well that a man should bear the yoke in his youth. Let him remember that his promotion will depend largely upon his showing the ability to do independent work; let him take care not to be so absorbed in the duties of his temporary position as to fail to produce some little bit of scholarly or scientific achievement for himself. I have occasionally thought that the university accepts the labours of men in the lower grades of the service with a rather step-motherly disregard for their futures.

Come now to the ‘students,’ or whose sake, certainly, Harvard College was founded, whatever may have been the case with English colleges, and whose presence casts upon those responsible for academic policy duties which they cannot escape, if they would. Grant that education and education as Jowett understood it, the training of character as well as mere instruction — is the main business of a university, what is to be said of the situation of affairs? That we do as much here for the average man as the Oxford tutorial system accomplishes, it would be idle to affirm. The introduction of the tutorial system, however, is out of the question: it needs the small college for its basis; it requires that the tutor should enjoy a prestige which we cannot give him; and it is still further shut out by ‘elective’ studies. Yet in its way the Harvard practice suffers from the same defects as the Oxford; it does too much for the men. Take the matter of examinations, for instance. Surely it would be better to relax the continuous pressure — which after all is not in any worthy sense effective — and to reinforce it instead at special points. It was the conviction, we are told, of Professor Freeman that ‘if examinations were necessary evils, they should be few, searching, and complete, not many and piecemeal.’ At present, there are so many ‘tests,’ of one sort or another, that no one examination sufficiently impresses the undergraduate mind. The kind of work done by a student who is so persistently held up by hour-examinations and conferences that he must be an abnormal fool to ‘fail’ at the end, cannot be regarded as really educational in any high sense of the word. By a great many men, the help showered upon them is regarded merely as the means of discovering just how little they can do, and still scrape through. To sweep away all examinations except the final annual one; to leave the student more to himself; to set a higher standard for passing, and ruthlessly reject those who do not reach it, would undoubtedly, in the long run, encourage a more manly spirit on the part of undergraduates, and a deeper respect for the university. This I say with the fuller confidence because, when I left Oxford, now (1900) some twelve years ago, I could see nothing but the evils of the examination system as it there affects students of promise. I am convinced that it would be possible and salutary in Harvard to add greatly to the awfulness of examination; and that much could be done in this direction without approaching within measurable distance of any results that need be feared.

From a natural distrust of examinations and a desire to encourage independent thought, it has of late become the practice to prescribe two or more theses during the progress of a ‘course.’ The result is that many a man has half a dozen or more theses to write during the year, for two or three different teachers. This undoubtedly ‘gets some work out of the men.’ But the too frequent consequence, with students who take their work seriously, especially with graduates, is that they have no time for anything but to get up their lectures and prepare their theses. Any parallel reading by the side of their lectures they find impracticable. But one of the best things a student can do is just to read intelligently. Certainly the graduate students, if not the undergraduates, would sometimes be the better for being left more to themselves.

These are, however, relatively minor matters. A good deal could be said about that cornerstone of Harvard academic policy, the ‘elective’ system. I must confess that I have hitherto failed to see the advantage of the completely elective plan (for any but exceptional students) over the plan of ‘groups,’ or ‘triposes,’ or ‘schools,’ with some degree of internal elasticity to suit particular tastes. That the elective system is an improvement on the old compulsory curriculum is likely enough; but I do not know that any great American university has ever yet fairly tried the group arrangement. Of all the educational agencies at Oxford, Oxford itself is the most potent.

That sweet city, with her dreaming spires;
She needs not June for beauty’s heightening.

Harvard, indeed, is truly ‘fair’ at Commencement, and in the evening lights the Yard has always a sober dignity. But Harvard in the daytime sadly needs May or October for beauty’s heightening. The disadvantages of youth and climate may not be altogether surmountable; yet Cambridge surroundings could doubtless be made more comely and restful with comparatively little trouble. There must be a certain atrophy of the æsthetic sense when luxuriously furnished dormitories have no difficulty in securing tenants though they face rubbish dumps, when rowing-men can practise with equanimity beneath a coal-dealer’s mammoth advertisement, and when the crash and jangle of street-cars are permitted to destroy what little remains of the quiet of the Yard. What is to be desired for every student — most of all for those from homes of little cultivation — is that he should live in the presence of grace and beauty and stateliness. The lesson of good taste cannot be learnt from lectures, and is imbibed unconsciously. Here we must turn to our masters, the Corporation, and to the worshipful benefactors to come. Is all the thought taken that might be taken, all the pressure used that might be exerted, to increase the amenity of the neighbourhood? And, further, is it utopian to imagine that some benefactor will yet arise who will enable Harvard to imitate the noble example of Yale, and erect dormitories that shall delight the eye? Is it too much to hope that the university may soon be enriched with at least one more building such as Memorial Hall? For many a Harvard student his daily meals in Memorial Hall, in that ample space, beneath the glowing colours of the windows and surrounded by the pictures of the Harvard worthies of the past, constitute the most educative part of his university career, though he may not know it. Only half the students can now be brought within this silent influence. A second dining-hall, of like dignity, is perhaps the most urgent educational need of Harvard, and the need most easily supplied.[*]

[*I leave this sentence, for obvious reasons, in spite of the recent erection of Randall Hall. The desirability of a large infusion of other than immediately utilitarian elements in the policy of the Corporation is emphasised, I think, by the increasingly evident tendency towards social segregation in the student body. The English reader who desires to know more of the atmosphere of the greater American universities may be referred to Mr. Bliss Perry’s article on ‘The Life of a College Professor’ in Scribner’s Magazine for October 1897; while the American reader who is interested in Oxford may with advantage consult Mr. F. C. S. Schiller on ‘Philosophy at Oxford’ in the Educational Review for October 1899. ]

Source:  W.  J. Ashley, “Jowett and the University Ideal” in Surveys, Historic and Economics 1900, pp. 445-463. Originally published in Atlantic Monthly, July 1897.

 

Categories
Exam Questions Harvard

Harvard. Economics semester final examinations, 1900-01.

In the first full academic year of the twentieth century the Harvard economics department offered the following courses. The course links take you to the official course announcement, instructor names, enrollment figures, and the transcribed semester examinations.

Economics 1. Outlines of Economics
Economics 2. Economic Theory of the 19th Century
Economics 3. Principles of Sociology
Economics 5. Railways and Other Public Works
Economics 6. Economic History of the U.S.
Economics 8. Money
Economics 9. Labor Question in Europe and the U.S.
Economics 10. European Mediaeval Economic History
Economics 12. Banking and Leading Banking Systems
Economics 12a. International Payments and Gold/Silver Flows
Economics 13. Methods of Economic Investigation
Economics 17. Economic Organization and Resources in Europe
Economics 18. Principles of Accounting
Economics 19. General View of Insurance
Economics 20d. Adam Smith and Ricardo

__________________________________

Economics 1.
Outlines of Economics

Primarily for Undergraduates.

Course Announcement
  1. Outlines of Economics. , Wed., Fri., at 9. Professor Taussig, Dr. Sprague, Mr. Andrew, and Messrs. — and — .

Source: Harvard University. Announcement of the Courses of Instruction provided by the Faculty of Arts and Sciences for the Academic Year 1900-1901, p. 41.

Course Enrollment
  1. Professor [Frank W.] Taussig, Drs. [Oliver Mitchell Wentworth] Sprague and [Abram Piatt] Andrew, and Messrs. [Charles] Beardsley and [James Horace] Patten. — Outlines of Economics.

Total 442: 23 Seniors, 70 Juniors, 257 Sophomores, 29 Freshmen, 63 Others.

Source: Harvard University. Report of the President of Harvard College 1900-1901, p. 64.

1900-01
ECONOMICS 1
[Mid-year Examination]

Arrange your answers strictly in the order of the questions.

  1. In what manner do you think that (a) the individual efficiency of laborers, (b) their collective efficiency, would be affected by the general adoption of profit sharing? of socialism?
  2. It has been said that the original formation of capital is due to abstinence or saving, but its permanent maintenance is not. What do you say to either statement?
  3. Wherein is Walker’s presentation of the forces that make the general rate of wages better than Mill’s, wherein not so good?
  4. “The extra gains which any producer or dealer obtains through superior talents for business, or superior business arrangements, are very much of a similar kind. …All advantages, in fact, which one competitor has over another, whether natural or acquired, whether personal or the result of social arrangements, bring the commodity, so far, into the Third Class, and assimilate the advantage to a receiver of rent.”
    Explain (a) what is this Third Class, and what is the law of value applicable to it; (b) what Mill would say as to the proposition here stated; (c) what Walker would say?
  5. What qualifications of the general principle of rent can you state, in its application to (a) premises used for building purposes, (b) dwelling-houses, (c) mines?
  6. If all men had the same start in life, would there be differences of wages? If so, of what sort? If not, why not?
  7. “Since cost of production here fails us, we must revert to a law of value anterior to cost of production and more fundamental…” In what cases does cost of production fail us? Will “cost of reproduction” cover such cases? Is there another law more fundamental?
  8. Under what circumstances. if ever, will a general rise in wages affect the relative values of commodities? Would your answer be the same as to a general rise in profits?
  9. In what manner do you believe business profits, interest, and wages would be affected by the general adoption of the various forms of consumers’ coöperation? of producers’ cooperation?

Source: Harvard University Archives. Harvard University Mid-year Examinations, 1852-1943. Box 4, Bound volume: Examination Papers, Mid-Years, 1900-01.

1900-01
ECONOMICS 1
[Year-end Examination]

I.
Answer three.

  1. How will the value of land be affected by
    1. an increase in population,
    2. a reduction in the rate of interest,
    3. a protective tariff on agricultural produce.
  2. How will the price of grain be affected by
    1. a tax proportioned to the economic rent of the land,
    2. an equal tax upon all land.
  3. “Profits do not form a part of the price of the products of industry, and do not cause any diminution of the wages of labor.”
    Would Mill agree to this statement? Would you?
  4. Upon what does the general level of wages depend (a) according to Mill, (b) according to Walker? What would you expect these writers to say as to the effect of a protective tariff on the general level of wages?

II.
Answer two.

  1. If a country exports on a large scale a commodity not previously exported, will its other exports be affected? If so, how? If not, why not?
  2. Can a country have a permanently “unfavorable” balance of trade? If so, under what conditions? If not, why not?
    Can a country permanently export specie? If so, under what conditions? If not, why not?
    Can the rate of foreign exchange in a country be permanently at the specie-shipping point? If so, under what conditions? If not, why not?
  3. How would you expect the issue of a paper currency to effect foreign trade,—
    1. While the notes were still redeemable;
    2. After they had become irredeemable.

III.
Answer two.

  1. Define the following terms

Seignorage,
Clearing house loan certificates,
Silver Certificates,
United States notes,
Inconvertible paper.

  1. How would the adoption of bimetallism affect the stability of the value of money?
    1. according to Mill,
    2. according to Walker,
    3. in your own opinion.
  2. How is the value of money in a country likely to be affected by an increase in
    1. the quantity of commodities produced and sold,
    2. the quantity of bank notes,
    3. the volume of bank deposits.

Which of these changes would you expect to exercise most influence? Which least? Give your reasons.

IV.
Answer all.

  1. Compare and explain the operations of the Bank of England and those of the New York banks in a time of crisis,
  2. Arrange these items…

Government Securities 40.
Surplus 3.
Notes 38.
Specie 40.
Deposits 55.
Capital 14.
Loans 30.

    1. … in their proper order, as they would stand in an account of the Bank of France.
    2. … as they would stand in an account of a national bank of the United States; and state (1) whether this could be an account of a national bank, and (2) whether the proportions of the different items are such as you would be likely to find in an account of such a bank.
    3. … as they would stand in an account of the Bank of England, assuming the uncovered issue to be 17.

Source: Harvard University Archives. Harvard University, Examination Papers, 1873-1915. Box 5, Bound volume: Examination Papers, 1900-01. Papers Set for Final Examinations in History, Government, Economics, Philosophy, Education, Fine Arts, Architecture, Landscape Design, Music in Harvard College (June, 1901), pp. 21-23.

 

__________________________________

Economics 2.
Economic Theory
in the 19th Century

For Undergraduates and Graduates.

Course outline and readings.

Course Announcement
  1. Economic Theory in the Nineteenth Century. , Wed., Fri., at 2.30 Professor Taussig. [note: Professor Carver taught the course]

Source: Harvard University. Announcement of the Courses of Instruction provided by the Faculty of Arts and Sciences for the Academic Year 1900-1901, p. 41.

Enrollment
  1. Professor Carver. — Economic Theory in the Nineteenth Century.

Total 45: 6 Graduates, 15 Seniors, 16 Juniors, 5 Sophomores, 3 Other.

Source: Harvard University. Annual Reports of the President and Treasurer of Harvard College, 1900-01, p. 64.

1900-01
ECONOMICS 2
[Mid-year examination]
  1. Define value and explain why one commodity possesses more value in proportion to its bulk than another.
  2. Explain the various uses of the term diminishing returns, and define it as you think it ought to be defined.
  3. In what sense does a law of diminishing returns apply to all the factors of production.
  4. State briefly Böhm-Bawerk’s explanation of the source of interest.
  5. What, if any, is the relation of abstinence to interest.
  6. Would you make any distinction between the source of wages and the factors which determine rates of wages? If so, what? If not, why not?
  7. Discuss the question: Is a demand for commodities a demand for labor?
  8. What is the relation of the standard of living to wages.
  9. Discuss briefly the following questions relating to speculators’ profits. (a) Do speculators as a classmake any profits? (b) Are speculators’ profits in any sense earned?
  10. In what sense, if any, does the value of money come under the law of marginal utility?

Source: Harvard University Archives. Harvard University Mid-year Examinations, 1852-1943. Box 4, Bound volume: Examination Papers, Mid-Years, 1900-01.

1900-01
ECONOMICS 2
[Year-end Examination]

Discuss the following topics.

  1. The bearing of the marginal utility theory of value upon the questions of wages and interest.
  2. The definitions of capital as given by Taussig and Clark.
  3. Clark’s explanation of the place of distribution within the natural divisions of economics.
  4. Clark’s method of distinguishing between the product of labor and the product of capital.
  5. Clark’s distinction between rent and interest.
  6. Böhm-Bawerk’s theory of the nature of capital.
  7. The origin of capital, according to Böhm-Bawerk and Clark.
  8. The meaning of the word “productive” in the following proposition: “Protection is an attempt to attract labor and capital from the naturally more productive, to the naturally less productive industries.”
  9. The incidence of tariff duties.
  10. The theory of production and the theory of valuation as the two principal departments of economics.

Source: Harvard University Archives. Harvard University, Examination Papers, 1873-1915. Box 5, Bound volume: Examination Papers, 1900-01. Papers Set for Final Examinations in History, Government, Economics, Philosophy, Education, Fine Arts, Architecture, Landscape Design, Music in Harvard College (June, 1901), pp. 23-24.

 

__________________________________

Economics 3.
Principles of Sociology

For Undergraduates and Graduates.

Course Announcement
  1. The Principles of Sociology. — Development of the Modern State, and of its Social Functions. , Wed., and (at the pleasure of the instructor) Fri., at 1.30. Mr. —.

Source: Harvard University. Announcement of the Courses of Instruction provided by the Faculty of Arts and Sciences for the Academic Year 1900-1901, p. 41.

Enrollment
  1. Asst. Professor [Thomas Nixon] Carver. — The Principles of Sociology. Development of the Modern state, and of its Social Functions.

Total 57: 9 Graduates, 22 Seniors, 8 Juniors, 14 Sophomores, 4 Others.

Source: Harvard University. Annual Reports of the President and Treasurer of Harvard College, 1900-01, p. 64.

1900-01
ECONOMICS 3.
[Mid-year Examination]

Answer only ten questions.

  1. Upon what does Kidd base his argument that religion is necessary to keep men from taking such political action as would suspend economic competition, and what is the crucial point in his argument?
  2. In the light of Kidd’s theory of social evolution, discuss the question: Can there be a permanent civilization? Or, do the conditions which promote progress also ensure decay?
  3. Classify the sanctions for conduct which originate outside the individual and explain your classification.
  4. Explain and illustrate the meaning of the following: “Generalizing this struggle and extending it to every form existing in the social life — linguistic, religious, political, artistic, and moral, as well as industrial — we see that the really fundamental social opposition must be sought for in the bosom of the social individual himself.” (Tarde, Social Laws. Ch. II. p. 83.)
  5. What is meant by social stratification? How does it originate? What are some of its consequences?
  6. Compare Herbert Spencer’s theory of progress with Lester F. Ward’s, giving special attention to the argument which each offers in support of his theory.
  7. What, according to Patten, are the chief obstacles to a progressive evolution.
  8. Explain the following. “The difference between that society of conscious units which we call mind, and a society of human beings on our planet, is in the completeness of the mechanism.” (Patten, Theory of Social Forces. Ch. II. p. 21.)
  9. What, according to Patten, is the significance of the transition from a pain to a pleasure economy.
  10. How does Bagehot account for the origin of national traits?
  11. Discuss the question: Does charity retard the process of race improvement?

Source: Harvard University Archives. Harvard University Mid-year Examinations, 1852-1943. Box 4, Bound volume: Examination Papers, Mid-Years, 1900-01.

1900-01
ECONOMICS 3.
[Year-end Examination]

Discuss the following topics

  1. The definition of progress.
  2. Charity as a factor in human selection.
  3. The way in which, according to Spencer, the different classes of institutions are related to one another.
  4. The sanctions for conduct.
  5. A moral ideal as a factor in human selection.
  6. The natural antagonism of human interests and the problem of evil.
  7. The storing of the surplus energy of society.
  8. The influence of property on the relations of the sexes.
  9. Labor and service as bases of distributive justice
  10. The influence of militarism upon race development, or deterioration.

Source: Harvard University Archives. Harvard University, Examination Papers, 1873-1915. Box 5, Bound volume: Examination Papers, 1900-01. Papers Set for Final Examinations in History, Government, Economics, Philosophy, Education, Fine Arts, Architecture, Landscape Design, Music in Harvard College (June, 1901), p. 24.

 

__________________________________

Economics 5.
Railways and Other Public Works

For Undergraduates and Graduates.

Course Announcements

51 hf. Railways and Other Public Works, under Public and Corporate Management. Half-course (first half-year). Tu., Th. and (at the pleasure of the instructor) Sat., at 1.30. Mr. Meyer.

52 hf. Railways and Other Public Works (advanced course). Half-course (second half-year). Tu., Th. and (at the pleasure of the instructor) Sat., at 1.30. Mr. Meyer.

Source: Harvard University. Announcement of the Courses of Instruction provided by the Faculty of Arts and Sciences for the Academic Year 1900-1901, p. 42.

Enrollments

[Economics] 51 hf. Mr. [Hugo Richard] Meyer.— Railways and other Public Works, under Public and Corporate Management.

Total 86: 4 Graduates, 52 Seniors, 17 Juniors, 4 Sophomores, 9 Others.

[Economics] 52 hf. Mr. Meyer.— Railways and other Public Works (advanced course).

Total 9: 3 Graduates, 4 Seniors, 1 Junior, 1 Sophomore.

Source: Harvard University. Annual Report of the President of Harvard College, 1900-1901, p.64.

1900-01
ECONOMICS 51
[Mid-year Examination]

Omit the last question if the paper seems too long

  1. The construction put upon the long and short haul clause: by the Interstate Commerce Commission; by the Supreme Court.
  2. The decisions of the Interstate Commerce Commission on group rates.
  3. The railway rate situation in Germany [Prussia]; does it throw any light on the railway problem in the United States?
  4. “If pooling produces any beneficial result, it necessarily does so at the expense of competition. It is only by destroying competition that the inducement to deviate from the published rate is wholly removed….By the legalizing of pooling the public loses the only protection which it now has against the unreasonable exactions of transportation agencies.”—Give your reasons for accepting or rejecting this statement.
    Alternative:—
    The reasons for the instability of pools in the United States.
  5. The Iowa Railroad Commission.
    Alternative:—
    To what extent was the long and short haul clause of the Interstate Commerce Act enforced; what was the effect of that enforcement: on railway revenues; on intermediate shipping or distributing points?
  6. The body of administrative law to be found in the decisions of the Massachusetts Gas and Electric Light Commission’s decisions upon petitions for reductions in the price of gas.
  7. (a) Is it to the public interest to insert in street railway charters provisions seeking to secure to the municipality or the state a share in any excess of profit over the normal rate?
    Alternative: (b) and (c).
    (b) The evidence as to the return on capital obtainable in street railway ventures.
    (c) What questions of public policy were raised in the case of the Milwaukee Street Railway and Electric Light Co. vs. The City of Milwaukee?
  8. What statistics were used in illustrating in a general way the statement that railway charges are based upon what the traffic will bear; in discussing the bearing of stock-watering upon railway rates; in discussing the return obtained by capital invested in railway enterprises in the United States?

Source: Harvard University Archives. Examination Papers, 1873-1915. Box 5. Bound Volume: Examination Papers 1900-01. Papers Set for Final Examinations in History, Government, Economics, Philosophy, Education, Fine Arts, Architecture, Landscape Design, Music in Harvard CollegeJune, Pages 24-25.

1900-01
ECONOMICS 52
[Year-end Examination]
  1. The railways and the national finances in Prussia and Australia.
  2. Railway rates and the export trade of the United States since 1893, or, 1896.
  3. The economic situation in Australia since 1892, and the Australian railways.
  4. “A fatal objection to the income or preference bond is that it is an attempt to combine two contradictory commercial principles.”
    Discuss this statement fully. What does it mean? Is it true?
  5. If you had access to all the accounts of a railroad, how should you determine the value to it of one of its branch lines?
  6. To what accounts would you charge the following expenditures? (If you do not remember the exact Interstate Commerce Commission classification, use your best judgment.) State reasons in each case.
    Engineer’s wages on a special train conveying the general manager to an extensive flood covering the line.
    Fireman’s wages on an engine employed exclusively in switching to and from the repair shops.
    Conductor’s wages on a worktrain engaged in taking up rails on an abandoned branch.
    Brakeman’s wages on a train engaged solely in hauling company’s coal for company’s use.
    Cost of taking up comparatively new sound rails judged too light for heavy rolling stock.
    Cost at a competitive point of a new station to replace an old one which was large enough but old-fashioned.
  7. State the commonest problems facing a reorganization committee for an insolvent road, and then suggest and defend one course of procedure for each problem.
  8. Combine and arrange the following items so as to give the best information about the operation and condition of the road. (Do not rewrite the names but use the corresponding numbers where possible.)
1. Passenger train miles 2,000,000
2. Freight train miles 3,400,000
3. Passenger train earnings $2,400,000
4. Freight train earnings $5,500,000
5. Income from investments $100,000
6. Dividends $500,000
7. Operating expenses $4,700,000
8. Av. no. pass. cars per train 4
9. Av. no. passengers per car 11
10. Tons freight carried 2,800,000
11. Av. load per car (loaded and empty), tons 8.2
12. Av. no. loaded cars per train 12.3
13. Av. no. empty cars per train 6.7
14. Interest charge for year $2,200,000
15. Due other roads $100,000
16. Stocks and bonds owned $4,900,000
17. Supplies on hand $500,000
18. Taxes for the year $300,000
19. Accounts receivable $500,000
20. Cash $1,000,000
21. Surplus for the year $300,000
22. Profit and loss account $1,000,000
23. Taxes accrued but not due $100,000
24. Capital stock $50,000,000
25. Interest due $700,000
26. Funded debt $45,000,000
27. Due from other roads $100,000
28. Interest accrued not due $300,000
29. Franchises and property $90,400,000
30. Bonds of the company in its treasury $800,000
31. Accounts payable $1,000,000
32. No. of passengers carried 2,300,000

Source: Harvard University Archives. Examination Papers, 1873-1915. Box 5. Bound Volume: Examination Papers 1900-01. Papers Set for Final Examinations in History, Government, Economics, Philosophy, Education, Fine Arts, Architecture, Landscape Design, Music in Harvard College. June, 1901. Pages 25-27.

 

__________________________________

Economics 6.
Economic History of the U.S.

For Undergraduates and Graduates.

Course Announcement
  1. The Economic History of the United States. Tu., Th., at 2.30, and a third hour at the pleasure of the instructor. Mr. —.

Source: Harvard University. Announcement of the Courses of Instruction provided by the Faculty of Arts and Sciences for the Academic Year 1900-1901, p. 42.

Enrollment
  1. Professor Taussig. — The Economic History of the United States.

Total 164: 9 Graduates, 63 Seniors, 68 Juniors, 13 Sophomores, 11 Others.

Source: Harvard University. Annual Reports of the President and Treasurer of Harvard College, 1900-01, p. 64.

1900-01
ECONOMICS 6
[Mid-year Examination]

Arrange your answers strictly in the order of the questions. Answer all the questions,

  1. The nature and object of the scales of depreciation established by Congress and by the States at the close of the war of the Revolution; and how far these objects were accomplished.
  2. “The year 1789 marks no such epoch in economies as it does in political history.” — Taussig. How far is this true as to (1) financial legislation; (2) tariff legislation; (3) the course of foreign trade; (4) the growth of manufactures?
  3. Explain how you would distinguish Treasury Notes designed to circulate as currency from those designed simply to meet financial needs; and state when and under what circumstances, between 1789 and 1860, the United States resorted to issues of the first kind.
  4. Suppose the charter of the first Bank of the United States had been renewed: would the effect have been favorable or unfavorable for the finances of the government, for the bank, for the community, in 1812-1815?
  5. Suppose the charter of the second Bank of the United States had been renewed: would the effect have been favorable or unfavorable for the finances of the government, for the bank, for the community, in 1835-40?
  6. Describe the Independent Treasury system, as first established and as finally settled (give dates). Do you believe it better than the alternative system proposed by its opponents? Why?
  7. The causes of the crises of 1837 and 1857: wherein similar, wherein different.
  8. State what were the duties on cotton goods in 1809, 1819, 1839, 1859; and give your opinion whether the duties at these several dates were designed to give protection, and whether protection was then expedient.
  9. Why the early railway enterprises of the States were undertaken as public enterprises; and how far their history may be fairly cited for or against the policy of public management.

Source: Harvard University Archives. Harvard University Mid-year Examinations, 1852-1943. Box 4, Bound volume: Examination Papers, Mid-Years, 1900-01.

1900-01
ECONOMICS 6
[Year-end Examination]

Arrange your answers strictly in the order of the questions

  1. Explain summarily at what dates and to what extent land-grants and bond-subsidies were extended to railways by the United States; and state whether you believe these measures brought advantage to the country.
  2. Was the management of the finances during the Civil War fraught with more or less evil consequences than that during the War of 1812, as regards (1) the currency, (2) the banks?
  3. State what main sources of revenue were expected to be used, what were used in fact, by the United States in each of the years 1862, 1863, 1864; and explain how the resort to the sources actually used came about.
  4. For the decade 1870-80, explain the connection between the course of prices, foreign trade, railway operations, and currency legislation.
  5. For the decade 1880-90, connect the history of the public debt, the national revenues, the banking system, the silver currency.
  6. Does the argument for protection to young industries find support in the history of (1) the cotton manufacture before 1830, (2) the silk manufacture since 1870, (3) the tin plate industry since 1890.
  7. Explain how the theory of comparative costs may be applicable to the present situation as regards carpet wool, beet sugar, glassware, woollen cloths (take three).
  8. What changes were made in the duties on raw and refined sugar in 1890, 1894, 1897? Which mode of treatment do you regard the most advisable, and why?
  9. State what causes you believe to have chiefly promoted the growth and maintenance of the sugar and oil combinations; and consider which of these two you regard as typical, and as instructive for forecasting the future of combinations.

Source: Harvard University Archives. Examination Papers, 1873-1915. Box 5. Bound Volume: Examination Papers 1900-01. Papers Set for Final Examinations in History, Government, Economics, Philosophy, Education, Fine Arts, Architecture, Landscape Design, Music in Harvard College. June, 1901. Pages 27-28.

 

__________________________________

Economics 81
Money

For Undergraduates and Graduates.

Course Announcement

81 hf. Money. Half-course (first half-year). Tu., Th., Sat., at 11. Mr. Andrew.

Source: Harvard University. Announcement of the Courses of Instruction provided by the Faculty of Arts and Sciences for the Academic Year 1900-1901, p. 42.

Enrollment

81 hf. Dr. [Abram Piatt] Andrew. Money.

Total 122: 3 Graduates, 56 Seniors, 41 Juniors, 8 Sophomores, 1 Freshman, 13 Others.

Source: Harvard University. Annual Reports of the President and Treasurer of Harvard College, 1900-01, p. 64.

1900-01
ECONOMICS 8
[Mid-year Examination]

Answer only three questions from each group, but consider the questions strictly in the order of their arrangement 

I

  1. What is meant by

(1) a “double” standard;
(2) a “parallel” standard;
(3) a “limping” standard;
(4) a “single” standard;

Cite at least two historic examples of each, giving approximate dates.

  1. The following are estimates which have been made of the average production of silver, and its annual average export to the Orient in millions of ounces:

Production Export to East
1851-55 28 mill.

20 mill.

1855-60

29  ” 52  ”
1861-65 35  ”

53  ”

1865-70

43  ”

25  ”

State the causes of the singular situation revealed in these figures, and explain its actual effect upon the relative values of gold and silver in Europe.

  1. Suppose that the British government in 1870 had used the right conferred by the act of 1816, and had proclaimed the free coinage of silver at the ratio then current. What differences do you think would have occurred in the subsequent currency history of the world?
  2. Describe the effect of the suspension of the coinage of silver upon the value of the currency in each of the following cases:—
    (1) in Holland; (2) in Austria; (3) in Russia; (4) in India.

II

  1. “Before 1873 we had coined in the United States only about eight million silver dollars ($8,031,238) while since the date fixed as the beginning of demonetization we have coined nearly five hundred millions ($485,427,703).”
    How do you explain (1) the small amount of dollars coined before 1873? (2) the large amount coined since then?
  2. What in your opinion was the real significance of (1) the act of 1803? (2) the act of 1873?
  3. “With the exception of the brief period of fifteen years (1544-60) the English coins have never been debased.”
    In what sense and to what extent is this statement correct?
  4. In writing of the currency history of England during the years, immediately succeeding the great recoinage (1696) Mr. Dana Horton says:—
    “And so the full weight standard coin of the Realm, to create a stock of which the State had spent a sum greater than its regular annual revenue, and equal to perhaps a fourth of the country’s total stock of cash, — was allowed to find its way back to the melting-pot in exchange for cheaper gold.”
    Explain the situation to which he refers, and the reasons for this disappearance of the “standard coin.”

III

  1. (a) What were the main arguments which Lord Liverpool advanced in favor of a single gold standard?
    (b) What were the legislative acts in which his influence is to be traced?
  2. (a) Do falling prices necessarily mean an increase in the burden of debts?
    (b) Do they in the long run inevitably diminish the productiveness of industry?
    (c) To what extent are they prejudicial to the interests of the working classes?
  3. “It is generally agreed that every fall in the value of silver acted at the time as a stimulus to Indian exports and as a check on imports into India.”
    (1) Explain this statement, (2) state how far it is confirmed by commercial statistics, and (3) show whether such a condition is ever likely to be of prolonged duration.
  4. It is alleged that the Russian government, by stimulating exports, and hindering imports, has endeavored to secure a favorable balance of trade, with the idea of increasing the quantity of gold in the country? What do you think would be the ultimate effect of such a policy if continuously pursued?

Source: Harvard University Archives. Harvard University Mid-year Examinations, 1852-1943. Box 4, Bound volume: Examination Papers, Mid-Years, 1900-01.

Also: Harvard University Archives. Examination Papers, 1873-1915. Box 5. Bound Volume: Examination Papers 1900-01. Papers Set for Final Examinations in History, Government, Economics, Philosophy, Education, Fine Arts, Architecture, Landscape Design, Music in Harvard College (June, 1901), pp. 28-30.

 

__________________________________

Economics 9.
The Labor Question in Europe and the U.S.

For Undergraduates and Graduates.

Taught by W. F. Willoughby (Edward Cummings’ successor).

Course Announcement
  1. The Labor Question in Europe and the United States. — The Social and Economic Condition of Workingmen. Tu., Th., Sat., at 10. Mr. —.

Source: Harvard University. Announcement of the Courses of Instruction provided by the Faculty of Arts and Sciences for the Academic Year 1900-1901, p. 42.

Enrollment

92 hf. Mr. W. F. Willoughby. — The Labor Question in Europe and the United States. The Social and Economic Condition of Workingmen.

Total 146: 3 Graduates, 53 Seniors, 40 Juniors, 35 Sophomores, 3 Freshmen, 12 Others.

Source: Harvard University. Annual Reports of the President and Treasurer of Harvard College, 1900-01, p. 64.

1900-01
ECONOMICS 9
[Year-end examination]
  1. Show how the change in the organization of industry from the handicraft system and production on a small scale to the factory system and production on a large scale has led to; (a) efforts to supplant the wages system by socialism[,] coöperation, etc., (b) the trade union movement, and (c) compulsory compensation acts.
  2. Give the arguments for and against profit-sharing as regards (a) it being a more just system of enumeration than the wages system, and (b) its practical advantages.
  3. What are the two systems of coöperative production now practice in Great Britain, and why are they meeting with more success than earlier efforts?
  4. Describe the trade agreement between the National Metal Trades Association and the International Association of Machinists in such a way as to show its essential character and significance, and particularly its relation to the trade union movement and the question of the prevention and adjustment of industrial disputes.
  5. What was the nature of the “new unionism” movement in Great Britain, and its success?
  6. What is the general character of the Massachusetts State Board of Conciliation and Arbitration; what its duties and its powers?
  7. Describe the essential features of the French Workmen’s Compensation Act.
  8. Give a brief sketch of the Social Democratic Party in Germany, with the names of its early leaders and important events in its history.
  9. In what ways can the municipality take action for the improvement of the housing condition of the poorer classes without itself building tenements? What are some of the objections to the municipalities themselves undertaking building operations?
  10. Show why employment bureaus can do but little for the solution of the general problem of the unemployed.

Source: Harvard University Archives. Examination Papers, 1873-1915. Box 5. Bound Volume: Examination Papers 1900-01. Papers Set for Final Examinations in History, Government, Economics, Philosophy, Education, Fine Arts, Architecture, Landscape Design, Music in Harvard College (June, 1901), pp. 30-31.

Enrollment (Economics 9a)

9a2 hf. Mr. W. F. Willoughby. — Provident Institutions. Workingmen’s Insurance, Friendly Societies, Savings Banks.

Total 22: 1 Graduate, 13 Seniors, 5 Juniors, 2 Sophomores, 1 Other.

Source: Harvard University. Annual Reports of the President and Treasurer of Harvard College, 1900-01, p. 64.

1900-1901
ECONOMICS 9a
[Year-end Examination]
  1. What is the general situation in France at the present time in respect to insurance against old age and invalidity? Describe briefly the organization and workings of important institutions, and show particularly how the government is attempting to further this kind of insurance.
  2. What has been the general policy of the British government in respect to the regulation of Friendly Societies? Give the main features of law now regulating them.
  3. Describe the Fraternal Beneficial Orders of the United States as regards (a) their general scheme of organization, and (b) system of insurance.
  4. Show wherein this insurance system is defective by contrasting it with that of ordinary life insurance companies: indicate reforms that are necessary and how they can best be brought about.
  5. Contrast the systems of savings banks in Great Britain and the United States.
  6. In what respects are coöperative banks of the Schulze-Delitzsch and Raiffeisen type more valuable social institutions than the ordinary savings banks?
  7. Describe the principles upon which all coöperative building and loan associations in this country are organized, and indicate ways in which they might profitably be subjected to more rigid state control.
  8. Why is it impracticable to insure against unemployment?
  9. Outline briefly the system of sick insurance in Germany.

Source: Harvard University Archives. Examination Papers, 1873-1915. Box 5. Bound Volume: Examination Papers 1900-01. Papers Set for Final Examinations in History, Government, Economics, Philosophy, Education, Fine Arts, Architecture, Landscape Design, Music in Harvard College (June, 1901), p. 31.

 

__________________________________

Economics 10.
Mediaeval Economic History of Europe.

For Undergraduates and Graduates.

Course Announcement

For Undergraduates and Graduates.

  1. The Mediaeval Economic History of Europe. Tu., Th., and (at the pleasure of the instructor) Sat., at 12. Professor Ashley.

Source: Harvard University. Announcement of the Courses of Instruction provided by the Faculty of Arts and Sciences for the Academic Year 1900-1901, p. 41.

Enrollment
  1. Professor Ashley. The Mediaeval Economic History of Europe.

Total 11: 6 Graduates, 4 Seniors, 1 Junior.

Source: Harvard University. Annual Reports of the President and Treasurer of Harvard College, 1900-01, p. 64.

1900-01
ECONOMICS 10
[Mid-year Examination]

Not more than six questions should be attempted, of which the first should be one.

  1. Translate, and briefly comment upon
    1. Toto regis Willelmi primi tempore perseveravit haec institutio, usque tempora regis Henrici filii ejus; adeo ut viderim ego ipse quosdam qui victualia statutis temporibus de fundis regiis ad curiam deferri viderint.
    2. In Kateringes sunt X hidae ad geldum Regis. Et de istis X hidis tenent XL villani XL virgas terrae.
    3. Compotus Roberti Oldeman praepositi de Cuxham, ab in crastino Sancti Jacobi anno regni Regis Edwardi filii Regis Edwardi decimo usque ad in crastinum Sancti Jacobi proxime sequentis anno regni Regis Edwardi praedicti undecimo intrante.
    4. Rogamus . . . ademptum sit jus etiam procuratoribus nedum conductori adversus colons ampliandi partes agrarias aut operarum praebitionem jugorumve.
    5. Orgeterix ad judicium omnem suam familiam, ad hominum milia decem, undeque coëgit et omnes clientes obaeratoque suos quorum magnum numerum habebat eodem conduxit.
  2. What materials have we for forming a judgment as to the position of the rural population of England in the period from the eleventh to the fourteenth centuries? Classify them, and indicate the value of each class for the purposes of this enquiry.
  3. Wherein did the status of the coloni of the later Roman Empire resemble or differ from that of the medieval villein?
  4. Describe the constitution and working of manorial courts. What light does their history throw on the evolution of social classes?
  5. “Wie das Wort Dorf … dem Sinne nach einen Haufen bezeichnet, so ist auch haufenförmig oder Haufendorf der geeignetste Ausdruck für diese Art der Dorfenlage.” Explain and comment.
  6. “M. Fustel took for his point of departure the Provincial villa; Dr. Hildebrand takes the Kirghises of modern Asia.” Explain, and then show the peculiar dangers of each method.
  7. “We may safely follow Palgrave in taking the Anglo-Saxon townships as the integral molecules out of which the Anglo-Saxon State was formed.” Why? or why not?
  8. What was the gwely? What bearing has it on the general problem of “tribal” organization?
  9. What are the assumptions or postulates of modern Political Economy? To what extent were they true of the Middle Ages?
  10. Which book read in connection with this course has interested you most? Describe its method and estimate the value of its contribution to economic history.

Source: Harvard University Archives. Harvard University Mid-year Examinations, 1852-1943. Box 4, Bound volume: Examination Papers, Mid-Years, 1900-01.

1900-01
ECONOMICS 10
[Year-end Examination]

Not more than six questions should be attempted, of which the first should be one

  1. Briefly comment upon the following passages, and translate such of them as are not in English:—
    1. Colunt discreti ac diversi, ut fons, ut campus, ut nemus placuit. Vicos locant non in nostrum morem connexis et cohaerentibus aedificiis: suam quisque domum spatio circumdat.
    2. If a man agree for a yard of land or more, at a fixed rent, and plough it; if the lord desire to raise the land to him to service and to rent, he need not take it upon him, if the lord do not give him a dwelling.
    3. Ego S. … et ego P. … aliquantulam agri partem pro remedio animarum nostrorum W. episcopo in dominio donare decrevimus; id est xxx cassatorum in loco qui dicitur T.
    4. Si quis super alterum in villa migrare voluerit, et unus vel aliqui de ipsis qui in villa consistunt eum suscipere voluerit, si vel unus extiterit qui contradicat, migrandi ibidem licentiam non habebit.
    5. Qui habebant de tenentibus per diaetas totius anni, ut assolet de nativis, oportebat eos relaxare et remittere talia opera.
    6. If any one does an injury who is not of the gild and is of the franchise … he shall lose his franchise.
  2. Explain the position of Maitland’s Domesday Book and Beyond in the discussion concerning the origin of the manor.
  3. Distinguish between the several characteristics of mediaeval towns, and indicate the part played by each, in your opinion, in the formation of specifically urban conditions.
  4. Examine the relations between questions of personal status and questions of economic condition in relation to the ‘peasants’ of the Middle Ages.
  5. What is the nature of our evidence as to the Peasants’ Rising of 1381? Is there any reason for ascribing anything like an economic programme to the leaders of the movement?
  6. Indicate briefly (a), the several influences tending towards a corporate organization of industry in the later Middle Ages and (b) the advantages or disadvantages of such an organization.
  7. Distinguish between the several immigrations of foreign work people to England before the accession of James I, and explain the nature of their contributions to the development of English manufactures.
  8. The relation of John Major and Juan Vives to the development of the English ‘Poor Law.’
  9. What changes, if any, did the Reformation bring about in social life?

Source: Harvard University Archives. Examination Papers, 1873-1915. Box 5. Bound Volume: Examination Papers 1900-01. Papers Set for Final Examinations in History, Government, Economics, Philosophy, Education, Fine Arts, Architecture, Landscape Design, Music in Harvard College (June, 1901), pp. 32-33.

 

__________________________________

Economics 122.
Banking and the History of the Leading Banking Systems.

For Undergraduates and Graduates.

Course Announcement

122 hf. Banking and the history of the leading Banking Systems. Half-course (second half-year). Tu., Th., Sat., at 11. Dr. Sprague.

Source: Harvard University. Announcement of the Courses of Instruction provided by the Faculty of Arts and Sciences for the Academic Year 1900-1901, p. 43.

Enrollment

122 hf. Dr. [Oliver Mitchell Wentworth] Sprague. — Banking and the History of the Leading Banking Systems.

Total 128: 4 Graduates, 51 Seniors, 43 Juniors, 16 Sophomores, 14 Others.

Source: Harvard University. Report of the President of Harvard College, 1900-1901, p. 64.

1900-01
ECONOMICS 122
[Year-end Examination]

Arrange your answers strictly in the order of the questions. Answer all the questions under A and two of those under B

A

  1. Explain in detail and under different circumstances the effect of an advance of the rate of discount by the Bank of England upon the money market of London and upon the foreign exchanges.
  2. Taking the separate items of a bank account point out how those of the Bank of Amsterdam differed from those of a modern bank.
  3. Define and explain:—
    1. Bill broker.
    2. Banking Principle.
    3. The State Bank of Indiana.
    4. The banking law of Louisiana.
    5. Clearing House Certificates.
  4. The extent and banking consequences of government control of the Bank of France and the Reichsbank.
  5. How do government receipts and expenditures affect the money market (a) of London, (b) of New York?
  6. Explain with illustrations from the crises of 1857 and 1893 the nature of the demand for cash in time of crisis, and consider how far that demand may be met under a flexible system of note issue.

B

  1. (a) How far and with what qualifications may banking experience in the United States before 1860 be appealed to in the discussion of changes in our banking system? (b) How far, similarly, may Canadian experience be applied?
  2. “Why compel banks to send home for redemption a multitude of notes which can as well be used in payments and are sure to be reissued at once? Why impede the free use of its power of circulation by any enterprising bank by requiring the early redemption of notes which the holder does not in fact care or need to have redeemed?”
    Explain from past experience what regulations may be expected to bring about these results, and give the reasons for demanding them.
  3. Discuss the question of branch banking with reference to the United States, including in your discussion considerations of safety and economy. Would branch banking be more desirable than at present if notes were issued against general banking assets.

Source: Harvard University Archives. Examination Papers, 1873-1915. Box 5. Bound Volume: Examination Papers 1900-01. Papers Set for Final Examinations in History, Government, Economics, Philosophy, Education, Fine Arts, Architecture, Landscape Design, Music in Harvard College (June, 1901), pp. 34-35.

 

__________________________________

Economics 12a1.
International Payments and the Flow of Precious Metals.

For Undergraduates and Graduates.

Course Announcement

12a2 hf. International Payments and the Flow of the Precious Metals. Half-course (second half-year). Three times a week. Mr—.

Source: Harvard University. Announcement of the Courses of Instruction provided by the Faculty of Arts and Sciences for the Academic Year 1900-1901, p. 43.

Enrollment

[Economics] 12a1 . Mr. [Hugo Richard] Meyer.—International Payments and the Flow of the Precious Metals.

Total 16: 2 Graduates, 9 Seniors, 4 Juniors, 1 Other.

Source: Harvard University. Report of the President of Harvard College, 1900-1901, p. 64.

1900-01
ECONOMICS 12a1.
[Mid-Year Examination]

Observe strictly the order in which the questions are arranged.

  1. Sidgwick’s criticisms on Mill’s doctrine of international trade and their validity.
  2. What temporary changes in the general level of prices in this country should you expect to see, as the result of a large permanent withdrawal of foreign capital? What ultimate change of prices should you expect?
  3. Suppose the exportation of specie from the United States to be prohibited (or, as has sometimes been suggested, to be slightly hindered), what would be the effect on rates of exchange, and on prices of goods, either domestic or foreign? Would the country be a loser or not? [See Ricardo (McCulloch’s ed.), page 139.]
  4. The conditions which led to the flow of gold to the United States in the fiscal years 1880 and 1881?
  5. What economic conditions or events tended to make the year 1890 a turning point both in domestic and in international finance?

Alternative:

The reasons for the return flow from Europe of American securities in the years 1890-1900?

  1. What sort of wealth did France actually sacrifice in paying the indemnity? What was the process?
  2. Is Mr. Clare justified in making the general statement that “the gold-points mark the highest level to which an exchange may rise, and the lowest to which it may fall”?
  3. Why is it that certain trades bills are drawn chiefly, or even exclusively, in one direction, e.g. by New York on London and not vice versa; and how is this practice made to answer the purpose of settling payments which have to be made in one direction?

Alternative:

Why has England become the natural clearing-house for the world?

Source: Harvard University Archives. Harvard University Mid-year Examinations, 1852-1943. Box 4, Bound volume: Examination Papers, Mid-Years, 1900-01.

Also: Harvard University Archives. Examination Papers, 1873-1915. Box 5. Bound Volume: Examination Papers 1900-01. Papers Set for Final Examinations in History, Government, Economics, Philosophy, Education, Fine Arts, Architecture, Landscape Design, Music in Harvard College (June, 1901), pp. 33-34.

 

__________________________________

Economics 13.
Methods of Economic Investigation.

Primarily for Graduates.

Course Announcement
  1. Methods of Economic Investigation.—English Writers. German Writers. Tu., Th., at 1.30. Professor Taussig.
    Courses 15 and 13 are usually given in alternate years.

[15. The History and Literature of Economics to the close of the Eighteenth Century. Mon., Wed., and (at the pleasure of the instructor) Fri., at 12. Professor Ashley.
Omitted in 1900-01.]

Source: Harvard University. Announcement of the Courses of Instruction provided by the Faculty of Arts and Sciences for the Academic Year 1900-1901, p. 43.

Enrollment
1900-01

Economics 132 hf. Asst. Professor Carver. — Methods of Economic Investigation.

Total 10: 4 Graduates, 6 Seniors.

Source: Harvard University. Report of the President of Harvard College, 1900-1901, p. 64.

1900-01
ECONOMICS 13
[Year-end Examination]

Discuss ten of the following topics.

  1. The subdivision of economics into departments.
  2. The fields for the observation of economic phenomena.
  3. The place of historical and statistical research in economic investigation.

4, 5, 6. The methods of investigating:

    1. The causes of poverty.
    2. The effect of immigration on the total population of the United States.
    3. The effect of protection on the production of flax fibre, on the iron industry, or on any other industry which you may select.
  1. The nature of an economic law.
  2. The relation of the theory of probabilities to economic reasoning.
  3. The use of hypotheses in economic reasoning.
  4. The use of the terms “static” and dynamic in economic discussion.
  5. The use of diagrams and mathematical formulae in economic discussion.

Source: Harvard University Archives. Examination Papers, 1873-1915. Box 5. Bound Volume: Examination Papers 1900-01. Papers Set for Final Examinations in History, Government, Economics, Philosophy, Education, Fine Arts, Architecture, Landscape Design, Music in Harvard College (June, 1901), p. 35.

 

__________________________________

Economics 17.
Economic Organization and Resources of European Countries.

For Undergraduates and Graduates.

Course Announcement
  1. The Economic Organization and Resources of European countries. Mon., Wed., and (at the pleasure of the instructor) Fri., at 12. Professor Ashley.

Source: Harvard University. Announcement of the Courses of Instruction provided by the Faculty of Arts and Sciences for the Academic Year 1900-1901, p. 42.

Enrollment
  1. Professor Ashley. The Economic Organization and Resources of European countries.

Total 34: 5 Graduates, 14 Seniors, 9 Juniors, 3 Sophomores, 3 Others.

Source: Harvard University. Annual Reports of the President and Treasurer of Harvard College, 1900-01, p. 64.

1900-01
ECONOMICS 17
[Mid-year Examination]

Not more than eight questions should be attempted

  1. “It is less important for a particular community than ever it was to be in possession of cheap food and raw materials produced within its own domain.” Discuss this proposition.
  2. Describe very briefly the main features of the physical geography of England (illustrating your answer, if possible, with a map) and indicate in general terms their economic consequences.
  3. Set forth some of the general considerations which should be taken into account in answering the question whether the industrial development of Ireland has been injuriously affected by English legislation.
  4. Compare the number and character of the several classes maintained by agriculture in England, with those of the agricultural classes in the U.S. and on the continent of Europe.
  5. Explain the powers of dealing with his estate enjoyed at present by an English tenant for life under a settlement.
  6. What districts of England are now suffering most severely from agricultural depression, and why?
  7. Can any lessons be drawn for the U.S. from the recent history of productive coöperation in England? Give your reasons.
  8. Give a rapid survey of the apparent coal resources of the world.
  9. What points of especial interest are there to the economist in the history, situation, character, etc. of the South Wales Coal Field?
  10. What is meant by Collective Bargaining? What are its prerequisites? What examples of it are you acquainted with in America?

Source: Harvard University Archives. Harvard University Mid-year Examinations, 1852-1943. Box 4, Bound volume: Examination Papers, Mid-Years, 1900-01.

1900-01
ECONOMICS 17
[Year-end Examination]

Not more than eight questions should be attempted

  1. The British Chancellor of the Exchequer proposes to levy a duty of one shilling per ton upon the export of coal from the United Kingdom: He argues that the tax will not be borne by the producer, but mainly, if not wholly, by the foreign consumer. Consider (a) what are the conditions under which this is likely to be the case, (b) how far these conditions are at present realized in the case of England.
  2. Distinguish the successive stages in the technological history of iron and steel, and connect them with the industrial development of the several countries concerned.
  3. What were the questions at issue in England in the Engineering dispute of 1897? What, with your present knowledge, do you think ought to have been your attitude, had you then been (a) an English engineering employer, (b) a leading official of the employees’ union.
  4. Give a brief account of the organization of the English cotton manufacture (as distinguished from the securing either of the material or of a market for the product). Contrast it with American conditions; and consider how England and New England are likely to be affected by the growth of the manufacture in the Southern States.
  5. Distinguish between the several forms of capitalist combination at present to be observed in England. What general causes have led to the movement? What, if any, advantages does it promise, and what, if any, dangers does it threaten?
  6. Compare Bradford and Roubaix in any aspects which seem to you worthy of attention.
  7. “Lorsque il n’y a point d’hommes riches qui aient de gros capitaux à mettre dans les entreprises d’agriculture, lorsque les récoltes ne suffisent pas pour assurer aut entrepreneurs des profits égaux à ceux qu’ils tireraient de leur argent en l’employant de toute autre manière, on ne trouve point de fermiers qui veuilient louer les terres. Les propriétaires sont forcées de les faire cultiver par les métayers hors d’état de faire aucunes avances et de bien cultiver. Le propriétaire fait lui-même des avances médiocres qui lui produisent un très médiocre revenu.”
    Translate the passage from Turgot; and then consider how far his description applies to existing conditions in France and Italy.
  8. Show the relation of the great manufacturing industries of France to the distribution of coal in that country.
  9. Would the construction of the Rhine-Elbe canal be a benefit to Germany? Give your reasons.
  10. “Wir müssen uns Rechenschaft ablegen, ob ohne eine grössere Macht zur See, ohne eine solche die unsere Küsten vor Blockaden schützt, unseren Kolonialbesitz und unseren Welthandel absolut sicher stellt, unsere wirtschaftliche Zukunft gesichert sei.”
    Are there sufficient reasons in the contemporary situation of Germany for this anxiety on the part of Professor Schmoller?
  11. (a) Give a brief account of the contents, and then (b) compare the method and general attitude toward the subject, of von Schulze-Gaevernitz’s Social Peace and de Rousers’ Labour Question in Britain.
  12. What in the light of the experience in the English coal, iron, and cotton industries, would seem to you the most satisfactory form to be taken by joint wage agreements in the great industries of America?

Source: Harvard University Archives. Examination Papers, 1873-1915. Box 5. Bound Volume: Examination Papers 1900-01. Papers Set for Final Examinations in History, Government, Economics, Philosophy, Education, Fine Arts, Architecture, Landscape Design, Music in Harvard College (June, 1901), pp. 36-37.

 

__________________________________

Economics 18.
Principles of Accounting.

For Undergraduates and Graduates.

Course Announcement

181 hf. The Principles of Accounting. — Lectures, discussions, and reports. Half-course (first half-year). Mon., Wed., and (at the pleasure of the instructor) Fri., at 3.30. Mr. W. M. Cole.

Source: Harvard University. Announcement of the Courses of Instruction provided by the Faculty of Arts and Sciences for the Academic Year 1900-1901, p. 43.

Enrollment

181 hf. Mr. W. M. Cole. — The Principles of Accounting.

Total 56: 43 Seniors, 4 Juniors, 2 Sophomores, 7 Others.

Source: Harvard University. Annual Reports of the President and Treasurer of Harvard College, 1900-01, p. 64.

1900-01
ECONOMICS 18
[Mid-year Examination]

Problems 1 to 5 inclusive form a connected whole;
but 6
and 7 may be substituted for 4 and 5

I

  1. Construct a rough ledger (omitting rulings and index-memoranda) to correspond with the following trial-balance:
Real estate $150,000 Proprietor $244,275
Plant $60,000 Merchandise $401,000
Patents $40,000 Rent $6,000
Supplies $228,000 Bills payable $14,000
Wages $127,000 Accounts payable $43,000
Coal $9,000 Reserve fund $12,000
Insurance $4,500
Trade discounts $8,000
Interest $1,500
Bills receivable $10,000
Accounts receivable $68,000
Cash $14,275
$720,275 $720,275
  1. The above trial-balance is supposed to be taken from manufacturing books that are kept on the ordinary commercial plan, i.e., without distinctive accounts for stores, manufacturing, stock, or trading; and to construct such accounts now is supposed to be either impossible or undesirable.
    If you were required to determine profit and loss for the year which these figures cover, what questions about the business should you wish to ask before reaching your conclusions? [Give your answer in the form of questions consecutively numbered.]
  2. State what would be fairly reasonable answers to your own questions above numbering the answers to correspond with the questions; and then, assuming your answers to be the real answers show a complete statement of resources and liabilities and of profit and loss.
  3. Close for the year the ledger that you constructed indicating all balances that you have transferred to other accounts and all balances that you have carried down for the new year.
  4. From the ledger as it now stands draw off a balance sheet showing the condition of the business at the beginning of the new year, assuming that the loss or gain is carried directly to the proprietor’s account.
  5. Journalize the following:

A gives you his note for $100, bearing interest, dated a month ago.
You discount at a bank a note for $100 payable in a month
B gives you A’s note for $100 payable in one month, and buy goods for $100 on one month’s time.
Your book-keeper charged bills receivable and credited B when B paid his bill by your own note returned to you. A counter entry is to be made, so that the original wrong entry need not be erased

  1. What is the distinguishing feature of double entry? Are two postings made for every entry? If not, what devices are employed for reducing the number of postings?

II
Omit one

  1. The balance sheet of a corporation on January 1, 1899, stood as follows:
Real estate $50,000 Capital stock $200,000
Plant $95,000 Accounts payable $20,000
Horses, etc. $15,000 Bills payable $25,000
Patents $20,000 Profit and loss $15,000
Merchandise $30,000
Accounts receivable $30,000
Cash $20,000
$260,000 $260,000

On January 1, 1900, the books showed the following facts:

Real estate $55,000 Capital stock $200,000
Plant $88,000 Accounts payable $12,000
Horses, etc. $12,000 Bills payable $17,000
Patents $19,000 Profit and loss $33,000
Merchandise $42,000
Accounts receivable $28,000
Cash $18,000
$262,000 $262,000

What has become of the profits earned?

Should you recommend that a dividend be declared? State your reasons.

  1. How should you treat interest received on a bond bought above par?
  2. Describe the following, and state the distinguishing feature of each: a real account; a nominal account; a suspense account; reserve fund: a sinking fund
  3. If payments are received on account of goods in process of manufacture, should such payments appear on the balance sheet? If so, where?
  4. Describe three different methods of treating depreciation, and show how each would appear upon the books. To what circumstances on a railroad is each adapted?
  5. A corporation is formed to unite and continue the business of three concerns, A, B, and C, engaged in the same industry. The books of the concerns show the following:
A B C
Assets (valuation) $80,000 $160,000 $120,000
Liabilities (external) $20,000 $80,000 $90,000
Average profit, last three years 10% 14% 30%
Average profit, preceding three years 9 17 25
Average profit, prior three years 10 20 20

On what basis should you determine the total amount of capital stock to be issued by the new corporation, and on what basis should you apportion it to these three concerns?

Source: Harvard University Archives. Harvard University Mid-year Examinations, 1852-1943. Box 4, Bound volume: Examination Papers, Mid-Years, 1900-01.

 

__________________________________

Economics 19.
General View of Insurance.

Primarily for Graduates.

Course Announcement

192 hf. A General View of Insurance. — Lectures and reports. Half-course (second half-year). Mon., Wed., and (at the pleasure of the instructor) Fri., at 3.30. Professor Wambaugh.
Course 19 cannot be counted towards the degree of A.B.

Source: Harvard University. Announcement of the Courses of Instruction provided by the Faculty of Arts and Sciences for the Academic Year 1900-1901, p. 43.

Enrollment

192 hf. Professor Wambaugh. — A General View of Insurance.

Total 9: 6 Seniors, 3 Others.

Source: Harvard University. Annual Reports of the President and Treasurer of Harvard College, 1900-01, p. 64.

1900-01
ECONOMICS 192
[Year-end Examination]

One of the paragraphs may be omitted.

  1. From the point of view of the person procuring the policy, what is the purpose of insurance?
  2. From the point of view of the community, what are the advantages and the disadvantages of insurance?
  3. Give some account of three insurance books, pamphlets, or periodicals.
  4. Tell what you know of the history of insurance.
  5. Give a classification of the provision of the New York standard form of fire insurance policy,
  6. If either party to the fire insurance contract wishes to terminate the insurance, what are his rights?
  7. What are the benefits and the dangers of fire insurance by government?
  8. Describe ordinary life policies, single payment life policies, twenty payment life policies, endowment policies, tontine policies, assessment insurance.
  9. If a person thirty years of age wishes to obtain a life insurance policy for a single premium, how is the premium calculated?
  10. What are the chief differences between fire insurance and marine insurance?
  11. Discuss any insurance topic of which you have made a special study. 

Source: Harvard University Archives. Examination Papers, 1873-1915. Box 5. Bound Volume: Examination Papers 1900-01. Papers Set for Final Examinations in History, Government, Economics, Philosophy, Education, Fine Arts, Architecture, Landscape Design, Music in Harvard College (June, 1901), p. 40.

 

__________________________________

Economics 20d.
Adam Smith and Ricardo.

Primarily for Graduates.

Course Announcement

20d. Adam Smith and Ricardo. Half-course. Professor Taussig.

Source: Harvard University. Announcement of the Courses of Instruction provided by the Faculty of Arts and Sciences for the Academic Year 1900-1901, p. 43.

Enrollment

20d1 hf. Professor Taussig. — Adam Smith and Ricardo.

Total 12: 7 Graduates, 5 Seniors.

Source: Harvard University. Annual Reports of the President and Treasurer of Harvard College, 1900-01, p. 64.

1900-01
ECONOMICS 20d
[Final examination]
  1. Compare Ricardo’s conclusions with Adam Smith’s on the course of wages, profits, and rent, as society advances: discussing not only the conclusions themselves, but the reasoning by which the two writers arrive at them.
  2. Under what circumstances are real wages high, according to Adam Smith? according to Ricardo?
  3. Adam Smith’s doctrine on labor as a measure of value; Ricardo’s strictures thereon; and Ricardo’s own doctrine.
  4. S. Mill in his Autobiography says that “it was one of my father’s main objects to make me apply to Smith’s more superficial view of political economy the superior lights of Ricardo, and to detect what was fallacious in Smith’s arguments or erroneous in his conclusions.” Set forth how you believe the two Mills (father and son) set about this task as to Adam Smith’s reasoning on the following topics:—
    1. the mode in which the payment of heavy foreign obligations is brought about by the exportation of goods, not by the outflow of specie;
    2. the distinction between that land which always affords rent, and that which sometimes does and sometimes does not;
    3. the effect of foreign trade in raising the general rate of profits in a country.
  5. “That able but wrong-headed man, David Ricardo; shunted the car of Economic science on to a wrong line, a line, however, on which it was further urged by his equally able and wrong-headed admirer, John Stuart Mill.” — W. S. Jevons.
    What grounds are there for assenting to this judgment? What grounds for dissenting from it?

Source: Harvard University Archives. Examination Papers, 1873-1915. Box 5. Bound Volume: Examination Papers 1900-01. Papers Set for Final Examinations in History, Government, Economics, Philosophy, Education, Fine Arts, Architecture, Landscape Design, Music in Harvard College (June, 1901), pp. 40-41.

Image Source: Detail from cover of the Harvard Class Album 1946.

Categories
Courses Exam Questions Harvard

Harvard. Economics Course Descriptions, Enrollments, and Exams. 1897-98

 

For the academic year 1897-1898 we are blessed with ample records for the economics courses offered (and bracketed) at Harvard. Detailed course descriptions, enrollment figures, semester-end exams are available and have been transcribed below for almost every course.

_______________________

ECONOMICS.
GENERAL STATEMENT.

Course 1 is introductory to the other courses. It is intended to give a general survey of the subject for those who take but one course in Economics, and also to prepare for the further study of the subject in advanced courses. It is usually taken with most profit by undergraduates in the second or third year of their college career. It may be taken with advantage in the second year by those who are attracted to political and social subjects. A knowledge of general history (such as is given in Course 1 in History) is a useful preparation.

The advanced courses divide themselves into two groups. The first group contains Courses 2, 3, 13, 14, 15, which are concerned chiefly with economic and social theory. Courses 2 and 15 follow the development of economic theory from its beginnings to the present time, with critical examination of the conclusions reached by economists of the past and the present. Course 13, on scope and method in economic investigation, continues the same subjects; it is taken to best advantage after either 2 or 15. Course 3 considers the wider aspects of economic and social study, and reviews the progress of sociological inquiry. Course 14 takes up the history and literature of socialistic and communistic proposals, and leads to a discussion of the foundations of existing institutions.

The second group contains the remaining courses, which are of a more descriptive and historical character. In all of them, however, attention is given to principles as well as to facts, and some acquaintance with the outlines of economic theory is called for.

Before taking any of the advanced courses, students are strongly advised to consult with the instructors. Courses 2, 3, 4, 7, 10, 12, 13, 14, 15, 16 may not be taken without the previous consent of the instructors. It is advised that Course 1 be taken in all cases as a preparation for the advanced courses; and such students only as have passed satisfactorily in Course 1 will be admitted to Courses 2, 3, 4, 12, 13, 14, 15, 16. But Courses 5, 7, and 9, may also be taken by Juniors and Seniors of good rank who are taking Course 1 at the same time; Course 6 is open to students who have taken or are taking cither History 13 or Economies 1; and Courses 10 and 11 are open to students who have passed satisfactorily either in History 1 or in Economics 1.

The Seminary in Economics is intended primarily for Graduate Students; but Seniors in Harvard College, who have had adequate training in the subject, may be admitted to it.

Source: Harvard University. Faculty of Arts and Sciences. Division of History and Political Science Comprising the Departments of History and Government and Economics, 1897-98,  pp. 30-31.

_______________________

Outlines of Economics
Economics 1

I. Outlines of Economics. —Principles of Political Economy.— Lectures on Social Questions and Monetary Legislation. Mon., Wed., Fri., at 9. Professor  [Frank William] Taussig, Asst. Professor Edward Cummings, Dr. John Cummings, assisted by Messrs. [Charles Sumner] Griffin, [Edward Henry] Warren, and ——.

Course 1 gives a general introduction to economic study, and a general view of Economics sufficient for those who have not further time to give to the subject. It begins with a consideration of the principles of production, distribution, exchange, money, and international trade, which is continued through the first half-year. In the second half-year, some of the applications of economic principles and some wider aspects of economic study are taken up. Social questions and the relations of labor and capital, the theory and practice of banking, and the recent currency legislation of the United States, will be successively treated in outline.

Course 1 will be conducted mainly by lectures. A course of reading will be laid down, and weekly written exercises will test the work of students in following systematically and continuously the lectures and the prescribed reading. Large parts of Mill’s Principles of Political Economy will be read, as well as parts of other general books; while detailed references will be given for the reading on the application and illustration of economic principles.

Source: Harvard University. Faculty of Arts and Sciences. Division of History and Political Science Comprising the Departments of History and Government and Economics, 1897-98,  p. 31.

Economics 1: Enrollment

[Economics] 1. Professor [Frank William] Taussig, Asst. Professor [Edward] Cummings, Dr. [John] Cummings, and Messrs. [Charles Sumner] Griffin, [Charles Whitney] Mixter and [Edward Henry] Warren. — Outlines of Economics. — Principles of Political Economy.— Social Questions, and Financial Legislation.  3 hours.

Total 381: 32 Seniors, 99 Juniors, 199 Sophomores, 14 Freshmen, 37 Others.

Source:   Harvard University, Annual Reports of the President and Treasurer of Harvard College, 1897-98, p. 77.

1897-98
Economics 1.
[Mid-year Examination]

  1. Is a shop building, situated on a busy city street, capital? is the land on which it stands capital? Is a dwelling on a fashionable city street capital? the land on which it stands?
  2. Does the rent of a piece of land determine its price, and if so, how? or does its price determine the rent, and if so, how?
  3. Do you believe that differences in wages in different occupations would cease if, by gratuitous education and support, access to each occupation were made equally easy for all?
  4. Mention a case in which the income received by a person doing no manual labor is to be regarded as wages; one in which it is to be regarded as profits; one in which it is to be regarded as interest; and one in which the classification would be regarded as doubtful.
  5. Explain what is meant by the effective desire of accumulation; and consider whether, in a country like England, the minimum return on capital fixed by it has been reached.
  6. “The quantity demanded [of any commodity] is not a fixed quantity, even at the same time and place; it varies according to the value; if the thing is cheap, there is usually a demand for more of it than when it is dear. The demand, therefore, depends partly on the value. But it was laid down before that value depends on the demand. From this contradiction, how shall we extricate ourselves? How solve the paradox, of two things, each depending on the other?”
    What answer did Mill give to the question thus put by him?
  7. Does the proposition that value is determined by cost of production hold true of gold?
  8. Is it advantageous to a country to substitute paper money completely for specie?
  9. Trace the consequences of an issue of inconvertible paper, greater in amount than the specie previously in circulation, on prices, on the foreign exchanges, and on the relations of debtor and creditor.

Source: Harvard University Archives. Harvard University, Mid-year examinations, 1852-1943. Box 4. Bound Volume: Examination Papers, Mid-Years 1897-98.

1897-98.
Economics 1.
[Final Examination]

Arrange your answers strictly in the order of the questions.

  1. In what sense does Mill use the terms “value” and “price”? Professor Hadley? What do you conceive to be meant by the “socialistic theory” of value?
  2. “Many people regard the luxury of the rich as being on the whole a means of preventing harm to the poor. They regard free expenditure of the capitalists’ money as a gain to laborers, and its saving as a loss.” Is this view sound?
  3. What services are rendered to society by commercial speculation? by industrial speculation?
  4. Patent-laws, protection by customs duties, private ownership of land, — wherein analogous, in Professor Hadley’s view? in your own view?
  5. Does Mill regard the rent of land as an “unearned increment”? Does Professor Hadley? On what grounds do they reach their conclusions?
  6. “By far the most important form of consumers’ coöperation is exemplified in government management of industrial enterprises.” Why, or why not, is government management to be regarded as a form of consumers’ coöperation? What other forms of such coöperation have had wide development?
  7. The peculiarities of labor considered as a commodity; and the grounds on which it is concluded that “the members of trade unions are in a condition entirely like that of the sellers of other commodities.”
  8. Consider how, according to Mill, successive issues of paper-money will affect the supply of specie in a country; and explain how far this theoretical conclusion was or was not verified by the mode in which the silver currency (dollars and certificates) issued under the act of 1878 affected the supply of gold in the United States.
  9. On what ground does Mill object to the issue of inconvertible paper? On what ground does Professor Dunbar object to the legal tender paper now issued by the United States? Wherein are the objections similar, wherein different?
  10. “If we try to make things for which we have only moderate advantages, and in so doing divert labor and capital from those where we have extraordinary ones, we do not, in general, make money; we lose more than we gain.” — HADLEY. Point out wherein this statement is akin to the analysis of international trade by Mill, and explain precisely what is here meant by making or losing money.

Source: Harvard University Archives. Examination Papers, 1873-1915. Box 5, Examination Papers 1898-99, Bound Volume, pp. 40-41.

_______________________

Mediaeval Economic History of Europe
Economics 10
[Omitted in 1897-98.]

[*10. The Mediaeval Economic History of EuropeTu., Th., (and at the pleasure of the instructor) Sat., at 12. Professor [William James] Ashley.]

The object of this course is to give a general view of the economic development of society during the Middle Ages. It will deal, among others, with the following topics: — the manorial system in its relation to mediaeval agriculture and serfdom ; the merchant gilds and the beginnings of town life and of trade ; the craft gild and the gild-system of industry, compared with earlier and later forms; the commercial supremacy of the Hanseatic and Italian merchants ; the trade routes of the Middle Ages and of the sixteenth century ; the merchant adventurers and the great trading companies ; the agrarian changes of the fifteenth nd sixteenth centuries and the break-up of the mediaeval organization of social classes ; the appearance of new manufactures and of the domestic industry.

Special attention will be devoted to England, but that country will be treated as illustrating the broader features of the economic evolution of the whole of western Europe; and attention will be called to the chief peculiarities of the economic history of France, Germany, and Italy.

Students will be introduced in this course to the use of the original sources, and they will need to be able to translate easy Latin.

It is desirable that they should already possess some general acquaintance with mediaeval history, and those who are deficient in this respect will be expected to read one or two supplementary books, to be suggested by the instructor. The course is conveniently taken after, before, or in conjunction with History 9; and it will be of especial use to those who intend to study the law of Real Property.

Source: Harvard University. Faculty of Arts and Sciences. Division of History and Political Science Comprising the Departments of History and Government and Economics, 1897-98, pp. 31-32.

_______________________

Modern Economic History of Europe and America
(from 1500)
Economics 11

11. The Modern Economic History of Europe and America (from 1500)Tu., Th., (and at the pleasure of the instructor) Sat., at 12. Professor [William James] Ashley.

This course, — which will usually alternate with Course 10 in successive years, — while intended to form a sequel to Course 10, will nevertheless be independent, and may usefully be taken by those who have not followed the history of the earlier period. The main thread of connection will be found in the history of trade; but the outlines of the history of agriculture and industry will also be set forth, and the forms of social organization dependent upon them. England, as the first home of the “great industry,” will demand a large share of attention; but the parallel or divergent economic history of the United States, and of the great countries of western Europe, will be considered side by side with it.

Source: Harvard University. Faculty of Arts and Sciences. Division of History and Political Science Comprising the Departments of History and Government and Economics, 1897-98, p. 32.

 Economics 11: Enrollment
1897-98

[Economics] 11. Professor Ashley— The Modern Economic History of Europe and America (from 1500). 2 or 3 hours.

Total 16: 9 Graduates, 5 Seniors, 1 Junior, 1 Sophomore.

Source:   Harvard University, Annual Reports of the President and Treasurer of Harvard College, 1897-98, p. 77.

1897-98.
Economics 11.
[Mid-year Examination]

N.B.—Not more than eight questions should be attempted.

  1. “Locutus sum de breviori via ad loca aromatum per maritimam navigationem quam sit ea quam facitis per Guineam.” Translate and comment upon this.
  2. Give some account of the Fairs of Champagne.
  3. What light does Jones’ account of agricultural conditions in Europe in his own time cast upon the agrarian history of England in the 15th and 16th centuries? Be as definite as possible in your answer.
  4. What do you suppose happened to the “craft-gilds” of England during the reign of Edward VI?
  5. Discuss the purpose and effect of the statute 5 Eliz. c. 4, in the matter of the Assessment of Wages.
  6. What were the essential characteristics of the “Domestic System” of Industry?
  7. Give some account of the industrial legislation of France in the 16th century.
  8. “The policy of Europe occasions a very important inequality in the whole of the advantages and disadvantages of the different employments of labour and stock.” What had the writer of this passage in mind?
  9. Give some account of any four of the following: Albuquerque, Veramuyden, Jacob Fugger, John Hales, Jacques Cartier, Bartholomew Diaz, Barthelemy Laffemas.
  10. Give a critical account of any really important work, not prescribed, of which you have read any considerable portion in connection with this course.

Source: Harvard University Archives. Harvard University, Mid-year examinations, 1852-1943. Box 4. Bound Volume: Examination Papers, Mid-Years 1897-98.

1897-98.
Economics 11.
[Year-end Examination]

N.B.- Not more than eight questions should be attempted.

  1. “Publicae mendicationis licentiam posse civium legibus cohiberi ad liquidum ostendit ille absolutus Theologus, loannes Major.” Translate; and shew the significance of the position thus maintained.
  2. Illustrate from the history of Hamburg the change in the position of the Hanseatic League during the 16th century.
  3. Distinguish between the various races of immigrants into England since 1500, and state shortly the several respects in which the trade and industry of England were influenced by each.
  4. “Perhaps the wisest of all the commercial regulations of England.” Give a brief account of the enactment of which Adam Smith thus speaks; distinguish between the various aspects in which it may be regarded; and give your own opinion as to the justice of Adam Smith’s observation.
  5. Explain the position of “les Six Corps” at Paris. Does London furnish any analogous institutions?
  6. “Hitherto,” i.e. up to 1750, “industry had been chiefly carried on in England by numbers of smaller capitalists who were also manual workmen.” Criticise this as a bit of exposition.
  7. The position of Arthur Young in economic history.
  8. The commercial policy of the younger Pitt.
  9. Mention, with the briefest possible comment, some of the more important features in which the agricultural, industrial and commercial life of the England of to-day differs from that of the England of 1750.
  10. What were the principal defects in the administration of the English Poor Laws prior to 1834, and how was it sought to remedy them?
  11. Explain the need for the English Factory Acts, and give some account of their history.
  12. Give a critical estimate of any really important book, not prescribed, of which you have read any considerable proportion in connection with this course during the second half-year.

Source: Harvard University Archives. Examination Papers, 1873-1915. Box 5, Examination Papers 1898-99, Bound Volume, pp. 50-51.

_______________________

The Economic History of the United States
Economics 6

6. The Economic History of the United StatesTu., Th., at 2.30, and a third hour at the pleasure of the instructors. Mr. [Guy Stevens] Callender.

Course 6 gives a general survey of the economic history of the United States from the formation of the Union to the present time, and considers also the mode in which economic principles are illustrated by the experience so surveyed. A review is made of the financial history of the United States, including Hamilton’s financial system, the second Bank of the United States and the banking systems of the period preceding the Civil War, coinage history, the finances of the Civil War, and the banking and currency history of the period since the Civil War. The history of manufacturing industries is taken up in connection with the course of international trade and of tariff legislation, the successive tariffs being followed and their economic effects considered. The land policy of the United States is examined partly in its relation to the growth of population and the inflow of immigrants, and partly in its relation to the history of transportation, including the movement for internal improvements, the beginnings of the railway system, the land grants and subsidies, and the successive bursts of activity in railway building. Comparison will be made from time to time with the contemporary economic history of European countries.

Written work will be required of all students, and a course of reading will be prescribed, and tested by examination. The course is taken advantageously with or after History 13. While an acquaintance with economic principles is not indispensable, students are strongly advised to take the course after having taken Economics 1, or, if this be not easy to arrange, at the same time with that course.

Source: Harvard University. Faculty of Arts and Sciences. Division of History and Political Science Comprising the Departments of History and Government and Economics, 1897-98, pp. 32-33.

Economics 6: Enrollment

[Economics] 6. Dr. Callender. — The Economic History of the United States. 3 hours.

Total 94: 4 Graduates, 38 Seniors, 41 Juniors, 8 Sophomores, 1 Sophomore, 2 Others.

Source:   Harvard University, Annual Reports of the President and Treasurer of Harvard College, 1897-98, p. 78.

1897-98.
Economics 6.
[Mid-year Examination]

[Omit one question from each group]

I.

  1. “The effect of England’s policy was, through a restriction of the market, to render the production of those staple commodities (i.e. of agriculture, and the fisheries) less profitable. Thus New England, and later the middle colonies, not being allowed to exchange their normal products for England’s manufactures, were forced to begin manufacturing for themselves.”—
    “Briefly describe the measures designed to prevent the rise of manufactures in the colonies; and state whether in your opinion the growth of manufactures in the northern colonies was stopped chiefly by this legislation or by other causes.
  2. The American colonies during the Revolution were in much the same economic position as the South during the Rebellion. The chief resource of the latter was the value of its cotton crop to the world; that of the former was the supposed value of their trade to the nations of Europe. Describe the various ways in which the Revolutionary statesmen made use of this resource.
  3. Judging from the opinions of statesmen as well as from acts of legislation what would you say were the leading objects of American Commercial policy from 1783 to 1789?
  4. State briefly the exact circumstances which permitted the growth of American commerce during the years from 1793 to 1806. How did this temporary commercial prosperity affect the subsequent growth of manufactures?

II.

  1. Compare the conditions which gave rise to manufactures in the northern colonies before 1760 with those which prevailed during the years immediately following 1783 and 1815; and indicate what conclusions you would draw from such a comparison, as to the necessity or expediency of protective legislation to secure the development of manufactures in a new country.
  2. Discuss the effect of the duties on Cotton and Iron during the period from 1816 to 1833.
  3. Compare the treatment of wages in Hamilton’s Report on Manufactures with that which appeared in the debate on the Tariff Act of 1846. How do you explain the change?
  4. Mention several industries which were created or greatly promoted by inventions between 1840 and 1860.

III.

  1. Henry Clay declared in 1832 that the seven years preceding 1824 “exhibited a scene of the most widespread dismay and desolation,” while the seven years following 1824 exhibited the “greatest prosperity which this people, bare enjoyed since the establishment of the present constitution.” How do you explain this change?
  2. In what ways have the people of the United States made use of the Federal and State governments to provide transportation facilities? How do you explain this tendency to State interference in industrial affairs at so early a date in America?
  3. Describe the abuses in Railroad management which the Interstate Commerce Act was intended to correct.
  4. Explain why competition proves less effective in regulating freight rates than in regulating the price of most commodities.

Source: Harvard University Archives. Harvard University, Mid-year examinations, 1852-1943. Box 4. Bound Volume: Examination Papers, Mid-Years 1897-98.

1897-98.
Economics 6.
[Year-end Examination]

Answer at least eight questions.

  1. Give your reasons for agreeing to, or dissenting from, the following proposition: Until the wars of the French Revolution temporarily suspended the colonial policy of Continental Europe, the United States was in a more unfavorable economic position than they had been in prior to the Revolution
  2. Why was the adoption of a liberal tariff policy by the U.S. in 1846 more justifiable than in 1816?
  3. “The provisions of the constitution were universally considered as affording a complete security against the danger of paper money. The introduction of the banking system met with a strenuous opposition on various grounds; but it was not apprehended that bank notes, convertible at will into specie, and which no person could be legally compelled to take in payment, would degenerate into pure paper money, no longer paid at sight in specie… It was the catastrophe of 1814 which first disclosed not only the insecurity of the American banking system, as it then existed, but also that when a paper currency, driving away, and suspending the use of gold and silver, has insinuated itself through every channel of circulation, and become the only medium of exchange, every individual finds himself, in fact compelled to receive such currency, even when depreciated more than twenty per cent. in the same manner as if it had been a legal tender.” — GALLATIN.
    Prior to the adoption of the national banking system in 1863, how did the federal government attempt to prevent the evil here described. and with what success?
  4. How far do the conditions, which render competition ineffective as a regulator of transportation charges, prevail in any of the industries in which Trusts have been formed? — or to ask the same thing in another way, how far is it possible to justify Trusts on the same grounds as Railroad Pools?
  5. What reasons would you assign for the change in the relative value of gold and silver which occurred after 1873?
  6. What difficulty did the Treasury department encounter in administering the silver act of 1878, and what means were used to overcome it?
  7. Compare the effect of the protective duties on wool and woollens since 1867 with the effect of those on silk and steel during the same time.
  8. “In the division of employments which has taken place in America, the far preferable share, truly, has fallen to the Northern States…The states, therefore, which forbid slavery, having reaped the economical benefits of slavery, without incurring the chief of its moral evils, seem to be even more indebted to it than the slave states.” — WAKEFIELD.
    How would you explain this statement?
  9. According to Mr. Cairnes, what constituted the economic basis of Negro slavery in the Southern States and enabled it to successfully resist the competition of free white labor? Do you consider this economic basis of slavery to have been permanent?
  10. Describe the most important change in Southern agrarian conditions which has resulted from emancipation.
  11. What influences can you mention that have contributed to the fall in the prices of the staple products of Northern agriculture during the last ten years?
  12. Why has this fall in the price of agricultural products caused greater hardship to the farmers than the corresponding fall in the price of manufactured products has caused among manufacturers?

Source: Harvard University Archives. Examination Papers, 1873-1915. Box 5, Examination Papers 1898-99, Bound Volume, pp. 45-47.

_______________________

History and Literature of Economics to the Close of the 18th Century
Economics 15

*15. The History and Literature of Economics to the Close of the Eighteenth CenturyMon., Wed., (and at the pleasure of the instructor) Fri., at 12.     Professor[William James] Ashley.

The course of economic speculation will here be followed, in its relation alike to the general movement of contemporary thought and to contemporary social conditions. The lectures will consider the economic theories of Plato and Aristotle; the economic ideas underlying Roman law; the mediaeval church and the canonist doctrine; mercantilism in its diverse forms; “political arithmetic;” the origin of the belief in natural rights and its influence on economic thought; the physiocratic doctrine; the work and influence of Adam Smith; the doctrine of population as presented by Malthus; Say and the Erench school; and the beginnings of academic instruction in economics.

The lectures will be interrupted from time to time for the examination of selected portions of particular authors; and careful study will be given to portions of Plato’s Republic and Aristotle’s Politics (in translation) to Mun’s England’s Treasure, Locke’s Considerations of the Consequences of the Lowering of Interest, certain Essays of Hume, Turgot’s Réflexions, and specified chapters of Adam Smith’s Wealth of Nations, and Malthus’ Essay. Students taking the course are expected to procure the texts of the chief authors considered, and to consult the following critical works:

Ingram, History of Political Economy; Cossa, Introduction to the Study of Political Economy; Cannan, History of the Theories of Production and Distribution; Bonar, Philosophy and Political Economy; Böhm-Bawerk, Capital and Interest; Taussig, Wages and Capital.

Source: Harvard University. Faculty of Arts and Sciences. Division of History and Political Science Comprising the Departments of History and Government and Economics, 1897-98, pp. 33-34.

Economics 15: Enrollment
1897-98

[Economics] 15. Professor Ashley. — The History and Literature of Economics to the Middle of the Nineteenth Century. 2 or 3 hours.

Total 6: 3 Graduates, 1 Senior, 2 Sophomores.

Source:   Harvard University, Annual Reports of the President and Treasurer of Harvard College, 1897-98, p. 77.

1897-98.
Economics 15.
[Mid-year Examination]

N.B.—Not more than eight questions should be attempted.

  1. Compare the Republic of Plato with the Politics of Aristotle, as to purpose and temper.
  2. Expound Aristotle’s teaching with regard to Slavery.
  3. “He is supposed to have given a striking proof of his wisdom, but his device for getting money is of universal application.” Comment, and explain the context.
  4. What parts of Aristotle’s criticism of Communism seem to you pertinent to modern Socialism. Explain what particular kind of Socialism you have in mind.
  5. Set forth, and criticise, Maine’s account of the influence in modern times of the conception of a Law of Nature.
  6. Were the early Christians communists?
  7. How did “Inter-est [sic],” in its original meaning, differ from “Usury.”
  8. The position in economic literature of Nicholas Oresme.
  9. What principles, if any, of the canonist teaching seem to you to have any bearing on modern economic problems.
  10. What were “the particular ways and means to encrease our exportations and diminish our importations,” according to Mun?

Source: Harvard University Archives. Harvard University, Mid-year examinations, 1852-1943. Box 4. Bound Volume: Examination Papers, Mid-Years 1897-98.

1897-98.
Economics 15.
[Year-end Examination]

  1. Criticise the current conception of “Mercantilism” in the light of your own study of the later English mercantilist writers.
  2. The place of Locke in English economic thought.
  3. “Tout ce qu’il y a de vrai dans ce volume estimable, mais pénible à lire, en deux gros volumes in-4°, se trouve dans les Réflexions de Turgot; tout ce qu’Adam Smith y a ajouté manque d’exactitude et même de fondement.”
    Translate, and then criticise this remark of Du Pont’s.
  4. Trace the various elements which went to make up the idea of Nature in Adam Smith’s mind, and then explain Smith’s application of it to any particular subject.

Source: Harvard University Archives. Examination Papers, 1873-1915. Box 5, Examination Papers 1898-99, Bound Volume, p. 53.

_______________________

Economic Theory
in the 19th Century
Economics 2

*2. Economic Theory in the Nineteenth CenturyMon., Wed., Fri., at 2.30. Professor [Frank William] Taussig.

Course 2 is designed to acquaint the student with the history of economic thought during the nineteenth century, and to give him at the same time training in the critical consideration of economic principles. The exercises are accordingly conducted mainly by the discussion of selected passages from the important writers; and in this discussion students are expected to take an active part. Lectures are given at intervals, tracing the general movement of economic thought and describing its literature. Special attention will be given to the theory of distribution.

The course opens with an examination of Ricardo’s doctrines, selections from Ricardo’s writings being read and discussed. These will then be compared with the appropriate chapters in Mill’s Principles of Political Economy, and further with passages in Cairnes’ Leading Principles. The theory of wages, and the related theory of business profits, will then be followed in the writings of F. A. Walker, Sidgwick, and Marshall, and a general survey made of the present stage of economic theory in England and the United States. The development on the continent of Europe will be traced chiefly in lectures; but toward the close of the year a critical examination will be made of the doctrines of the modern Austrian school.

Course 2 is taken with advantage in the next year after Course 1; but Course 15 may also be taken with advantage after Course 1, and then followed by Course 2, or taken contemporaneously with it.

Source:  Harvard University. Faculty of Arts and Sciences. Division of History and Political Science Comprising the Departments of History and Government and Economics, 1897-98, p. 34.

 Economics 2: Enrollment
1897-98

[Economics] 2. Professor Taussig. — Economic Theory in the Nineteenth Century. 3 hours.

Total 32: 9 Graduates, 9 Seniors, 11 Juniors, 3 Sophomores.

Source:   Harvard University, Annual Reports of the President and Treasurer of Harvard College, 1897-98, p. 77.

1897-98.
Economics 2.
[Mid-year. 1898.]

[Arrange your answers in the order of the questions. One question may be omitted.]

  1. According to Ricardo, what is the effect, if any, of a rise in the price of food on wages? on profits? on the prices of commodities?
  2. “Ricardo expresses himself as if the quantity of labor which it costs to produce a commodity and bring it to the market, were the only thing on which its value depended. But since the cost of production to the capitalist is not labor but wages, and since wages may be greater or less, the quantity of labor being the same; it would seem that the value of the product cannot be determined solely by the quantity of labor, but by the quantity together with the remuneration; and that values must depend on wages.” — Mill.
    What do you conceive Ricardo would have said to this?
  3. “We have therefore remarked that the difficulty of passing from one class of employments to a class greatly superior, has hitherto caused the wages of all those classes of laborers who are separated from one another by any very marked barrier, to depend more than might be supposed upon the increase of population of each class, considered separately; and that the inequalities in the remuneration of labor are much greater than could exist if the competition of the laboring people generally could be brought practically to bear on each particular employment. It follows from this that wages in each particular employment do not rise or fall simultaneously, but are, for short and sometimes even for long periods, nearly independent of each other. All such disparities evidently alter the relative costs of production of different commodities, and will therefore be completely represented in the natural or average value.” — Mill.
    What has Cairnes added to this?
  4. “He [Mr. Longe] puts the case of a capitalist who, by taking advantage of the necessities of his workmen, effects a reduction in their wages; and asks how is this sum, thus withdrawn, to be restored to the fund? . . . The answer to the case put by Mr. Longe is easy on his own principles; and I am disposed to flatter myself that the reader who has gone with me in the foregoing discussion will not have much difficulty in replying to it on mine.” — Cairnes.
    Give the reply.
  5. “Fixity or definiteness is the very essence of the supposed wages-fund. No one denies that some amount or other must within a given period be disbursed in the form of wages. The only question is whether that amount be determinate or indeterminate.” — Thornton.
    What is Cairnes’s answer to the question put in this passage?
  6. What would you expect the relation of imports to exports to be in a country whose inhabitants had for a long time been borrowing, and were still borrowing, from the inhabitants of other countries?
  7. Are general high wages an obstacle to a country’s exporting?
  8. “Granted a certain store of provisions, of tools, and of materials for production, sufficient, say, for 1000 laborers, those who hold the wage-fund theory assert that the same rate of wages (meaning thereby the actual amount of necessaries, comforts, and luxuries received by the laborer) would prevail whether these laborers be Englishmen or East Indians. . . . On the contrary, it is not true that the present economical quality of the laborers, as a whole, is an element in ascertaining the aggregate amount that can now be paid in wages; that as wages are paid out of the product, and as the product will be greater or smaller by reason of the workman’s sobriety, industry, and intelligence, or his want of these qualities, so wages may and should be higher or lower accordingly?”
    Give your opinion.
  9. What do you conceive to be the “no profits class of employers” in President Walker’s theory of distribution?

Source: Harvard University Archives. Harvard University, Mid-year examinations, 1852-1943. Box 4. Bound Volume: Examination Papers, Mid-Years 1897-98.

1897-98.
Economics 2.
[Final Examination]

The answer to one question may be omitted.

  1. The analysis of capital in its relation to labor and wages at the hands of Ricardo and of Böhm-Bawerk, — wherein the same? wherein different?
  2. The contributions of permanent worth for economic theory by Cairnes? by F.A. Walker? [Consider one.]
  3. The position of Carey and Bastiat in the development of economic theory.
  4. “If the efficiency of labor could be suddenly doubled, whilst the capital of the country remained stationary, there would be a great and immediate rise in real wages. The supplies of capital already in existence would be distributed among the laborers more rapidly than would otherwise be the case, and the increased efficiency of labor would soon make good the diminished supplies. The fact is that an increase in the efficiency of labor would bring about an increase in the supply of capital.” — Marshall. Why? or why not?
  5. “The capital of the employer is by no means the real source of the wages even of the workmen employed by him. It is only the intermediate reservoir from which wages are paid out, until the purchasers of the commodities produced by that labor make good the advance and thereby encourage the undertaker to purchase additional labor.” W. Roscher.
    What do you say to this?
  6. “If the rate of profit falls, the laborer gets more nearly the whole amount of the product. But if the rate of wages falls, we have a corresponding fall in prices and little change in the relative shares of labor and capital.” Hadley.
    Why, or why not, in either case?
  7. “In the present condition of industry, most sales are made by men who are producers or merchants by profession, and who hold an amount of commodities entirely beyond any needs of their own. Consequently, for them the subjective use-value of their own wares is, for the most part, very nearly nil; and the figure which they put on their own valuation almost sinks to zero.” Explain the bearing of this remark on the theory of value as developed by Böhm-Bawerk.
  8. What, according to Böhm-Bawerk, is the explanation of interest derived from “durable consumption goods”? And what is your own view?
  9. How far do you conceive that there is a “productivity” of capital, serving to explain the existence of interest, and the rate?

Source: Harvard University Archives. Examination Papers, 1873-1915. Box 5, Examination Papers 1898-99, Bound Volume, pp. 41-42.

_______________________

Scope and Method in Economic Theory and Investigation
Economics 132

*132 hf. Scope and Method in Economic Theory and Investigation. Half-course (second half-year). Mon., Wed., Fri., at 11. Professor [William James] Ashley.

Course 13 will examine the methods by which the important writers, from Adam Smith to the present time, have approached economic questions, and the range which they have given their inquiries; and will consider the advantage of different methods, and the expediency of a wider or narrower scope of investigation. Mill’s essay on the Definition of Political Economy; Cairnes’ Logical Method of Political Economy; Keynes’ Scope and Method of Political Economy; certain sections of Wagner’s Grundlegung and Schmoller’s essay on Volkswirthschaft will be carefully examined. The conscious consideration of method by the later writers of the classic school and by their successors in England; the rise of the historical school and its influence; the mode in which contemporary writers approach the subject, — will he successively followed.

Course 13 is open to students who take or have taken Course 2 or Course 15. A fair reading knowledge of German as well as of French will be expected of students, and the opportunity will be taken to assist them to acquire facility in reading scientific German. Subjects will be assigned for investigation and report, and the results of such investigations will be presented for discussion.

Source: Harvard University. Faculty of Arts and Sciences. Division of History and Political Science Comprising the Departments of History and Government and Economics, 1897-98, pp. 34-35.

Economics 132: Enrollment
1897-98

[Economics] 132. Professor Ashley. — Scope and Method in Economic Theory and Investigation. 3 hours.

Total 5: 3 Graduates, 1Senior, 1 Sophomore.

Source:   Harvard University, Annual Reports of the President and Treasurer of Harvard College, 1897-98, p. 77.

1897-98.
Economics 132.
[Year-end Examination]

  1. “Ganz unabhängig von der deutschen historischen National-Ökonomie haben Sociologen wie A. Comte ähnliche, freilich auch zu weit gehende Bedenken gegen Deduction und Abstraction der britischen Oekonomik erhoben.”
    Translate this; and then (1) state Comte’s position with regard to economic method, (2) criticise it.
  2. “Die besondere Leistung des wissenschaftlichen Socialismus ist der Nachweis des beherrschenden Einflusses der Privateigenthums ordnung, speciell des Privateigenthums‚ an den sachlichen Productionsmitteln’, auf die Gestaltung der Production und der Vertheilung des Productionsertrag, zumal bei Wegfall aller Beschränkungen der Verfügungsbefugnisse des Privateigenthümers im System der freien Concurrenz…Durch den Socialismus ist aber auch das andere grosse Hauptproblem, dasjenige der Freiheit und ihrer Rechtsordnung, in ein neues Stadium getreten. Hier begeht der Socialismus nun jedoch trotz seiner scharfen Kritik der wirthschaftlichen Freiheit im System der ökonomischen Individualismus und Liberalismus principiell denselben Fehler, wie letzterer: auch er fasst die Freiheit als Axiom, statt als Problem auf, ein schwerstes Problem gerade jeder socialistischen Rechts- und Wirtschaftsordnung.”
    (1) Translate, (2) explain, and (3) comment on this.
  3. Discuss the questions raised by the application to Economies of the distinction between a Science and an Art.
  4. What did J. S. Mill mean by the Historical Method? Consider (1) the source of the idea, (2) its characterization by Mill, and (3) the bearing of his utterances with regard to it upon the question of economic method.
  5. Examine either (1) the Malthusian doctrine of Population or (2) the Ricardian doctrine of Rent as a specimen of an economic “law.”

Source: Harvard University Archives. Examination Papers, 1873-1915. Box 5, Examination Papers 1898-99, Bound Volume, pp. 52-53.

_______________________

Principles of Sociology
Economics 3

*3. The Principles of Sociology. Mon., Wed., and (at the pleasure of the instructor) Fri., at 1.30. Asst. Professor Edward Cummings.

Course 3 begins with a general survey of the structure and development of society; showing the changing elements of which a progressive society is composed, the forces which manifest themselves at different stages in the transition from primitive conditions to complex phases of civilized life, and the structural outlines upon which successive phases of social, political, and industrial organization proceed. Following this, is an examination of the historical aspects which this evolution has actually assumed: Primitive man, elementary forms of association, the various forms of family organization, and the contributions which family, clan and tribe have made to the constitution of more comprehensive ethnical and political groups ; the functions of the State, the circumstances which determine types of political association, the corresponding expansion of social consciousness, and the relative importance of military, economic, and ethical ideas at successive stages of civilization. Special attention is given to the attempts to formulate physical and psychological laws of social growth; to the relative importance of natural and of artificial selection in social development; the law of social survival; the dangers which threaten civilization; and the bearing of such general consideration upon the practical problems of vice, crime, poverty, pauperism, and upon mooted methods of social reform.

The student is thus acquainted with the main schools of sociological thought, and opportunity is given for a critical comparison of earlier phases of sociological theory with more recent contributions in Europe and the United States. Regular and systematic reading is essential. Topics are assigned for special investigation in connection with practical or theoretical aspects of the course.

Source: Harvard University. Faculty of Arts and Sciences. Division of History and Political Science Comprising the Departments of History and Government and Economics, 1897-98, p. 35.

 Economics 3: Enrollment
1897-98

[Economics] 3 Asst. Professor E. Cummings. — The Principles of Sociology. — Development of the Modern State, and of its Social Functions. 2 or 3 hours.

Total 59: 4 Graduates, 30 Seniors, 13 Juniors, 6 Sophomores, 6 Others.

Source:   Harvard University, Annual Reports of the President and Treasurer of Harvard College, 1897-98, p. 77.

1897-98.
Economics 3.
[Mid-year examination]

[Answer the questions in the order in which they stand. Give one hour to each group.]

I.

Discuss the merits and defects of the following conceptions of society:

A) Society as an organism.
B) Society as a physio-psychic organism.
C) Society as an organization.
D) Society as an “organisme contractuel.”

What in your opinion are the essential differences between an ant hill and a human society?

II.

Give a critical summary and comparison of the views of Spencer, Giddings, Ritchie in regard to (a) the origin, (b) the development and forms, and (c) the functions of political organization.
Contrast the ancient, medieval and modern views of the relations of the State to Society and to the Individual.

III.

Discuss the views of Spencer, Westermarck, Giddings and others on the causes and the effects of the successive phases of family organization.
What claims has the family to be regarded as the “social unit”?
Discuss the significance of existing tendencies.

Source: Harvard University Archives. Harvard University, Mid-year examinations, 1852-1943. Box 4. Bound Volume: Examination Papers, Mid-Years 1897-98.

1897-98.
Economics 3.
[Final Examination]

I.

The nature, the causes, and the criteria of progress, according to (a) Spencer, (b) Kidd, (c) La Pouge, (d) Haycraft, (e) Giddings, (f) Tarde? State and illustrate by historical examples your own views in regard to the “curve of progress.”

II.

“The special feature of the final adjustment secured by our occidental civilizations, contrary to what has been seen on the earth before them, will therefore have been the subordination of the social to the individual. This singularly daring enterprise is the true novelty of modern times. It is well worth living to second it or to participate in it.”— TARDE.
“There seems no avoiding the conclusion that these conspiring causes must presently bring about that lapse of self-ownership into ownership by the community, which is partially implied by collectivism and completely by communism.” — SPENCER.
Discuss carefully the merits of these opinions, and the evidence on which they rest.

III.

What do you conceive to be some of the dangerous tendencies of our civilization? And what are the remedies?

IV.

State the subject of your final report and the reading you have done in connection with it.

Source: Harvard University Archives. Examination Papers, 1873-1915. Box 5, Examination Papers 1898-99, Bound Volume, p. 42.

_______________________

Socialism and Communism
Economics 14

*14. Socialism and Communism. — History and Literature. Tu., Th., and at the pleasure of the instructor) Sat., at 9. Asst. Professor Edward Cummings.

Course 14 is primarily an historical and critical study of socialism and communism. It traces the history and significance of schemes for social reconstruction from the earliest times to the present day. It discusses the historical evidences of primitive communism, the forms assumed by private ownership at different stages of civilization, the bearing of these considerations upon the claims of modern socialism, and the outcome of experimental communities in which socialism and communism have actually been tried. Special attention, however, is devoted to the recent history of socialism, — the precursors and the followers of Marx and Lassalle, the economic and political programmes of socialistic parties in Germany, France, and other countries.

The primary object is in every case to trace the relation of historical evolution to these programmes; to discover how far they have modified history or found expression in the policy of parties or statesmen; how far they must be regarded simply as protests against existing phases of social evolution; and how far they may be said to embody a sane philosophy of social and political organization.

The criticism and analysis of these schemes gives opportunity for discussing from different points of view the ethical and historical value of social and political institutions, the relation of the State to the individual, the political and economic bearing of current socialistic theories.

The work is especially adapted to students who have had some introductory training in Ethics as well as in Economics. A systematic course of reading covers the authors discussed; and special topics for investigation may be assigned in connection with this reading.

Source: Harvard University. Faculty of Arts and Sciences. Division of History and Political Science Comprising the Departments of History and Government and Economics, 1897-98, pp. 35-36.

Economics 14: Enrollment

[Economics] 14. Asst. Professor E. Cummings. — Communism and Socialism. — History and Literature. 2 or 3 hours.

Total 12: 3 Graduates, 5 Seniors, 2 Juniors, 2 Sophomores.

Source: Harvard University. Report of the President of Harvard College, 1897-98, p. 78.

1897-98.
Economics 14.
[Mid-year Examination]

Outline briefly the characteristics of socialistic theory and practice in ancient, medieval and modern times, — devoting about an hour to each epoch, and showing —

(a) so far as possible the continuity of such speculations; the characteristic resemblances and differences;
(b) the influence of peculiar historical conditions;
(c) the corresponding changes in economic theory and practice.

Source: Harvard University Archives. Harvard University, Mid-year examinations, 1852-1943. Box 4. Bound Volume: Examination Papers, Mid-Years 1897-98.

No Year-end Examination for 1898 found.

_______________________

Labor Question
in Europe and the U.S.
Economics 9.

9. The Labor Question in Europe and the United States. — The Social and Economic Condition of Workingmen. Tu., Th., Sat., at 10. Asst. Professor Edward Cummings and Dr. John Cummings.

Course 9 is a comparative study of the condition and environments of workingmen in the United States and European countries. It is chiefly concerned with problems growing out of the relations of labor and capital. There is careful study of the voluntarily organizations of labor, — trade unions, friendly societies, and the various forms of cooperation; of profit-sharing, sliding scales, and joint standing committees for the settlement of disputes; of factory legislation, employers’ liability, the legal status of laborers and labor organizations, state courts of arbitration, and compulsory government insurance against the exigencies of sickness, accident, and old age. All these expedients, together with the phenomena of international migration, the questions of a shorter working day and convict labor, are discussed in the light of experience and of economic theory, with a view to determining the merits, defects, and possibilities of existing movements.

The descriptive and theoretical aspects of the course are supplemented by statistical evidence in regard to wages, prices, standards of living, and the social condition of labor in different countries.

Topics will be assigned for special investigation, and students will be expected to participate in the discussion of selections from authors recommended for a systematic course of reading.

The course is open not only for students who have taken Course 1, but to Juniors and Seniors of good rank who are taking Course 1.

Source: Harvard University. Faculty of Arts and Sciences. Division of History and Political Science Comprising the Departments of History and Government and Economics, 1897-98, pp. 36-37.

Economics 9: Enrollment

[Economics] 9. Asst. Professor E. Cummings and Dr. J. Cummings. — The Labor Question in Europe and the United States. — The Social and Economic Condition of Workingmen. 3 hours.

Total 108: 1 Graduate, 39 Seniors, 51 Juniors, 12 Sophomores, 5 Others.

Source: Harvard University. Report of the President of Harvard College, 1897-98, p. 78.

No Mid-year Examination found.

1897-98.
Economics 9.
[Year-End Examination]

I.
WORKINGMEN’S INSURANCE.

“After a preliminary examination of the various kinds of working-men’s insurance, and the chief methods by which its provision can be accomplished, we have considered the history and present condition of the problem in each of the great countries of Europe and in the United States. It now remains to pass in review the whole field, to contrast, in a measure, the various policies that have been pursued, and to indicate some of the ways in which this rich experience can be of assistance in any attempt that may be made in this country to further similar movements.”
Devote one hour (a) to analyzing the present condition in each country; (b) to indicating the ways in which this rich experience can be of assistance.

II.

a) Give the name, the size, the characteristics of the important labor organizations in the United States.

b) Compare the development and present condition of labor organizations in the United States, with the movement in England.

c) How do you account for the differences in success attending trade union and coöperative enterprises in the two countries?

III.

a) What agencies, public and private, are available for settling disputes between employers and employed in the United States?

b) To what important legal questions have these disputes given rise? What has been the attitude of the judiciary and what are the merits of the present controversy in regard to injunctions?

c) What has been the general character and value of labor legislation during the last decade?

IV.

Indicate approximately the husband’s earnings, the family income and the standard of living among laborers in coal, iron, steel, textile or other industries,

1) in the United States.
2) in European countries.
3) Compare the native with the foreign-born American in these respects.
4) What conclusions do you draw from the evidence?

V.

What is the subject of your special report? State briefly (a) the method of your research, (b) the conclusions reached.

Source: Harvard University Archives. Examination Papers, 1873-1915. Box 5, Examination Papers 1898-99, Bound Volume, pp. 49-50.

_______________________

Statistics
Economics 4

*4. Statistics. — Applications to Social and Economic Problems. — Studies in Movements of Population. — Theory and Method. Mon., Wed., Fri., at 11. Dr. John Cummings.

This course deals with statistical methods used in the observation and analysis of social conditions, with the purpose of showing the relation of statistical studies to Economics and Sociology, and the scope of statistical inductions. It undertakes an examination of the views entertained by various writers regarding the theory and use of statistics, and an historical and descriptive examination of the practical methods of carrying out statistical investigations. The application of statistical methods is illustrated by studies in political, fiscal, and vital statistics, in the increase and migration of population, the growth of cities, the care of criminals and paupers, the accumulation of capital, and the production and distribution of wealth.

Source: Harvard University. Faculty of Arts and Sciences. Division of History and Political Science Comprising the Departments of History and Government and Economics, 1897-98, p. 37.

Economics 4: Enrollment
1897-98

[Economics] 4. Dr. J. Cummings. — Statistics. — Applications to Economic and Social Questions. — Studies in the Movement of Population. — Theory and Method. 3 hours.

Total 18: 7 Seniors, 7 Juniors, 3 Sophomores, 1 Other.

Source: Harvard University. Report of the President of Harvard College, 1897-1898, p. 78.

1897-98.
Economics 4.
[Mid-year Examination]

[Divide your time equally between A. and B.]

A.

[Take two.]

  1. In what sense do you understand Quetelet’s assertion that “the budget of crime is an annual taxation paid with more preciseness than any other”?
    Comment upon the “element of fixity in criminal sociology.”
    What are the “three factors of crime”?
    Can you account for the “steadiness of the graver forms of crime”? for the increase or decrease of other crimes?
    Define “penal substitutes.”
    What determines the rate of criminality?
    Comment upon the tables relating to crime in the last federal census, and explain how far they enable one to estimate the amount of crime committed and the increase or decrease in that amount.
  2. Comment upon the movement of population in the U. S. as indicated in the census rates of mortality and immigration. Upon the movement of population in France and in other European countries during this century. Can you account for the decline in the rates of mortality which characterize these populations?
    Give an account of the growth of some of the large European cities and of the migratory movements of their populations.
    Give an account for the depopulation of rural districts which has taken place during this century?
  3. Give some account of the Descriptive School of Statisticians and of the School of Political Arithmetic.
    Of the organization and work of statistical bureaus in European countries during this century.
    Of the census bureau in the United States.

B.

[Take four.]

  1. What are some of the “positive” statistical evidences of vitality in a population? “negative”?
  2. Define “index of mortality.”
  3. Comment upon the density and distribution of population in the United States.
  4. What do you understand by “normal distribution of a population according to sex and age”? Define “movement of population.”
  5. Explain the various methods of estimating a population during intercensal years.

Source: Harvard University Archives. Harvard University, Mid-year examinations, 1852-1943. Box 4. Bound Volume: Examination Papers, Mid-Years 1897-98.

1897-98.
Economics 4.
[Final Examination]

A.

I.

“The wealth of a nation is a matter of estimate only. Certain of its elements are susceptible of being approximated more closely than others; but few of them can be given with greater certainty or accuracy than is expressed in the word ‘estimated.’” Why? State the several methods used for determining the wealth of a nation. Give some account of the increase and of the present distribution of wealth in the United States.

II.

What statistical data indicate the movement of real wages during this century? What facts have to be taken into account in determining statistically the condition of wage earners? State the several methods of calculating index numbers of wages and prices, and explain the merits of each method. Explain the use of weighted averages as indexes, and the considerations determining the weights. What has been the movement of wages and prices in the United States since 1860?

III.

Statistical data establishing a hierarchy of European races, the fundamental “laws of anthropo-sociology,” and the selective influences of migratory movements and the growth of cities.

B.

Take six.

  1. “I have striven with the help of biology, statistics and political economy to formulate what I consider to be the true law of population.” (Nitti.) What is this law? Is it the true law? Why?
  2. Upon what facts rests the assertion that “the fulcrum of the world’s balance of power has shifted from the West to the East, from the Mediterranean to the Pacific”?
  3. What factors determine the rate of suicide? Consider the effect upon the rate of suicide of the sex and age distribution of the population, of the social and physical environment, and of heredity.
  4. Statistical determination of labor efficiency, and the increase of such efficiency during this century.
  5. How far are statistics concerning the number of criminal offenders indicative of the amount of criminality? Statistics of prison populations? Of crimes? What variables enter in to determine the “rate of criminality”? What significance do you attach to such rates?
  6. The statistical method.
  7. Graphics as means of presenting statistical data.

Source: Harvard University Archives. Examination Papers, 1873-1915. Box 5, Examination Papers 1898-99, Bound Volume, pp. 43-44.

_______________________

Railways and other Public Works
Economics 52

52 hf. Railways and other Public Works, under Government and Corporate management. Half-course (second half-year). Tu., Th., and (at the pleasure of the instructor) Sat., at 1.30. Mr. [Hugo Richard] Meyer.

In this course it is proposed to review the history and working of different modes of dealing with railway transportation, and to deal summarily with other similar industries, such as the telegraph, street railways, water and gas supply. Consideration will be given to the economic characteristics of these industries, the theory and history of railway rates, the effects of railway service and railway charges on other industries, the causes and consequences of monopoly conditions. The history of legislation in the more important European countries will be followed, as well as the different modes in which they have undertaken the regulation and control of private corporations, or have assumed direct ownership, with or without management and operation. Some attention will be given also to the experience of the British colonies, and more especially of those in Australia. In the United States, there will be consideration of the growth of the great systems, the course of legislation by the federal government, the working of the Interstate Commerce Act, and the modes of regulation, through legislation and through Commissions, at the hands of the several States. So far as time permits, other industries, analogous to railways, will be discussed in a similar manner.

Written work, in the preparation of papers on assigned topics, will be expected of all students in the course.

Source: Harvard University. Faculty of Arts and Sciences. Division of History and Political Science Comprising the Departments of History and Government and Economics, 1897-98, pp. 37-38.

Economics 52: Enrollment
1897-98

[Economics] 52. Mr. Meyer. — Public Works, Railways, Postal and Telegraph Service, and Monopolized Industries, under Corporate and Public Management. Hf. 3 hours. 2d half year.

Total 65: 31 Seniors, 16 Juniors, 8 Sophomores, 10 Others.

Source: Harvard University. Report of the President of Harvard College, 1897-1898, p. 78.

1897-1898.
Economics 52.
[Final Examination]

  1. “The principle [of railway rates] commonly advocated by the antagonists of the railways, as well as by the would-be reformers, is that of cost of service. Charges should be regulated in accordance with the cost of the particular transaction to the company. This is certainly not the actual method. Is it the correct method?”
    Give your reasons for accepting or rejecting the “cost of service” principle.
  2. What were the causes of the so-called granger agitation of 1871-74; of the reappearance of this agitation in 1886-88?
  3. What were the principal reasons for the instability of railway pools in the United States?
  4. By what means did the Trunk Line Associations which succeeded the Trunk Line Pool seek to limit competition and attain the effects of pooling?
  5. Discuss the working of the Interstate Commerce Act under the following headings:—
    The prohibition of undue or unreasonable preference or advantage and the prohibition of pooling.
    The construction by the United States Courts of the clause that the findings of the Commission shall be prima facie evidence in judicial proceedings.
    Legal embarrassments and other obstacles encountered by the Commission in obtaining testimony in penal cases.
    The attitude of the railways to the Act.
  6. The history of the application of the long and short haul clause to competitive rates made by railways not subject to competition from railways which are beyond the jurisdiction of the Interstate Commerce Commission; and to rates on imported commodities. Discuss under the following heading:—
    “The construction put upon the long and short haul clause by the Interstate Commerce Commission; by the United States Supreme Court.
  7. Discuss the working of the German legislation prescribing for distances over 100 km a uniform rate per ton per kilometer.
    Should you expect the practice of equal mileage charges to work with more friction or with less in the United States than in Germany?
    Alternative:
    The important points of difference between the management of the Prussian State Railways and the management of the Australian State Railways; between the management of the English Railways and the management of the American Railways.
  8. The reasons for the failure of the De Freycinet (1879) railway construction schemes; and the effect upon the French Budget of the “agreements” negotiated in 1883 between the French Government and the Six Companies.
    Alternative:
    The effect upon the Italian Budget of the “conventions” made in 1885 between the Italian Government and the Three Companies. The effect upon the Italian Exchequer of the railway construction carried out under the act of 1879 and the supplementary acts of 1881, 1882, and 1885.

Source: Harvard University Archives. Examination Papers, 1873-1915. Box 5, Examination Papers 1898-99, Bound Volume, pp. 44-45.

_______________________

Theory and Methods of Taxation
Economics 71

*71 hf. The Theory and Methods of Taxation, with special reference to local taxation in the United States. Half-course (first half-year). Tu., Th., and (at the pleasure of the instructor) Sat., at 1.30. Professor [Frank William] Taussig.

Course 71 undertakes an examination of the theory of taxation, based upon the comparative study of methods as practised in different countries and in different States of the American Union. This examination necessarily includes some discussion of leading questions in revenue legislation, such as the taxation of incomes and personal property, the single tax, progressive taxation, and indirect taxes.

Source: Harvard University. Faculty of Arts and Sciences. Division of History and Political Science Comprising the Departments of History and Government and Economics, 1897-98, p. 38.

Economics 71: Enrollment

[Economics ] 71. Professor Taussig.—The Theory and Methods of Taxation, with special reference to Local Taxation in the United States. 2 or 3 hours. 1st half year.

Total 42: 5 Graduates, 27 Seniors, 9 Juniors, 1 Sophomore.

Source: Annual Reports of the President and the Treasurer of Harvard College 1897-98, (Cambridge, Massachusetts, 1899), p. 78.

Economics 71.
Readings

Seligman—Essays in Taxation.
Bastable—Public Finance.
Leroy-Beaulieu—Science des Finances, Vol. I.
Say—Dictionnaire des Finances.
Quarterly Journal of Economics, cited as Q. J. E.
Dowell—History of Taxation in England.

PRELIMINARY QUESTIONS: CLASSIFICATION.

Seligman, Ch. IX.
Bastable, Bk. II, Ch. I; Bk. III, Ch. 1

TAXES ON LAND.

{Leroy-Beaulieu. Bk. II, Ch. VI;
Say, article “Foncière (Contribution).” 233-241.}
Bastable, Bk. IV, Ch. I.
Dictionary of Political Economy, article “Land Tax.”

HABITATION TAXES.

{Leroy-Beaulieu, Bk. II, Ch. VII.
Say, article “Personelle-Mobilière,” 850-857.}
Dowell, Vol. III, 186-192.

INCOME TAXES.

Leroy-Beaulieu, Bk. II, Ch. X.
Bastable, Bk. IV, Ch. IV.
{Dowell, Vol. III, 99-122;
Article “Income Tax in the United Kingdom,” in Dictionary of Political Economy, Vol. II.}
J. A. Hill, The Prussian Income Tax, Q. J. E., January, 1892.
Seligman, Ch. X, iii, iv.

BUSINESS TAXES.

{Say,  article “Patentes,” pp. 743-752;
Leroy-Beaulieu, Bk. II, Ch. VIII.}
J. A. Hill—The Prussian Business Tax, Q. J. E., October, 1893.

SUCCESSION TAXES.

Seligman, Ch. V; Ch. IX, i.
Bastable, Bk. III, Ch. III.

PROGRESSION.

{Leroy-Beaulieu, Bk. II, Ch. II;
Bastable, Bk. III, Ch. III.}
Seligman, Progressive Taxation, pp. 190-200; pp. 39-53 (Switzerland).

DIRECT TAXES BY THE UNITED STATES.

C. F. Dunbar,The Direct Tax of 1861, Q. J. E., July, 1889; Vol. III, pp. 436-446.
J. A. Hill,The Civil War Income Tax, Q. J. E., July, 1894.
C. F. Dunbar, The New Income Tax, Q. J. E., October, 1894.

LOCAL TAXES IN ENGLAND.

Blunden, Local Taxation and Finance, Ch. III, IV, V.

LOCAL TAXATION IN THE UNITED STATES.

Seligman, Ch. II, IV, VI, XI.
Ely, Taxation in American States, part III, Ch. VII.
Plehn, The General Property in California, (Economic Studies, Vol. II, No. 3), Part II, 151-178.
Angell, The Tax Inquisitor System in Ohio, in Yale Review, February, 1897.

Source: Harvard University Archives. Syllabi, course outlines and reading lists in Economics, 1895-2003 (HUC 8522.2.1) Box 1, folder “1897-1898”.

1897-98.
Economics 71.
[Mid-year Examination]

[Arrange your answers strictly in the order of the questions. Give and answer, however brief, to each question]

  1. Consider which of the following combinations, if any, bring about “double taxation”: (1) the impôt sur la propriété batie and the personelle-mobilière, in France; (2) local rates and schedule A of the income tax, in Great Britain; (3) the taxation of mortgaged property and of mortgages, as commonly provided for in American States.
  2. It has been said that the taxation of merchants’ stock in trade in Massachusetts, by assessors’ estimate, if effect proceeds in a somewhat similar fashion to that of the French impôt des patentes and of the Prussian business tax. Why? or why not?
  3. Are there good reasons for taxing funded incomes at a higher rate than unfunded?
  4. It has recently been proposed in Great Britain to impose a general tax on property, based on the income tax returns, and levied at the rate of (say) five per cent. on the income derived from the property; reducing at the same time the income tax to one-half its present rate. Point out what important changes in the British tax system would result; consider what examples in other countries may have suggested the proposal: and give an opinion as to its expediency.
  5. What do you conceive to be the “compensatory theory” in regard to progressive taxation?
  6. What reasoning pertinent in regard to the principle of progression in taxation is also pertinent in regard to taxes on successions? in regard to the single tax?
  7. As between owner and occupier of real estate who is responsible for local rates in England? for local taxes in the United States? Do you believe that the differences have important consequences in the incidence of these taxes?
  8. Consider points of resemblance, points of difference, in the modes in which the States of Massachusetts and Pennsylvania tax (1) domestic corporations (2) the securities issued by foreign corporations.
  9. What grounds are there in favor, what against, the imposition of income taxes by the several States?

Source: Harvard University Archives. Harvard University, Mid-year examinations, 1852-1943. Box 4. Bound Volume: Examination Papers, Mid-Years 1897-98.

Also: Harvard University Archives. Examination Papers, 1873-1915. Box 5, Examination Papers 1898-99, Bound Volume, pp. 47-48.

Also: Harvard University Archives. Examination papers in economics 1882-1935, Prof. F. W. Taussig. Scrapbook. (HUC 7882), p. 61.

_______________________

Financial Administration and Public Debts
Economics 72

*72 hf. Financial Administration and Public Debts. Half-course (second half-year). Tu., Th., Sat., at 11. Professor [Charles Franklin] Dunbar.

Course 72 is devoted to an examination of the budget systems of leading countries, and their methods of controlling expenditure, the methods of borrowing and of extinguishing debts practised by modern states, the form and obligation of the securities issued, and the general management of public credit.

Topics will be assigned for investigation by the students, and a list of topics, references, and required reading will be used.

Source: Harvard University. Faculty of Arts and Sciences. Division of History and Political Science Comprising the Departments of History and Government and Economics, 1897-98, p. 38.

Enrollment data not published for 1897-98.

1897-98.
Economics 72
[Final Examination]

  1. What are the comparative advantages of (a) an Independent Treasury like that of the United States, and (b) the use of a bank or banks by the government, as practised in England or Germany?
  2. What changes (if any) of constitution, law or practice would be required, in order to establish a thorough-going budget system in the United States?
  3. Compare the French budget procedure with the English, and point out their respective advantages or disadvantages.
  4. Suppose a fiscal year to have ended before financial measures for the new year have been agreed upon. How would current expenditure be provided for in the United States? In England? In France? In Germany?
  5. What is the practice of those four countries respectively as regards the control of revenue by means of annual grants?
  6. Suppose the case of a country having a depreciated paper currency, but expecting the ultimate resumption of specie payments, and compelled to borrow on a large scale. Which method of borrowing upon bonds (principal and interest payable in gold) would be the best,—
    (a), To sell the bonds for par in gold and make the rate of interest high enough to attract buyers;
    (b) To sell the bonds for gold at such discount as might be necessary, their interest being fixed, say, at six per cent;
    (c) To sell the bonds for their nominal par in depreciated paper. Give the reasons for and against each method.
  7. State the probable effect on the selling value of bonds when their terms provide for, —
    (a) Annual drawings by lot for payment;
    (b) Reserved right to pay at pleasure after some fixed date;
    (c) Obligation to pay at some fixed date;
    (d) “Limited option” like that of the “five-twenties.”
  8. Examine the reasoning involved in the following expression of opinion:—
    “There is one essential difference between the anticipation of interest. payments, and the anticipation of the payment of the principal of a debt by purchases on the market. This latter procedure…requires a larger sum of money to extinguish a given debt than will be required after the debt comes to be redeemable; but no such result follows the anticipation of interest-payments. These are determined by the terms of the contract, and may be calculated with accuracy. The interest does not, like the market value of a debt, fall as the bonds approach the period of their redemption, and it is but the application of sound business rules to use any surplus money on hand in making advanced payments of interest.”
  9. Describe the existing arrangements for the reduction of the English debt.
  10. State, with reasons, your own conclusion as to the real advantage (if any) derived from the system of terminable annuities.

Source: Harvard University Archives. Examination Papers, 1873-1915. Box 5, Examination Papers 1898-99, Bound Volume, pp. 48-49.

_______________________

Banking and the History of the leading Banking Systems
Economics 122

*121 hf. Banking and the History of the leading Banking Systems. Half -course (first half-year). Tu., Th., Sat., at 11. Professor [Charles Franklin] Dunbar.]

[Note: originally announced as omitted for 1897-98.]

In Course 12[1] the modern system of banking by deposit and discount is examined, and its development in various countries is studied. The different systems of note-issue are then reviewed and compared, and the relations of banks to financial crises carefully analyzed. Practical banking does not come within the scope of this course. The study is historical and comparative in its methods, requiring some examination of important legislation in different countries, practice in the interpretation of banking movements, and investigation of the general effects of banking. The course, therefore, naturally leads to an examination of the questions now raised as to bank issues in the United States.

Source: Harvard University. Faculty of Arts and Sciences. Division of History and Political Science Comprising the Departments of History and Government and Economics, 1897-98, pp. 38-39.

Economics 12: Enrollment

[Economics ] 121. Professor Dunbar.—Banking and the History of the leading Banking Systems. Hf. 3 hours. 1st half year.

Total 12: 5 Graduates, 4 Seniors, 2 Juniors, 1 Other.

Source: Annual Reports of the President and the Treasurer of Harvard College 1897-98, (Cambridge, Massachusetts, 1899), p. 78.

1897-98.
Economics 121.
[Mid-year Examination]

A.

Give ONE THIRD of your time to these two questions.

  1. Suppose that, in the period 1848-70, India had had a banking system as extensively used and as efficient as that of England or the United States, and that in the East prices had depended upon competition as much as they did in the Western nations? How would these altered conditions have affected the drain of silver to India, and the value of the precious metals in America and Europe?
  2. What do you say to the general proposition, that England, “being a debtor nation,” can draw gold at pleasure from any part of the world?

B.

  1. A few years ago an American writer said:—
    “We will be able to resume specie payments when we cease to rank among the debtor nations, when our national debt is owed to our own people, and when our industry is adequate to the supply of the nation’s need of manufactured goods.”
    To what extent should you regard the circumstances of the resumption in 1879 as a verification of the reasoning implied in this statement?
  2. In what way did the payment of the French Indemnity, 1871-73, tend to stimulate affairs in England, Austria, and the United States?
  3. What economic conditions or events tended to make the year 1890 a turning point, both in domestic and in international finance? Give a clear statement of such as you recall.
  4. How do the banking and currency systems of England, France and the United States differ, as regards their ability respectively to resist export movements of gold?
  5. What temporary changes in the general level of prices in this country should you expect to see, as the result of a large permanent withdrawal of foreign capital? What ultimate change of prices should you expect?
  6. State the general conditions which determine the movement of gold as it issues from the mining countries and is distributes over the world?
  7. Cairnes discusses some of the conditions which determine the relative quickness with which countries raise their general scale of prices when a rapid depreciation of gold is in progress. Consider how far the effect upon a given country would be influenced by the fact that its exports were

(a) chiefly manufactured articles;
(b) chiefly articles of food.

Source: Harvard University Archives. Harvard University, Mid-year examinations, 1852-1943. Box 4. Bound Volume: Examination Papers, Mid-Years 1897-98.
Also: Harvard University Archives. Examination Papers, 1873-1915. Box 5, Examination Papers 1898-99, Bound Volume, pp. 51-52.

_______________________

International Payments and the Flow of the Precious Metals
Economics 121

[Was not offered first nor second term, instead see above]

[* 121 hf. International Payments and the Flow of the Precious Metals. Half-course (first half-year). Tu., Th., Sat., at 11. Professor [Charles Franklin] Dunbar and Mr. [Hugo Richard] Meyer.

Course 121 is taken up with the discussion of the movements of goods, securities, and money, in the exchanges between nations and in the settlement of international demands. After a preliminary study of the general doctrine of international trade and of the use and significance of bills of exchange, it is proposed to make a close examination of some cases of payments on a great scale, and to trace the adjustments of imports and exports under temporary or abnormal financial conditions. Such examples as the payment of the indemnity by France to Germany after the war of 1870-71, the distribution of gold by the mining countries, and the movements of the foreign trade of the United States since 1879, will be investigated and used for the illustration of the general principles regulating exchanges and the distribution of money between nations.

Source: Harvard University. Faculty of Arts and Sciences. Division of History and Political Science Comprising the Departments of History and Government and Economics, 1897-98, pp. 38-39.]

_______________________

Selected Topics in the Financial Legislation of the United States
Economics 162

*162 hf. Selected Topics in the Financial Legislation of the United States. Half-course (second half-year). Tu., Th., at 2.30. Professor [Charles Franklin] Dunbar.

The topics for study in this course for 1897-98 will be: (1) The Legal Tender Issues of the Civil War; (2) Development of the National Banking System. Subjects will be assigned and reports called for, requiring thorough investigation in the debates of Congress and other contemporary sources of information, for the purpose of tracing the history and significance of the legislative acts to be discussed, and a close study of such financial and commercial statistics as may throw light upon the operation of the acts.

Arrangements will be made by which graduate students and candidates for Final Honors in Political Science may take this course in connection with the Seminary in Economics as a full course running through the year.

Source: Harvard University. Faculty of Arts and Sciences. Division of History and Political Science Comprising the Departments of History and Government and Economics, 1897-98, p. 39.

Economics 162: Enrollment

[Economics ] 162. Professor Dunbar.—Selected Topics in the Financial History of the United States. Hf. 2 hours. 1st half year.

Total 8: 3 Graduates, 3 Seniors, 2 Juniors.

Source: Annual Reports of the President and the Treasurer of Harvard College 1897-98, (Cambridge, Massachusetts, 1899), p. 78.

1897-98.
Economics 162
[Year-end Examination]

A.

Give one-half of the time allowed for this examination to the discussion of any two of the questions stated under B.

B.

Answer, with such fulness as the remaining time allows, those of the following questions which you have not selected for discussion under A.

  1. Rhodes (History of the United States since 1850, iii., 567) states as “the conclusion which it seems to me a careful consideration of all the facts must bring us to,” that “The Legal Tender act was neither necessary nor economical.”
    Discuss this conclusion.
  2. In December, 1868, Senator Morton introduced a bill providing that specie payments should be resumed, by the government July 1, 1871, and by the banks January 1, 1872, greenbacks ceasing to be a legal tender at the latter date; gold to be provided in the Treasury by the accumulation of surpluses and by the sale of bonds, but no greenbacks to be redeemed until the date fixed for resumption by the United States.
    What would have been the probable operation of such a measure?
  3. Sherman said in January, 1874,—
    “The plan, which in my judgment presents the easiest and best mode of attaining specie payments, is to choose some bond of the United States which in ordinary times, by current quotations, is known to be worth par in gold in the money markets of the world, where specie is alone the standard of value, and authorize the conversion of notes into it.”
    Discuss the probable working of such a plan, having in view also Mr. Sherman’s strong objection to a contraction of the currency
  4. Suppose an Issue department of the Treasury, completely separated from all other business, provided with an ample reserve and strictly limited to the exchange of coin for notes and notes for coin as required by the public; what would you say would then be the nature and the force of the objections, if any, to the permanent maintenance of our legal tender issues?
  5. The greenbacks having been regarded originally as the temporary element in our paper currency and the bank notes as the permanent element, what were the one or two great turning points in the development which reversed this relation?
  6. If the issue of bank-notes were made equally available for all parts of the country, so far as the requirements of the system are concerned, would the South and South West find themselves more amply provided with paper currency than at present?
  7. What in your judgment is the most important function discharged by banks in this country, and what is your estimate of the importance and practicability of national supervision of their discharge of that function?
  8. The act just passed by Congress to provide ways and means for the expenditures occasioned by the war, contains the following section:—
    “That the Secretary of the Treasury is hereby authorized and directed to coin into standard silver dollars as rapidly as the public interests may require, to an amount, however, of not less than one and one-half millions of dollars in each month, all of the silver bullion now in the Treasury purchased in accordance with the provisions of the act approved July 14, 1890, entitled “An act directing the purchase of silver bullion and the issue of Treasury notes thereon, and for other purposes, and said dollars, when so coined, shall be used and applied in the manner and for the purposes named in said act.”
    State carefully the use and application of the dollars thus required by the act of 1890.

Harvard University Archives. Examination Papers, 1873-1915. Box 5, Examination Papers 1898-99, Bound Volume, pp. 554-55.

_______________________

Economics Seminary
Economics 20

20. Seminary in EconomicsMon., at 4.30. Professors [Charles Franklin] Dunbar, [Frank William] Taussig, and [William James] Ashley, and Asst. Professor Edward Cummings.

In the Seminary the instructors receive Graduate Students, and Seniors of high rank and adequate preparation, for training in investigation and discussion. No endeavor is made to limit the work of the Seminary to any one set of subjects. Subjects are assigned to students according to their needs and opportunities, and may be selected from any of the larger fields covered by the courses in which stated instruction is given. They may accordingly be in economic theory, in economic history, in applied economics, in sociology, or in statistics. It will usually be advisible for members of the Seminary to undertake their special investigation in a subject with whose general outlines they are already acquainted; but it may sometimes be advantageous to combine general work in one of the systematic courses with special investigation of a part of the field.

The general meetings of the Seminary are held on the first and third Mondays of each month. The members of the Seminary confer individually, at stated times arranged after consultation, with the instructors under whose special guidance they are conducting their researches.

At the regular meetings, the results of the investigations of members are presented and discussed. The instructors also at times present the results of their own work, and give accounts of the specialized literature of Economics. At intervals, other persons are invited to address the Seminary on subjects of theoretic or practical interest, giving opportunity for contact and discussion with the non-academic world. Among those who thus contributed to the Seminary in 1895-97 were President Francis A. Walker, Dr. Frederick H. Wines, Mr. S. N. D. North, Mr. A. T. Lyman, Mr. E. W. Hooper, and Mr. F. C. Lowell.

In 1896-97 the Seminary had fifteen members, of whom twelve were Graduate Students, two were Seniors in Harvard College, and one was a Law Student. Among the subjects under investigation in that year were: The Woollen Industry in England during the 17th and 18th centuries; Over-production and Over-accumulation in Economic Theory; The Taxation of Sugar in the United States and in Foreign Countries; The National Banking System with regard to its operation in the West and South; The Financial History of the Pennsylvania Railway; The Financial History of the Union Pacific Railway; The History of Immigration into the United States.

Source: Harvard University. Faculty of Arts and Sciences. Division of History and Political Science Comprising the Departments of History and Government and Economics, 1897-98, pp. 39-40.

Economics 20: Enrollment

[Economics ] 20. Professors Dunbar, Taussig and Ashley, and Asst. Professor Edward Cummings.—Investigation of topics assigned after consultation.

Total 12: 11 Graduates, 1Senior.

Source: Annual Reports of the President and the Treasurer of Harvard College 1897-98, (Cambridge, Massachusetts, 1899), p. 78.

Members of the Harvard Economics Seminary, 1897-1898

https://www.irwincollier.com/harvard-members-of-the-economics-seminary-1897-1898/

Image Source: Harvard Hall (1906). From the Center for the History of Medicine (Francis A. Countway Library of Medicine).

Categories
Economics Programs Exam Questions Harvard

Harvard. Mid-year and Year-End Final Exams in Economics and Social Ethics, 1896-1897

 

The collection of transcribed Harvard semester examinations here at Economics in the Rear-view Mirror is growing slowly. This post adds the exam questions from 1896-1897 for all the economics courses and for Francis Peabody’s philosophy course “The Ethics of the Social Questions”. 

_____________________

From the Preface to the Announcements
for 1896-97

The courses primarily for Undergraduates are open (and in some cases recommended) to Graduate Students, but are not ordinarily counted towards any of the higher degrees. The courses for Graduates and Undergraduates are, under certain limitations, open to any properly qualified student. To the courses primarily for Graduates Undergraduates are admitted only on the recommendation of the Instructor.

No starred (*) course and no course of research can be taken without the previous consent of the Instructor….

By recent action of the Governing Boards, the requirement of two years of residence at this University of a candidate for the degree of Doctor of Philosophy or Doctor of Science has been rescinded. The minimum requirement of residence in now one year, this period being fixed for all degrees by the Statutes. It is not the purpose of the change thus made to lower the standards for these two degrees; but only to reduce the amount of compulsory residence at this University. (April, 1896).

Source: Harvard University. Announcement of the Courses of Instruction Provided by the Faculty of Arts and Sciences for the Academic Year 1896-1897, pp. iii-iv.

_____________________

Philosophy 5.
The Ethics of the Social Questions.
1896-1897

Course Announcement

[Philosophy] 5. The Ethics of the Social Questions. — The problems of Poor-Relief, the Family, Temperance, and various phases of the Labor Question in the light of ethical theory. — Lectures, special researches, and required reading. Tu., Th., Sat., at 10. Professor Peabody.

Source: Harvard University. Announcement of the Courses of Instruction Provided by the Faculty of Arts and Sciences for the Academic Year 1896-1897, p. 26.

Course Enrollment

[Philosophy] 5. Professor Peabody. — The Ethics of the Social Questions. — The problems of Poor-Relief, the Family, Temperance, and various phases of the Labor Question in the light of ethical theory. — Lectures, special researches, and required reading. 3 hours.

Total 56: 1 Graduate, 31 Seniors, 10 Juniors, 3 Sophomores, 11 Others.

Source: Harvard University. Report of the President of Harvard College 1896-1897, p. 66.

*  *  *  *  *
[Mid-year examination,
still to be added]

*  *  *  *  *

Philosophy 5.
THE ETHICS OF THE SOCIAL QUESTIONS.
Year-End Examination,
1896-97

[This paper should be considered as a whole. The time should not be exhausted in answering a few questions, but such limit should be given to each answer as will permit the answering of all the questions in the time assigned.]

  1. Indicate, briefly, the place in the history of the modern Labor Question of:
    Chalmers;
    Von Ketteler;
    Lassalle;
    The Rochdale Pioneers;
    Carlyle;
  2. Ruskin as an Economist.
  3. The anarchist’s criticism of the socialist; the socialist’s criticism of the anarchist, and the communist as he is criticised by both.
  4. What do you understand to be the “quintessence” of socialism, as expounded by Schäffle; and what criticisms on this whole social programme appear to you most serious?
  5. Arbitration and conciliation — their differences, varieties, advantages, and limitations.
  6. The history of co-operation in Great Britain, its fortunes in the United States, and the conditions of its success.
  7. Various types of industrial partnership, — their special advantages and limitations.
  8. The Scandinavian Licensing System compared with the present Massachusetts Liquor Law. (Fanshawe, 187-229.)
  9. Sum up, briefly, the general doctrine of social duty which our study of various social questions is intended to illustrate.

Source: Harvard University Archives. Harvard University Examination Papers, 1873-1915. Box 4. Bound volume: Examination Papers 1896-97. Papers Set for Final Examinations in Philosophy, History, Government, Economics, Fine Arts, Architecture, and Music in Harvard College, June 1897, p. 7.

_____________________

ECONOMICS.
Primarily for Undergraduates.

_____________________

Economics 1.
Outlines of Economics,
1896-97

Course Announcement

[Economics] 1. Outlines of Economics. — Principles of Political Economy. — Lectures on Economic Development, Social Questions, and Financial Legislation. Mon., Wed., Fri., at 9. Professors Taussig and Ashley, Asst. Professor Edward Cummings, and Dr. John Cummings.

Source: Harvard University. Announcement of the Courses of Instruction Provided by the Faculty of Arts and Sciences for the Academic Year 1896-1897, p. 33.

Course Enrollment

[Economics] 1. Professors Taussig and Ashley, Asst. Professor Edward Cummings, and Dr. John Cummings. Outlines of Economics. — Principles of Political Economy. — Lectures on Economic Development, Social Questions, and Financial Legislation. 3 hours.

Total 464: 1 Graduate, 40 Seniors, 131 Juniors, 235 Sophomores, 12 Freshmen, 45 Others.

Source: Harvard University. Report of the President of Harvard College 1896-1897, p. 65.

Economics 1.
Mid-year Examination,
1896-97

  1. “Productive labor is that which produces utilities fixed and
  2. embodied in material objects. All other labor, however useful, is classed as unproductive.” Why? or why not?
  3. Capital is the result of saving: capital is produced by labor; all capital is consumed. Can you reconcile these propositions?
  4. “Those circumstances of a country, in which population can with impunity increase at its utmost rate, are rare and transitory.” What are they? and why rare? What is the utmost rate?
  5. Give examples of (a) differences in wages arising from different degrees of attractiveness in different employments; (b) differences arising from natural monopolies.
  6. Suppose a tax were imposed on land precisely equal to the economic rent paid for its use; could the owner of the land shift the tax to his tenant by charging a higher rent than before?
  7. What is meant when it is said that rent does not enter into the cost of production?
  8. It is said to be immaterial whether a community has a large or a small stock of money; and it is said to be harmful for a community to resort to inconvertible paper. Can both of these propositions be sound?
  9. “With enormous shortsightedness, the people of the United States send abroad every year over one hundred millions of dollars, with which to pay for sugar which might have been produced at home.” Why, or why not, is there shortsightedness in this operation?
  10. Wherein does a country gain, if other countries demand more of its exports?
  11. In the stationary state. as described by Mill, what determines the rate of interest? the rate of wages?

Source: Harvard University Archives. [Examinations] Scrapbook of F. W. Taussig, p. 58.

Economics 1.
Year-end Examination,
1896-97

[Answer nine questions, selecting at least one from each of the four groups. Arrange your answers strictly in the order of the questions selected. Give your reasons in all cases.]

I.

  1. Mention a case in which the income received for the use of a piece of real estate is to be regarded as rent; one in which it is to be regarded as interest; and one in which the classification would be doubtful.
  2. Mention a commodity whose value is permanently governed by cost of production; one whose value is permanently governed by the equation of demand and supply; and one whose value is permanently affected by both causes.
  3. Does the gain from foreign trade arise from the sale of exports? from the purchase of imports?

II.

  1. Is the law of Rent stated by Mill applicable either to the German peasant of the early part of the century or to the American farmer of to-day? Give your reasons.
  2. What are the functions of the entrepreneur? Give some account of the origin of the entrepreneur
  3. Illustrate the influence of the feeling of nationality as an economic factor.

III.

  1. How far have English trade unions tried to control (a) the wages of labor? (b) the supply of labor? In what respects have they been most useful?
  2. What do you understand by the rise of the modern factory system? In what sense is this system responsible for what is known as the labor problem?
  3. If all the productive and distributive business of a community were in the hands of cooperative societies would the labor question cease to exist?

IV.

  1. Point out wherein the deposits of a bank resemble its notes, and wherein they differ from its notes; and consider why one or the other should be regarded as part of the circulating medium of the community.
  2. What was the amount outstanding (in round numbers) of legal-tender notes in 1867? in 1877? in 1897? What were the laws under which the notes outstanding at those several dates had been issued, and what was the state of the legislation then in force for their redemption?
  3. Wherein does the Reichsbank of Germany, as to its management of notes and deposits, resemble the Bank of France? Wherein the Bank of England?

Source: Harvard University Archives. Harvard University Examination Papers, 1873-1915. Box 4. Bound volume: Examination Papers 1896-97. Papers Set for Final Examinations in Philosophy, History, Government, Economics, Fine Arts, Architecture, and Music in Harvard College, June 1897, pp. 37-38.

_____________________

ECONOMICS
For Graduates and Undergraduates.

The Courses for Graduates and Undergraduates are open to students who have passed satisfactorily in Course 1. Courses 5, 6, 7, 8, and 9 are also open to Juniors and Seniors of good rank who take Course 1 at the same time; and Course 11 is open to students who have taken either Economics 1 or History 1.

_____________________

Economics 10.
The Mediaeval Economic History of Europe

[[Economics] *10. The Mediaeval Economic History of Europe. Tu., Th., (and at the pleasure of the instructor) Sat., at 12. Professor Ashley.]

Omitted in 1896-97. Courses 10 and 11 are usually given in alternate years.

Source: Harvard University. Announcement of the Courses of Instruction Provided by the Faculty of Arts and Sciences for the Academic Year 1896-1897, p. 33.

_____________________

Economics 11.
The Modern Economic History of Europe and America (from 1600),
1896-97

Course Announcement

[Economics] 11. The Modern Economic History of Europe and America (from 1600). Tu., Th., (and at the pleasure of the instructor) Sat., at 12. Professor Ashley.

Courses 10 and 11 are usually given in alternate years.

Source: Harvard University. Announcement of the Courses of Instruction Provided by the Faculty of Arts and Sciences for the Academic Year 1896-1897, p. 33.

Course Enrollment

[Economics] 11. Professor Ashley. The Modern Economic History of Europe and America (from 1600). 2 hours.

Total 29: 16 Graduates, 6 Seniors, 1 Junior, 1 Sophomore, 5 Others.

Source: Harvard University. Report of the President of Harvard College 1896-1897, p. 65.

Economics 11.
Mid-Year Examination,
1896-97

N. B. — Not more than eight questions must be attempted.

  1. Give some account of Asiatic commerce in the middle ages.
  2. Enumerate very briefly some of the reasons for which Genoa deserves attention in economic history.
  3. “The mooste part of the lordes have enclosed their demeyn lands and meadows and kept them in severalties.” (Fitzherbert, c. 1530). Explain the nature and effects of the action here described.
  4. What features, if any, were common to the Peasant Risings in the different countries of Western Europe in the fourteenth, fifteenth, and sixteenth centuries?
  5. Compare the action of the English government in relation to industry in the sixteenth century with that of the French government.
  6. “Our merchants may do well to provide for the Russians such wares as the Dutch nation doth serve them of.” Explain and comment.
  7. Describe the industrial condition of Norfolk in the time of Defoe.
  8. Explain, with illustrations, what is meant by the “territorial” period in German economic development.
  9. What impressions do you derive from Defoe’s Essay upon Projects as to the constitution, temper and interests of the business circles of London in his time?
  10. Explain the following terms Droit de vaine pâture, Société en commandite, Niederlegung von Hufen, Hausindustrie, Fondaco.
  11. Give a critical account of any really important work (not on the printed list) of which you have read any considerable portion in connection with this course.

Source: Harvard University Archives. Harvard University Mid-Year Examination, 1852-1943. Box 4. Bound volume: Examination Papers, Mid-Years 1896-97.

Economics 11.
Year-end Examination,
1896-97

N.B. – Not more than eight questions must be attempted.

  1. “The fifteenth century was the golden age of the English labourer.” What is the evidence for that contention? How is that evidence to be interpreted?
  2. Explain the part played by Hamburg in the economic history of the sixteenth and seventeenth centuries.
  3. Give some account of the history, since the massacre of Amboyna, of the Dutch East Indian Empire.
  4. Indicate very briefly the chief points in Schmoller’s account of mercantilist policy. In what directions does it seem open to criticism?
  5. State the causes and criticize the alleged consequences of the drain of specie from the English colonies in the eighteenth century.
    [Not to be taken by those who have written theses on the Navigation Act.]
  6. “English industries could not have advanced so rapidly without protection.” Examine this statement.
  7. Describe the main features of English industrial life in the early part of the eighteenth century.
  8. What changes did the French Revolution make in the position of the rural population of France.
  9. Discuss the application of the Infant Industries argument to the United States during the early decades of the present century.
  10. Give some account of the competition between railroads and canals as means of transportation.
  11. What exactly were the English “Corn Laws,” repealed in 1846? Have the anticipations of Cobden been realized?
  12. “You made me look rather a fool, Arminius,” I began, “by what you primed me with in Germany last year about Stein settling your land question.” “I dare say you looked a fool,” says my Prussian boor, “but what did I tell you?” “Why,” says I, “you told me Stein had settled a land question like the Irish land question, and I said so in the Cornhill Magazine, and now the matter has come up again by Mr. Bright talking at Dublin of what Stein did, and it turns out he settled nothing like the Irish land question at all, but only a sort of title-commutation affair.” “Who says that?” asked Arminius. “A very able writer in the Times,” I replied. — May we have your opinion?

Source: Harvard University Archives. Harvard University Examination Papers, 1873-1915. Box 4. Bound volume: Examination Papers 1896-97. Papers Set for Final Examinations in Philosophy, History, Government, Economics, Fine Arts, Architecture, and Music in Harvard College, June 1897, p. 46.

_____________________

Economics 15.
The History and Literature of Economics to the Middle of the Nineteenth Century,
1896-97

Course Announcement

[Economics] *15. The History and Literature of Economics to the Middle of the Nineteenth Century. Mon., Wed., (and at the pleasure of the instructor) Fri., at 12. Professor Ashley.

Source: Harvard University. Announcement of the Courses of Instruction Provided by the Faculty of Arts and Sciences for the Academic Year 1896-1897, p. 34.

Course Enrollment

[Economics] 15. Professor Ashley. — The History and Literature of Economics to the Middle of the Nineteenth Century. 2 hours.

Total 14: 10 Graduates, 2 Seniors, 2 Juniors.

Source: Harvard University. Report of the President of Harvard College 1896-1897, p. 65.

Economics 15.
Mid-Year Examination,
1896-97

N.B. — Not more than eight questions must be attempted.

  1. Explain the influence of the Greek conception of the State upon the economic speculation of Greek philosophers.
  2. “The sacredness of property is a notion far more fixed in modern than in ancient times.” Comment upon this remark
  3. Explain and illustrate the influence of the example of Sparta on Greek social thought.
  4. In what sense is it true that Plato anticipated Adam Smith’s teaching concerning division of labour?
  5. In what sense is it true that Aristotle anticipated the modern distinction between Value in Use and Value in Exchange.
  6. Set forth briefly Aristotle’s doctrine as to Chrematistic.
  7. Discuss the question as to whether Christianity destroyed slavery.
  8. What sanction, if any, has Socialism or Communism in the teaching of the Christian Fathers?
  9. Trace the early history of the doctrine of “interest” in the original sense of that word.
  10. Distinguish between the various senses attached to the term Mercantilism. Which do you think most convenient?
  11. What ideas prominent in modern Protectionist argument are absent from Mercantilism as represented by Mun?

Source: Harvard University Archives. Harvard University Mid-Year Examination, 1852-1943. Box 4. Bound volume: Examination Papers, Mid-Years 1896-97.

Economics 15.
Year-End Examination,
1896-97

N.B. — Not more than eight questions must be attempted.

  1. Mention some of the practical questions which called forth economic pamphlets in the period 1650-1700, and give some illustrations.
  2. Illustrate and criticize the attitude of the mercantilist writers towards the regulation of internal industry.
  3. Explain the relation of Locke’s doctrine of price to the immediate purpose of his Considerations.
  4. Describe, as definitely as possible, the relation of the Physiocrats to the other reforming or revolutionary movements of their time.
  5. Criticize Adam Smith’s criticism of Physiocratic doctrine.
  6. What elements in his teaching do you conceive Adam Smith to have derived from Hume?
  7. What does Smith mean by “the component parts of the price of commodities”?
  8. Compare Smith’s definition of capital with that of John Stuart Mill.
  9. What does Smith mean by “the natural rates of wages”?
  10. What bearing has the teaching of Malthus on “Socialism.” Explain in your answer what sort of “Socialism” you have in mind, and mention the sources whence you have derived your impression of it.

Source: Harvard University Archives. Harvard University Examination Papers, 1873-1915. Box 4. Bound volume: Examination Papers 1896-97. Papers Set for Final Examinations in Philosophy, History, Government, Economics, Fine Arts, Architecture, and Music in Harvard College, June 1897, pp. 50-51.

_____________________

Economics 2.
Economic Theory from the Middle of the Nineteenth Century to the Present Time,
1896-97

Course Announcement

[Economics] *2. Economic Theory from the Middle of the Nineteenth Century to the Present Time. — English Writers. — The Austrian School. Mon., Wed., Fri., at 2.30. Professor Taussig.

Source: Harvard University. Announcement of the Courses of Instruction Provided by the Faculty of Arts and Sciences for the Academic Year 1896-1897, p. 34.

Course Enrollment

[Economics] 2. Professor Taussig. — Economic Theory from the Middle of the Nineteenth Century to the Present Time. — English Writers. — The Austrian School. 3 hours.

Total 42: 12 Graduates, 12 Seniors, 13 Juniors, 2 Sophomores, 3 Other.

Source:   Harvard University, Annual Reports of the President and Treasurer of Harvard College, 1896-97, p. 65.

Economics 2.
Mid-Year Examination,
1896-97

  1. “According to Ricardo, the exchange value of commodities contains neither return to capital nor rent, but simply labor.” Why? or why not?
  2. Sketch concisely the development of the general theory of value at the hands of Ricardo, Mill, Cairnes.
  3. “Skill, as skill, produces no effect on value; in other words, commodities do not under any circumstances exchange for each other in proportion to the degree of skill bestowed on them. Skill, though in itself inoperative on value, nevertheless affects it indirectly in two distinct ways; first, where competition is effective among producers, through the cost which must be undergone in acquiring the skill; . . . and secondly, in the absence of competition, through the principle of monopoly.” — Cairnes.
    Explain and illustrate.
  4. “If there really was a national fund the whole of which must necessarily be applied to the payment of wages, that fund could be no other than an aggregate of smaller similar funds possessed by the several individuals who compose the employing part of the nation. Does, then, any individual employer, possess any such fund? Is there any specific portion of any individual’s capital which the owner must necessarily expend upon labour? . . . May he not spend more or less on his family and himself, according to his fancy, — in the one case having more, in the other less, left for the conduct of his business? And of what is left, does he or can he determine beforehand how much shall be laid out on buildings, how much on materials, how much on labour? . . . Be it observed, fixity of definiteness is the very essence of the supposed wages-fund. No one denies that some amount or other must within any given period be disbursed in the form of wages. The only question is, whether that amount be determinate or indeterminate.” — Thornton, On Labour.
    State carefully, and consider critically, the answers Cairnes made to these questions.
  5. Would you accede to the statement that “President Walker’s theory is, in reality, not a theory of manager’s earnings at all, but a theory of differences in manager’s earnings”?
  6. “For an understanding of the machinery by which distribution is accomplished, the classification of sources of income should thus be different from that to be adopted for an explanation of the fundamental causes.” — Taussig.
    Wherein different?
  7. Explain what is meant by Consumer’s Rent; and consider how its significance is affected by inequalities in wealth.
  8. “As a rule, the poorer soils rise in value relatively to the richer, as the pressure of population increases.” — Marshall. Why?

Source: Harvard University Archives. Harvard University Mid-Year Examination, 1852-1943. Box 4. Bound volume: Examination Papers, Mid-Years 1896-97.

Economics 2.
Year-end Examination,
1896-97

  1. Do you believe that a permanent gain for the theory of wages has been made by Walker’s discussion of that subject? If so, wherein? if not, why not?
  2. Does Marshall’s analysis of the different grades of labor, and of the barriers between them, differ in essentials from Cairnes’s? from Mill’s?
  3. Explain what “quasi-rent” is, wherein it differs from true rent, wherein resembles true rent; and state whether the conception seems to you a helpful one, deserving to be permanently embodied in economic theory.
  4. What do you conceive the difference to be between what Walker calls “current product,” Marshall “the national dividend,” and the instructor in the course “real income”?
  5. On what grounds does Marshall maintain that “the extra income earned by natural abilities may be regarded as a rent, when we are considering the sources of the income of individuals, but not with reference to the normal earnings of a trade”? What is your own opinion?
  6. “The attribute of normal value implies systematic and continuous production.” Cairnes. Would Böhm-Bawerk accede to this proposition? Why, or why not? Give your own opinion.
  7. Explain what Böhm-Bawerk means by (subjective) “value”; and consider his analysis of the relation between value and cost.
  8. Enumerate the grounds on which Böhm-Bawerk maintains that “present goods have greater value than future goods of like kind and quantity”; consider to which of these grounds he gives most attention; and give your opinion as to the justice of this emphasis.

Source: Harvard University Archives. Harvard University Examination Papers, 1873-1915. Box 4. Bound volume: Examination Papers 1896-97. Papers Set for Final Examinations in Philosophy, History, Government, Economics, Fine Arts, Architecture, and Music in Harvard College, June 1897, p. 38. Previously transcribed: https://www.irwincollier.com/harvard-history-of-economic-theory-final-exam-questions-taussig-1897-1900/

_____________________

Economics 13.
Scope and Method in Economic Theory and Investigation

[*13 hf. Scope and Method in Economic Theory and Investigation. Half-course. Professor Taussig.]

Omitted in 1896-97; to be given in 1897-98.

Source: Harvard University. Announcement of the Courses of Instruction Provided by the Faculty of Arts and Sciences for the Academic Year 1896-1897, p. 34.

_____________________

Economics 3.
The Principles of Sociology.
1896-97

Course Announcement

[Economics] *3. The Principles of Sociology. —Development of the Modern State, and of its Social Functions. Mon., Wed., (and at the pleasure of the instructor) Fri., at 1.30. Asst. Professor Edward Cummings.

Source: Harvard University. Announcement of the Courses of Instruction Provided by the Faculty of Arts and Sciences for the Academic Year 1896-1897, p. 34.

Course Enrollment

[Economics] 3. Asst. Professor Edward Cummings. — The Principles of Sociology. — Development of the Modern State, and of its Social Functions. 2 hours.

Total 47: 6 Graduates, 23 Seniors, 13 Juniors, 2 Sophomores, 3 Other.

Source:   Harvard University, Annual Reports of the President and Treasurer of Harvard College, 1896-97, p. 65.

Economics 3.
Mid-Year Examination,
1896-97

(It is the purpose of the following questions to elicit first, an intelligent statement of the gist of what has been read or discussed; second, a free statement of an intelligent opinions you may be forming for yourselves.
Arrange your answers in the order in which the questions stand. Limit the discussion of each question to about an hour.)

I.

A critical comparison of Mr. Giddings’ treatise on the principles of Sociology with that of Mr. Spencer:—
(a) In regard to method, arrangement, and terminology :
(b) In regard to fundamental resemblances in the theories presented:
(c) In regard to supposed differences

II.

“Next in order come the problems of the social consciousness, or social mind, including its content of common memories and ideas, its aspirations and its volition.” What explanation of the phenomena in question is offered by (a) Giddings; (b) Spencer; (c) Durkheim; (d) Tarde? What is your opinion of the relative merits and the practical bearing of the several explanations?

IlI.

The significance and the function of the family (a) in the earlier and (b) in the later phases of social evolution.
(c) If you still have time, give some account of the successive theories and of the present state and significance of the controversy in regard to early forms of marriage.

Source: Harvard University Archives. Harvard University Mid-Year Examination, 1852-1943. Box 4. Bound volume: Examination Papers, Mid-Years 1896-97.

Economics 3.
Year-end Examination,
1896-97]

[Answer the questions in the order in which they stand. Give an hour to each group.]

I.

Discuss the conceptions of progress found in the following authors: Spencer, Comte, Giddings, Kidd, Kelly. Bluntschli. Indicate in each case (a) the nature of progress, (b) the criteria, (c) the chief causes.

What do you mean by progress?

What evidence of progress do you find in the historical sequence of the various forms of political organization which have lead to the modern state? Illustrate carefully.

Name and classify the principal types of political organization. Indicate briefly the social and industrial characteristics of each type.

II.

Give a critical summary of the views of Haycraft in Darwinism and Race Progress.

By what other writers, ancient or modern, have similar views been urged?

What importance do you attach to this school of thought?

What is the practical bearing of such views upon (a) the problems of scientific philanthropy and the treatment of defective and criminal classes? (b Upon socialism?

III.

State and criticise Bluntschli’s theory as to the nature and functions of the State, — the relation of the State to society and to the individual.

Compare Bluntschli’s theory with that of other writers, — Pollock, Spencer, Ritchie, Giddings.

What in your opinion are the merits and the defects of Bluntschli’s treatise?

Source: Harvard University Archives. Harvard University Examination Papers, 1873-1915. Box 4. Bound volume: Examination Papers 1896-97. Papers Set for Final Examinations in Philosophy, History, Government, Economics, Fine Arts, Architecture, and Music in Harvard College, June 1897, p. 39.

_____________________

Economics 14.
Socialism and Communism.

Course Announcement

[Economics] *14. Socialism and Communism. — History and Literature. Tu., Th., (and at the pleasure of the instructor) Sat., at 9. Asst. Professor Edward Cummings.

Source: Harvard University. Announcement of the Courses of Instruction Provided by the Faculty of Arts and Sciences for the Academic Year 1896-1897, p. 34.

Course Enrollment

[Economics] 14. Asst. Professor Edward Cummings.—Communism and Socialism.—History and Literature. 2 hours.

Total 13: 10 Seniors, 2 Juniors, 1 Sophomore.

Source: Harvard University. Report of the President of Harvard College, 1896-97, p. 65.

Economics 14.
Mid-Year Examination,
1896-97

(Arrange your answers in the order of the questions. Omit one.)

  1. The different senses in which the word Socialism is used. Where do you intend to draw the line between Socialism proper, and familiar forms of government interference and control – such as factory legislation, municipal water works, and government postal, telegraph or railroad services? Why?
  2. “National communism has been confused with the common ownership of the family; tenure in common has been confused with ownership in common; agrarian communism with village commons.” Discuss the evidence.
  3. “Just as Plato had his Republic, Campanella his City of the Sun, and Sir Thomas More his Utopia, St. Simon his Industrial System, and Fourier his ideal Phalanstery…. But the common criticism of Socialism has not yet noted the change, and continues to deal with the obsolete Utopias of the pre—evolutionary age.” What do you conceive to be the character of the change referred to? How far did earlier Utopias anticipate the ideals of the modern social democracy?
  4. What indication of Socialistic tendencies are to be found in the discipline of the Christian church? Explain the triple contract and its bearing on the doctrine of the usury.
  5. The contributions of Greek writers to the development of economic thought.
  6. To what extent are the theories of Karl Marx indebted to earlier writers in the 19th-century?
  7. How far are the economic series of (a) Lasalle, (b) Marx related to the theories of the so-called orthodox Economists? Explain critically.
  8. How far do you trace the influence of historical conditions in the social philosophies of Plato, More, Bacon, Rousseau, St. Simon, Karl Marx?

Source: Harvard University Archives. Harvard University Mid-Year Examination, 1852-1943. Box 4. Bound volume: Examination Papers, Mid-Years 1896-97.
Previously transcribed: https://www.irwincollier.com/harvard-exams-and-enrollment-for-economics-of-socialism-and-communism-edward-cummings-1893-1900/

Economics 14.
Year-end Examination,
1896-97

I.

  1. “The figures of Cardinal Manning and Monsignor Bagshawe in England, of De Mun in France, of Decurtins in Switzerland, of Abbé Hitz in Germany, and of Von Vogelsang in Austria, will ever stand apart as State Socialists who, while looking to the Church for moral reform, expected and wished all economic reforms to come from the State alone.”
    Give some account of the leaders mentioned, and discuss the peculiarities of so-called Catholic Socialism in each country.
  2. How far does the programme of “Catholic Socialism” in different countries harmonize with the programme of the German Social Democratic party.
  3. State carefully what has been the attitude of the Vatican towards Socialism? What are the personal views of Leo XIll?

II.

  1. Describe the origin, development, fortunes and present strength of the Social Democratic party in Germany, — with special reference to the Eisenach, Gotha and Erfurt programmes.
  2. State and criticise Marx’s Theory of Surplus Value. Explain carefully the formula

\left( S=P_{n}\times \frac{s\ l\  t}{n\  l\  t} \right)

  1. What are Schäffle’s chief criticisms of the Socialistic State?

III.

[Take one question.]

  1. “Though social conflicts are as old as civilization itself, Socialism as we now understand it is of scientific origin, and essentially modern.” State carefully your reasons for agreeing or disagreeing.
  2. What are the characteristics of modern Utopian ideals, as contrasted with the ideals embodied in earlier literature of the kind? Contrast Bellamy, Hertzka, Morris.
  3. Trace in a general way the influence of socialistic doctrines in the establishment of socialistic and communistic societies in the United States. What light has experiment thrown upon socialistic and communistic ideals?
  4. State accurately the reading you have done in this course during the second half-year. Give a careful summary of the views of the author you recently selected for your special reading.

Source: Harvard University Archives. Harvard University Examination Papers, 1873-1915. Box 4. Bound volume: Examination Papers 1896-97. Papers Set for Final Examinations in Philosophy, History, Government, Economics, Fine Arts, Architecture, and Music in Harvard College, June 1897, p. 49-50.

_____________________

Economics 9.
The Labor Question in Europe and the United States

Course Announcement

[Economics] 9. The Labor Question in Europe and the United States. — The Social and Economic Condition of Workingmen. Tu., Th., Sat., at 10. Asst. Professor Edward Cummings and Dr. John Cummings.

Source: Harvard University. Announcement of the Courses of Instruction Provided by the Faculty of Arts and Sciences for the Academic Year 1896-1897, p. 34.

Course Enrollment

[Economics] 9. Asst. Professor Edward Cummings and Dr. John Cummings. — The Labor Question in Europe and the United States. — The Social and Economic Condition of Workingmen. 3 hours.

Total 50: 5 Graduates, 9 Seniors, 27 Juniors, 7 Sophomores, 2 Others.

Source:   Harvard University, Annual Reports of the President and Treasurer of Harvard College, 1896-97, p. 65.

Economics 9.
Mid-Year Examination,
1896-97

Divide your time equally between A and B.

A.

  1. The labor problem and the unity of the labor movement as manifested in trade unionism, co-operation, and socialism.
  2. How, if at all, has the introduction of machinery directly modified methods of industrial remuneration?
    What are the several bases for determining remuneration?
    Explain the unpopularity, (a) with employers and (b) with employees, of certain methods of industrial remuneration, and, if you can, offer some general principle justifying the adoption of one method rather than another.
    Tell in what way, if at all, the amount of remuneration is affected by the method of remuneration.

B.

Take six.

  1. The “Old” and the “New” Trade-Unionism. Give an account of growth of the English Trade Unions.
  2. What social and economic motives have contributed to the growth of modern cities? Explain so far as you are able the migratory movements which have led to the aggregation of population within certain industrial centres.
  3. Define: “nibbling”; “lump system “; “pay-as-you-please” piece work; the “lump of labor” theory; “chasing”; “collective gain-sharing”; the “plus” system; “butty-gangs”; “tut-work”; “working in pocket”; “garret master”: “product-sharing”; “bribe participation.”
  4. How far are the various forms of profit-sharing “sops to Cerberus”? What is the essential difference between a profit-sharing firm and a co-operative association? How far does industrial co-operation enable workmen to become their own employers?
  5. To what extent is the development of modern machine industry dependent upon the location of the world’s coal fields?
  6. What are the economic and social conditions of industrial depressions?
  7. Are the evils of sweating due to underpay, to overwork, or to the method of remuneration? Are they peculiar to some particular method of remuneration?
  8. The methods of estimating the annual revenue of Great Britain and its distribution.
  9. Has the introduction of machinery lessened the demand for labor?

Source: Harvard University Archives. Harvard University Mid-Year Examination, 1852-1943. Box 4. Bound volume: Examination Papers, Mid-Years 1896-97.

Economics 9.
Year-end Examination,
1896-97

I.

  1. State the subject of your special research during the second half-year. How far were you successful in getting material from (a) newspapers, (b) magazines, (c) books, (d) other sources? What general results did you reach?
  2. Explain the essential difference between a socialistic policy and what Schäffle calls a “positive social policy.” What, according to Schäffle, are legitimate objects of protective legislation? Give some recount of German protective legislation, and the “factory labor” to which this legislation applies.
  3. What according to Mallock, determines the minimum wage in any occupation? How far is this minimum rate subject to legislative or other control? Has the introduction of machinery tended to raise or lower this rate? To increase or to lessen the proportion of wages to product?
  4. What do you understand by the statement that “today the labor contract is perfectly free”? Discuss the legality of (a) strikes; (b) boycots; (c) intimidation; (d) “molesting.”
  5. Describe accurately the German compulsory system of old-age insurance. State the precise arguments which are urged for and against the adoption of some such system in England. How far may the German system be said to be the product of peculiar local conditions?

II.

[Omit two questions.]

  1. Describe some of the chief agencies and methods devised for dealing with the unemployed. How far have they been successful?
  2. Describe the strength, composition and programme of the labor party in Belgium, and its relation to trade-unions, cooperation and socialism. Contrast the situation in Belgium with that in Germany.
  3. Show in what respects the general policy of France towards Associations of workingmen during the present century has differed from that of England. Indicate briefly the effects of this policy upon trade-unions, coöperation, etc.
  4. Discuss, the growth, character, programme and strength of existing labor organizations in the United States, — contrasting the situation in this country with the situation in England and explaining differences.
  5. By what peculiar local conditions in each case do you account for the success or the failure of the coöperative movements in (a) England, (b) France, (c) the United States? What do you predict for the future of coöperation?

Source: Harvard University Archives. Harvard University Examination Papers, 1873-1915. Box 4. Bound volume: Examination Papers 1896-97. Papers Set for Final Examinations in Philosophy, History, Government, Economics, Fine Arts, Architecture, and Music in Harvard College, June 1897, pp. 44-45.

_____________________

Economics 4.
Theory and Methods of Statistics,
1896-97

Course Announcement

[Economics] *4, Theory and Methods of Statistics. — Applications to Economic and Social Questions. —  Studies in the Movement of Population. Mon., Wed., Fri., at 11. Dr. John Cummings.

Source: Harvard University. Announcement of the Courses of Instruction Provided by the Faculty of Arts and Sciences for the Academic Year 1896-1897, p. 34.

_____________________

Course Enrollment

[Economics] 4. Dr. John Cummings. — Theory and Methods of Statistics. — Applications to Economic and Social Questions. — Studies in the Movement of Population. 3 hours.

Total 15: 8 Seniors, 7 Juniors.

Source: Harvard University. Report of the President of Harvard College, 1896-1897, p. 65.

Economics 4.
Mid-Year Examination,
1896-97

[Divide your time equally between A. and B.]

A.

  1. The development of scientific statistics and the statistical method as employed in the social sciences.
  2. Social and economic causes of the migratory movements which have taken place in the populations of Europe and America during this century, and the laws in accordance with which those migrations have taken place where you can formulate any.

B.

(Take five.)

  1. Rural depopulation and the growth of cities in the United States.
  2. Define: “mean after life,” “expectation of life,” “mean duration of life,” “mean age at death.” What relation does the mean age of those living bear to the mean age at death? To the mean duration of life?
  3. Anthropological tests of race vitality as applied to the American negro?
  4. Explain how the economic value of a population is effected by its age and sex distribution.
  5. The United States census: either (1) an historical account of it, or (2) an account of the work now undertaken by the Census Bureau.
  6. Explain the various methods of calculating the birth rate of a population.
  7. How far are social conditions in a community revealed in the birth rate, the death rate, the marriage rate? Of what are fluctuations in these rates evidence in each case?
  8. What do you understand by the “index of mortality”?

Source: Harvard University Archives. Harvard University Mid-year Examinations, 1852-1943. Box 4. Bound volume: Examination Papers, Mid-Years. 1896-97.

Economics 4.
Year-End Examination,
1896-97

I.

  1. Give an historical account of the United States census, and a general statement of the ground covered in the census of 1890; also show how the census taking is supplemented by work done in the Department of Labor and in the statistical bureaus established in connection with the several administrative departments.
  2. Define Körösi’s “rate of natality,” and state any statistical evidence you know that the rate is affected by the standard of living.
  3. “It must, at all times, be a matter of great interest and utility to ascertain the means by which any community has attained to eminence among nations. To inquire into the progress of circumstances which have given pre-eminence to one’s own country would almost seem to be a duty….The task here pointed out has usually been left to be executed by the historian.” Porter: “The Progress of the Nation.”
    What contribution has statistics to make in the execution of this task? What do you understand to be the nature of the statistical method, and what are the legitimate objects of statistical inquiry?

II.

[Take two.]

  1. What light does statistics throw upon the “natural history of the criminal man”?
    Give Ferri’s classification of the “natural causes” of crime, and comment upon that classification. Of criminals.
    What do you understand by “rate of criminality”? By “criminal saturation”?
  2. To what extent in your opinion is suicide an evidence of degeneration in the family stock?
    Discuss the influence upon the rate of suicide of education, religious creed, race, climate and other facts of physical, political and social environment.
  3. Comment critically upon the tables relating to crime in the last five federal censuses taken in the United States.
  4. What difficulties beset a comparative study of criminality in different countries?
  5. How far is it possible to give a quantitative statement to moral and social facts?

III.

[Take one.]

  1. What are some of the more salient facts concerning the movement of population and wealth in the United States, England, and France during the present century, so far as those facts are evidenced in the production, consumption and distribution of wealth?
  2. Discuss the movement of wages and prices in the United States since 1890.
  3. What do you understand by “index figures,” “average wages,” “average prices,” and “weighted averages”?

IV.

[Take one.]

  1. How do you account for the increase in the proportion of urban to rural population during this century? What statistical evidence is there that the increased density of a population affects the mean duration of life? What importance to you attach to this evidence?
    Explain the effect of migratory movements upon the distribution of a population according to age, sex and conjugal condition, and upon the birth rate, death rate and marriage rate.
  2. Define and distinguish: “mean age at death”; “mean duration of life”; “mean age of those living”; “expectation of life.”
  3. The “law of population” as formulated by Malthus and by subsequent writers.

Source: Harvard University Archives. Harvard University, Examination papers, 1873-1915. Box 4, Bound volume: Examination Papers 1896-97. Papers set for Final Examinations in History, Government, Economics, Philosophy, Fine Arts, Architecture, and Music in Harvard College. June 1897, pp. 39-41.
Previously transcribed: https://www.irwincollier.com/harvard-semester-exams-for-statistics-john-cummings-1896-1900/

_____________________

Economics 51.
Railway Transportation
1896-97

Course Announcement

[Economics] 51 hf. Railway Transportation. — Lectures and written work. Half-course. Tu., Th., Sat., at 1.30 (first half-year). Professor Taussig.

Source: Harvard University. Announcement of the Courses of Instruction Provided by the Faculty of Arts and Sciences for the Academic Year 1896-1897, p. 34.

Course Enrollment

[Economics] 51. Mr. Virtue. — Railway Transportation. 3 hours.

Total 62: 2 Graduates, 33 Seniors, 20 Juniors, 5 Sophomores, 2 Others.

Source:   Harvard University, Annual Reports of the President and Treasurer of Harvard College, 1896-97, p. 65.

Economics 51.
Final Examination,
1896-97

[Omit one.]

  1. Sketch the railway history of Italy.
  2. What light does the internal improvement movement in the United States throw upon the question of the public management of the railway industry?
  3. What is the present position of the courts with regard to the power of State legislatures to fix railway rates? On what ground does the public claim the right to interfere in the fixing of such rates.
  4. Discuss the attempt made by any one of the State governments to control railways by means of a commission.
  5. Describe the Hungarian system of passenger tariffs and its effects.
  6. As a basis of rate making, what is the relative importance of the principle of “charging what the traffic will bear,” and that of charging according to the “cost of service”?
  7. It is said of the railway business that “where combination is possible, competition is impossible.” Why is this true? Is it peculiarly true in the railway business? Give your reasons fully.
  8. Discuss the rate policy set forth in the following passages:
    “In the proposed reform of our transportation taxes it will probably be found advisable, at the beginning, to follow the example of Sir Rowland Hill in his reform of the old English postal system, and adopt, as the uniform rate for each class of service for all distances, the lowest rate now charged for the shortest distance for that class of service.”… “If distances of hundreds of miles, can be safely disregarded in the local transportation of milk and potatoes and grain, then surely there is every reason to believe that a general grouping of all the railway stations in the country with a uniform rate will prove to be the best possible system that can be devised for the common good of all.”
  9. A railroad company subject to the “Interstate Commerce Law” charges a much lower rate from New Orleans to San Francisco, for goods which have been imported than for like goods of domestic manufacture. What is the decision of the courts as to the legality of such a charge? What is the economic justification for the decision?
  10. What do you regard as the greatest defects of the “Interstate Commerce Law,” as at present interpreted, and what legislation should you suggest to remedy such defects?
  11. What is the “railway problem”?

Sources: Harvard University Archives. Harvard University Mid-Year Examination, 1852-1943. Box 4. Bound volume: Examination Papers, Mid-Years 1896-97. Copy also in Harvard University Archives. Harvard University, Examination papers, 1873-1915. Box 4, Bound volume: Examination Papers 1896-97. Papers set for Final Examinations in History, Government, Economics, Philosophy, Fine Arts, Architecture, and Music in Harvard College. June 1897, pp. 41-42.

_____________________

Economics 61
History of Tariff Legislation in the United States,
1896-97

Course Announcement

[Economics] 62 [sic] hf. History of Tariff Legislation in the United States. Half-course. Tu., Th., at 2.30, and a third hour at the pleasure of the instructor (second [sic] half-year). Professor Taussig.

Source: Harvard University. Announcement of the Courses of Instruction Provided by the Faculty of Arts and Sciences for the Academic Year 1896-1897, p. 34.

Course Enrollment

[Economics] 61. Professor Taussig. — History of Tariff Legislation in the United States. Hf. 2 hours. 1st half-year.

Total 74: 7 Graduates, 39 Seniors, 20 Juniors, 3 Sophomores, 5 Others.

Source:   Harvard University, Annual Reports of the President and Treasurer of Harvard College, 1896-97, p. 66.

Economics 61.
Final Examination,
1896-97

[Arrange your answers strictly in the order of the questions. Answer all the questions.]

  1. How was the course of commercial history between 1789 and 1810 connected with customs legislation and with the state of opinion on the tariff during the same period?
  2. What were the “abominations” of the tariff act of 1828? What became of them?
  3. Was there similarity between the stages in tariff policy reached by England and by the United States in 1846?
  4. “The ultimate reduction of the price of American to that of the British rolled iron can only, and ultimately will, be accomplished in that western region, which abounds with ore, and in which is found the most extensive formation of bituminous coal that has yet been discovered in any part of the globe.” — Gallatin, in 1832.
    What were the conditions as to the production of iron in the United States when Gallatin wrote? When and how was his prediction fulfilled?
  5. When did the argument appear that protective duties serve to maintain a high rate of wages in the United States? Why at that time? How far do you think it sound?
  6. Why is no carpet wool raised in the United States? Why is no flax cultivated for fibre? Are high wages an obstacle to the production of such commodities?
  7. Was Webster a consistent advocate of free trade in 1824? Gallatin in 1832? Secretary Walker in 1846?
  8. Sketch the main features in the history of duties on wool and woollens from 1846 to 1896.
  9. Wool, pig iron, sugar, — on which would you now remit duties first, and on which last? Why? What are the present duties?

Source: Harvard University Archives. Harvard University Mid-Year Examination, 1852-1943. Box 4. Bound volume: Examination Papers, Mid-Years 1896-97.

Copy also in Harvard University, Examination papers, 1873-1915. Box 4, Bound volume: Examination Papers 1896-97. Papers set for Final Examinations in History, Government, Economics, Philosophy, Fine Arts, Architecture, and Music in Harvard College. June 1897, pp. 42-43.

_____________________

ECONOMICS 81.
History of Financial Legislation in the United States

[81 hf. History of Financial Legislation in the United States. Half-course. Tu., Th., at 2.30, and a third hour at the pleasure of the instructor (first half-year). Professor Dunbar.]

Omitted in 1896-97.

Source: Harvard University. Announcement of the Courses of Instruction Provided by the Faculty of Arts and Sciences for the Academic Year 1896-1897, p. 34.

_____________________

ECONOMICS 162.
Selected Topics in the Financial History of the United States,
1896-97

Course Announcement

[Economics] *161 [sic] hf. Selected Topics in the Financial History of the United States. Half-course. Tu., Th., at 2.30(first half-year [sic]). Professor Dunbar.
Course 16 may be taken as a full course by Graduate Students and by candidates for Honors in Political Science.

Source: Harvard University. Announcement of the Courses of Instruction Provided by the Faculty of Arts and Sciences for the Academic Year 1896-1897, p. 34.

Course Enrollment

[Economics] 62. Professor Dunbar. — Selected Topics in the Financial History of the United States. Hf. 2 hours. 2dhalf-year.

Total 21: 11 Graduates, 6 Seniors, 4 Juniors.

Source:   Harvard University, Annual Reports of the President and Treasurer of Harvard College, 1896-97, p. 66.

Economics 162.
Final Examination,
1896-97

  1. Under the present arrangement of our financial system how far is the original theory of the Independent Treasury Act of 1846 preserved?
  2. What are the considerations for and against the use of banks as government depositaries?
  3. Trace the steps by which the idea that redeemed notes might be reissued became familiar and was finally embodied in the existing law.
  4. How far was the successful resumption in 1879 the result of fortunate circumstances, not to be foreseen in 1875?
  5. What is the legal authority for receiving United States notes in payment of duties on imports?
  6. What would be the probable effect on the Treasury, if Congress were now to adopt a tariff producing ample revenue, without making any change in the currency legislation?
  7. In Secretary Sherman’s Report for 1879 “it is respectfully recommended that by law the resumption-fund be specially defined and set apart for the redemption of United States notes, and that the notes redeemed shall only be issued in exchange for or purchase of coin or bullion.”
    Trace the effect of such a provision in case of large exports of gold. How far would it have served to prevent the difficulties which have actually occurred since 1893?
  8. In his Report for 1889, Secretary Windom recommended the following measure: —
    “Issue Treasury notes against deposits of silver bullion at the market price of silver when deposited, payable on demand in such quantities of silver bullion as will equal in value, at the date of presentation, the number of dollars expressed on the face of the notes at the market price of silver, or in gold, at the option of the Government; or in silver dollars at the option of the holder. Repeal the compulsory feature of the present coinage act.”
    What was this measure expected to accomplish? How would its operation probably have differed from that of the Silver Purchase Act of 1890?

Source: Harvard University Archives. Harvard University Examination Papers, 1873-1915. Box 4. Bound volume: Examination Papers 1896-97. Papers Set for Final Examinations in Philosophy, History, Government, Economics, Fine Arts, Architecture, and Music in Harvard College, June 1897, pp. 51-52.

____________________

Economics 72.
The Theory and Methods of Taxation,
1896-97

Course Announcement

[Economics] *72 hf. The Theory and Methods of Taxation, with special reference to Local Taxation in the United States. Half-course. Mon., Wed., Fri., at 9 (second half-year). Mr. ——.

Source: Harvard University. Announcement of the Courses of Instruction Provided by the Faculty of Arts and Sciences for the Academic Year 1896-1897, p. 35.

Course Enrollment

[Economics] 72. Professor Taussig. — The Theory and Methods of Taxation, with special reference to Local Taxation in the United States. Hf. 3 hours. 2d half-year.

Total 51: 6 Graduates, 17 Seniors, 18 Juniors, 5 Sophomores, 5 Others.

Source:   Harvard University, Annual Reports of the President and Treasurer of Harvard College, 1896-97, p. 66.

Economics 72.
Final Examination,
1896-97

[Arrange your answers strictly in the order of the questions.]

  1. Which among the following would you call a “tax,” and why,—
    (a) an assessment for betterment (e.g. for a sidewalk) on real estate;
    (b) a liquor license;
    (c) the charge for a postage stamp;
    (d) the charge for tobacco in France;
    (e) the charge for a ticket on a Prussian railway.
  1. How far is there separation of local taxes on real estate from state taxes on real estate in France? in Prussia? in England? In American states? Which adjustment seems to you the best, and why?
  2. State the points of resemblance and the points of difference between the system of local taxation in England and the usual method of local taxation in the United States.
  3. Is a tax like the French personelle-mobilière adapted for use in American states? one like the Prussian Business Tax?
  4. Do you think progressive taxation to be sound in principle? Why? or why not?
  5. Point out similarities and differences between the methods of taxing the holders of securities in England and in Pennsylvania.
  6. What do you conceive to be the methods and effects of the taxation of mortgage debts in Massachusetts? in California?
  7. On what grounds would you approve or disapprove of taxes on inheritances and successions, levied by the several American states? of income taxes, similarly levied?
  8. “The statute in Massachusetts, which taxes corporations on their capital stock less the value of real estate and machinery, is indefensible. According to the Massachusetts law, corporations are taxable locally only on their real estate and machinery, while they are taxable for commonwealth purposes only on the value of their capital stock deducting the value of the machinery and the real estate, they are therefore taxed only once on their total property. Individuals, on the other hand, pay not only a general tax for state purposes, but also another general property tax for local purposes. Corporations thus are treated more leniently than individuals.” — Seligman.
    Is this an accurate statement of the legislation in Massachusetts? and are corporations more leniently dealt with than individuals?

One of the following questions may be substituted for any one of the preceding.

  1. What distinctions are made, in the process of assessment under the Prussian income tax, between incomes under 3000 marks and those above?
  2. How are state and local control combined in the assessment of income taxes in Prussia? in England?

Source: Harvard University Archives. Harvard University, Examination papers, 1873-1915. Box 4, Bound volume: Examination Papers 1896-97. Papers set for Final Examinations in History, Government, Economics, Philosophy, Fine Arts, Architecture, and Music in Harvard College. June 1897, pp. 43-44.

_____________________

Economics 71.
Financial Administration and Public Debts

[[Economics] *71 hf. Financial Administration and Public Debts. Half-course. Tu., Th., Sat., at 11 (first half-year). Professor Dunbar.]

Omitted in 1896-97. Courses 71 and 72 are usually given in alternation with Courses 121 and 122.

Source: Harvard University. Announcement of the Courses of Instruction Provided by the Faculty of Arts and Sciences for the Academic Year 1896-1897, p. 35.

_____________________

Economics 121.
Banking and the History of the Leading Banking Systems

Course Announcement

[Economics] *121 hf. Banking and the History of the leading Banking Systems. Half-course. Tu., Th., Sat., at 11 (first half-year). Professor Dunbar.

Source: Harvard University. Announcement of the Courses of Instruction Provided by the Faculty of Arts and Sciences for the Academic Year 1896-1897, p. 35.

Course Enrollment

[Economics] 121. Professor Dunbar. — Banking and the History of the leading Banking Systems. Hf. 3 hours. 1st half-year.

Total 47: 1 Graduate, 18 Seniors, 24 Juniors, 4 Others.

Source:   Harvard University, Annual Reports of the President and Treasurer of Harvard College, 1896-97, p. 66.

Economics 121
Final Examination,
1896-97

  1. If, as McLeod says, all modern banks are banks of issue, how is it that discussions and legislation about banking are chiefly taken up with questions as to bank notes?
  2. When can a note currency be said to be elastic, and what is necessary to give it that quality? Illustrate by actual cases.
  3. Name as many banking systems as you can, which
    (a) protect creditors by a stockholders’ liability of any sort; or
    (b) protect notes by a prior lien on assets.
  4. What advantages are derived from the system of branch banks, and where is it used?
  5. Which of the great banks, — the Bank of England, the Bank of France, the Reichsbank, —  appears to have the best title to be called a government bank?
  6. Suppose that, in a country having well developed banks, a large issue of inconvertible legal tender notes should be made; in what way should you expect its inflating effect upon prices and credits to be produced?
  7. How is the unequal distribution of national banks in the United States, during the years when banking under the national act was most profitable, to be explained?
  8. The following items being given, viz. :
Public Deposits £7.7 Other Securities £30.0
Other Deposits £48.0 Notes in circulation £28.1
Government Securities £20.7 Coin and Bullion £40.4
Government Debts £11.0

construct a Bank of England account, with its separate Departments of Issue and Banking.

  1. Why is it that a comparison of the English country bank circulation with the Scotch shows that one is gaining while the other is dying out?
  2. Under the German bank act what are the two requirements as to holding cash? What is counted as cash under these requirements respectively? Under what conditions does a bank find one or the other of these requirements practically inoperative?
  3. The account of the Bank of France may be simplified as follows:—
Cash, Surplus, and Profits fr. 225. Loans and Investments fr. 1.141.
Sundries fr. 72. Government Securities fr. 353.
Deposits fr. 822. Sundries fr. 75.
Notes fr. 3.612 Cash fr. 3.162.
fr. 4.731. fr. 4.731.

How much of its circulation could the bank pay off and yet earn its present profit? State the account as it would appear if such a change were made?

Source: Harvard University Archives. Harvard University, Examination papers, 1873-1915. Box 4, Bound volume: Examination Papers 1896-97. Papers set for Final Examinations in History, Government, Economics, Philosophy, Fine Arts, Architecture, and Music in Harvard College. June 1897, pp. 47-48.

_____________________

Economics 122.
International Payments and the Flow of the Precious Metals,
1896-97

Course Announcement

[Economics] *122 hf. International Payments and the Flow of the Precious Metals. Half-course. Tu., Th., Sat., at 11(second half-year). Professor Dunbar and Mr. Meyer.
Courses 121 and 122 are usually given in alternation with Courses 71 and 72.

Source: Harvard University. Announcement of the Courses of Instruction Provided by the Faculty of Arts and Sciences for the Academic Year 1896-1897, p. 35.

Course Enrollment

[Economics] 122. Professor Dunbar and Mr. Meyer. — International Payments and the Flow of the Precious Metals. Hf. 3 hours. 2d half-year.

Total 20: 9  Graduates, 2 Seniors, 6 Juniors, 3 Others.

Source:   Harvard University, Annual Reports of the President and Treasurer of Harvard College, 1896-97, p. 66.

Economics 122.
Final Examination,
1896-97

It is expected that one-half of the time of this examination will be required for the questions in division A of this paper.

A.

  1. It is often said that the resumption of specie payments by France and by the United States and the adoption of the gold standard by Germany made nearly simultaneous demands upon the world’s stock of gold. Discuss this statement at length.
  2. The transfer of international securities,
    (a) during the funding operations of the United States, 1872-79;
    (b) as a consequence of the French Indemnity payments.
    Take one of these two cases.
  3. The conditions which led to the flow of gold to the United States,
    (a) in 1896;
    (b) in the fiscal years 1880 and 1881.
    Take one of these two cases.

B.

  1. What were the contributions of Ricardo, Mill and Cairnes, respectively, to the discussion of the laws determining the exchange of commodities between nations?
  2. To what extent are the principles involved in exchanges of goods and services between nations also applicable in domestic exchanges?
  3. Describe the” triangular” operation in exchange between three countries, whereby an export of specie may take place from one of them before the price of exchange has fallen to the shipping point. Illustrate by an actual case.
  4. Explain the difference in rates for long exchange as compared with short, and show the conditions under which an unusual divergence of rates may exist. Illustrate by an actual case.
  5. Why is it that in the dealings between England and other countries bills of exchange are chiefly drawn upon England and few are drawn by her upon others? How are the transactions between England and the United States adjusted, when the bills are for the most part drawn by us upon England?

Source: Harvard University Archives. Harvard University, Examination papers, 1873-1915. Box 4, Bound volume: Examination Papers 1896-97. Papers set for Final Examinations in History, Government, Economics, Philosophy, Fine Arts, Architecture, and Music in Harvard College. June 1897, pp. 48-49.

_____________________

 

Primarily for Graduates.

Economics Seminary Announcement

[Economics] 20. Seminary in Economics. Mon., at 4.30.
Professors Dunbar, Taussig, and Ashley and Asst. Professor Edward Cummings will guide competent students in research on topics assigned after consultation. The Seminary will hold weekly meetings; and in addition each student will confer once a week, with the instructor under whose guidance he carries on his investigations.

Source: Harvard University. Announcement of the Courses of Instruction Provided by the Faculty of Arts and Sciences for the Academic Year 1896-1897, p. 35.

Economics Seminary Enrollment

[Economics] 20. Professors Dunbar, Taussig, and Ashley and Asst. Professor Edward Cummings. — Seminary in Economics.

Total 20: 17 Graduates, 2 Seniors, 1 Other.

Source:   Harvard University, Annual Reports of the President and Treasurer of Harvard College, 1896-97, p. 66.

Image Source: Memorial Hall, Harvard University. From Library of Congress Prints and Photographs Division Washington, D.C.

 

 

 

 

Categories
Exam Questions Harvard

Harvard. Economics and Social Ethics Semester Examinations, 1895-96

 

Professor Charles Dunbar had a leave of absence for the 1895-96 academic year at Harvard. His courses on public finance were taught by Dr. John Cummings and Dr. J.A. Hill. Professor Frank Taussig returned from his year leave of absence for 1894-95 and taught (among other courses) the history of financial institutions, Dunbar’s second field of specialization.

___________________________

1895-96.
The Ethics of the Social Questions.

Course Enrollment for Philosophy 5

For Graduates and Undergraduates:—

[Philosophy] 5 . Professor Peabody. — The Ethics of the Social Questions. — The problems of Poor-Relief, the Family, Temperance, and various phases of the Labor Question in the light of ethical theory. — Lectures, special researches, and required reading. 3 hours.

Total 88: 7 Graduates, 49 Seniors, 12 Juniors, 2 Sophomores, 18 Others.

Source: Harvard University. Report of the President of Harvard College, 1895-96, p. 60.

*  *  *  *  *  *  *  *  *  *

Mid-Year Examination

1895-96.
PHILOSOPHY 5.
THE ETHICS OF THE SOCIAL QUESTIONS.

[This paper should be considered as a whole. The time should not be exhausted in answering a few questions, but such limit should be given to each answer as will permit the answering of all the questions in the time assigned.]

  1. What is meant by;

Exogamy;
Marriage by capture;
The Patriarchal theory;
“The family is the unit of civilization”?

  1. The stability of the family as affected by:

(a) city life.
(b) the conflict of State laws.
(c) the philosophy of individualism.
(d) the philosophy of collectivism.

  1. Spencer’s view of the regime of the family in relation to the regime of the State (Principles of Sociology I, 707 pp.), with criticisms.
  2. The distribution of wealth in Great Britain or in the United States, statistically illustrated; and its lessons,
  3. Illustrate the indirect economic value of judicious charity.
  4. Charles Booth’s Class B in East London; its character, dimensions, relation to the general problem of poverty, and suggested treatment. Life and Labor of the People, I, 39-44; 162-169.)
  5. The new inquiry undertaken by Mr. Charles Booth (Vol. V and VI, 1895); its relation to the preceding researches and its confirmation of earlier results.
  6. “What is good in the poor-administration of Germany is due to good citizenship. … We have not citizenship enough to administer it.” (C. S. Loch, Parliamentary Report of 1888, p. 88.) Compare, in the light of this comment, the English and German theories of municipal relief.
  7. The influx to the great cities in its effect on methods of poor-relief.

Source: Harvard University Archives. Harvard University, Mid-year Examinations 1852-1943. Box 3, Bound volume Examination Papers, Mid-Years 1895-96.

*  *  *  *  *  *  *  *  *  *

Final Examination

1895-96.
PHILOSOPHY 5.
THE ETHICS OF THE SOCIAL QUESTIONS.

[This paper should be considered as a whole. The time should not be exhausted in answering a few questions, but such limit should be given to each answer as will permit the answering of all the questions in the time assigned.]

  1. Explain and illustrate, briefly, what is meant by:

“The social questions are ethical questions.”
“The correlation of the social questions.”

  1. The doctrines of social progress in Carlyle and in Ruskin compared in their bearing on the modern industrial situation.
  2. Consider the principle of social labour-time as the standard of value:

(a) Mr. Ruskin’s theory of value;
(b) The plan proposed by scientific socialism (Schäffle, p. 81);
(c) Schäffle’s criticism of this view (ch. VI., VII.);
(d) Your own judgment.

  1. “Socialism has no necessary affinity with any forms of violence, or confiscation, or class selfishness, or financial arrangement. … The aim of socialism is the fulfilment of service; the aim of individualism is the attainment of some personal advantage, riches, or place, or fame.” — Bishop Westcott.
    “Socialism, as I understand it, is any theory of social organization which sacrifices the legitimate liberties of individuals to the will or interest of the community.” — Professor Flint.
    Which of these definitions appears to you more justified by the history and tendency of socialism? What do you understand to be the “quintessence” of socialism?
  2. The economic and ethical criticisms commonly urged against the programme of collectivism, and your estimate of their importance.
  3. The ethical place and lessons of:

Anarchism;
Communism;
Arbitration.

  1. Compare the plans of industrial unity illustrated by the Anzin collieries, the Val-des-Bois Mill, and the Hebden Bridge Mill.
  2. The coöperative movement in Great Britain, its principles, its expansion, and the conditions of success for the system in this country. In federalistic coöperation what should be, in your judgment, the principle of distributing the bonus?
  3. The polities and the ethics of the Maine liquor law (Fanshawe, VII.)

Source: Harvard University Archives. Harvard University, Examination papers 1873-1915. Box 4, Bound volume: Examination Papers 1896-97. Papers Set for Final Examinations in Philosophy, History, Government, Economics, Fine Arts, Architecture, and Music in Harvard College, June 1896.

___________________________

1895-96.
Outlines of Economics.

Course Enrollment for Economics 1

Primarily for Undergraduates:—

[Economics] 1. Professors Taussig and Ashley, Asst. Professor Edward Cummings, and Dr. John Cummings. — Outlines of Economics. — Mill’s Principles of Political Economy. — Lectures on Economic Development, Distribution, Social Questions, and Financial Legislation. 3 hours.

Total 338: 3 Graduates, 35 Seniors, 91 Juniors, 161 Sophomores, 8 Freshmen, 40 Others.

Source: Harvard University. Report of the President of Harvard College, 1895-96, p. 63.

*  *  *  *  *  *  *  *  *  *

Mid-Year Examination

1895-96.
ECONOMICS 1.

  1. Is all wealth produced by labor?
  2. Compare the distinction between fixed and circulating capital with the distinction between auxiliary and remuneratory capital; and state why one or the other distinction is the more satisfactory.
  3. Are differences in profits from employment to employment similar in kind to differences in wages from occupation to occupation?
  4. In what way are differences of wages affected by the absence of effective competition between laborers? by its presence?
  5. What are the grounds for saying that rent is a return differing in kind from interest?
  6. Trace the effects of an issue of inconvertible paper money, less in quantity than the specie previously in use, on (1) the circulation of specie, (2) the foreign exchanges, (3) the relations of debtor to creditor.
  7. State Mill’s reasoning as to the mode in which, under a double standard, one metal is driven from circulation; and explain how the actual process differs from that analyzed by Mill.
  8. What are the grounds for saying that the gain of international trade does not come from the sale of surplus produce beyond the domestic demand?
  9. In what manner is the price of landed property affected by an increased quantity of money? by a rise in the rate of interest?
  10. Wherein does monopoly value present a case different from that of the usual operation of the laws of value?

Source: Harvard University Archives. Harvard University, Mid-year Examinations 1852-1943. Box 3, Bound volume Examination Papers, Mid-Years 1895-96.

*  *  *  *  *  *  *  *  *  *

Final Examination

1895-96.
ECONOMICS 1.

[Answer ten questions. Arrange your answers strictly in the order of the questions.]

GROUP I.
[At least one.]

  1. Explain the meaning of two of the following terms, — margin of cultivation; wages of superintendence; rapidity of circulation (as to money).
  2. Do profits constitute a return different from interest?
  3. Explain what is meant by the law, or equation, of demand and supply; and in what manner it applies to commodities susceptible of indefinite multiplication without increase of cost.
  4. In what manner does a country gain from the division of labor in its domestic trade? In what manner from international trade?

GROUP II.
[At least one.]

  1. Does it fall within the province of the economist to discuss the institution of private property?
  2. Show the connection between the industrial development of the present century, and the discussion among economists as to the functions of the entrepreneur.
  3. Consider in what manner prices, or rents, [choose one] are differently determined according as they are under the influence of custom or of competition.
  4. “The idea that economic life has ever been a progress mainly dependent on individual action is mistaken with regard to all stages of civilization, and in some respects it is the more mistaken the farther we go back.” Explain and criticize.

GROUP III.
[At least one.]

  1. If coöperation were universally adopted, what would be left of the wages system?
  2. Is there anything in what you learned as to the laws governing wages, which the action of the English trade-unions in regard to wages has disregarded?
  3. Has the course of events justified Mill’s expectations in regard to the development of profit-sharing and of cooperation? Explain why, or why not.
  4. Describe the trade and benefit features of the English trade-unions.

GROUP IV.
[At least three.]

  1. Is the present position of the Treasury of the United States in any respect essentially similar to that of the Issue Department of the Bank of England? In any respect essentially dissimilar?
  2. What is the test of over-issue, as to inconvertible paper money? What light does the experience of the United States and of France throw on the probability of over-issue?
  3. Arrange in their proper order the following items in a bank account:
Capital 100,000 Bonds and Stocks 75,000
Specie 150,000 Surplus 50,000
Notes 100,000 Other Assets 50,000
Loans 400,000 Other Liabilities 60,000
Expenses 25,000 Undivided Profits 40,000
Deposits 350,000

Could this bank be a national bank of the United States? If such a bank, how would the account stand?

  1. Compare the policy of the Bank of England in times of financial crisis with the policy of the Associated Banks of New York; and give an opinion as to which is the more effective in allaying panic.

Source: Harvard University Archives. Harvard University, Examination papers 1873-1915. Box 4, Bound volume: Examination Papers 1896-97. Papers Set for Final Examinations in Philosophy, History, Government, Economics, Fine Arts, Architecture, and Music in Harvard College, June 1896.

___________________________

1895-96.
Economic Theory from Adam Smith to the present time.

Course Enrollment for Economics 2.

For Graduates and Undergraduates:—

[Economics] 2. Professors Ashley and Macvane. — Economic Theory from Adam Smith to the present time. — Selections from Adam Smith and Ricardo. — Modern Writers. —Lectures. 3 hours.

Total 37: 5 Graduates, 14 Seniors, 7 Juniors, 4 Sophomores, 7 Others.

Source: Harvard University. Report of the President of Harvard College, 1895-96, p. 63.

*  *  *  *  *  *  *  *  *  *

Mid-Year Examination.

1895-96.
ECONOMICS 2

N.B. — Not more than seven questions must be attempted.

  1. Compare the Aristotelian conception of Wealth with that of modern economists.
  2. Explain the growth, in the later Middle Ages, of the theory of “Interest.”
  3. Consider briefly the claims to consideration, in the history of economic thought, of Nicholas Oresme and Antoine de Montchrétien.
  4. “It was reserved for the eighteenth century to let in the grand idea of necessity, and to prove that the rate of wages established in a country was the inevitable consequence of the circumstances in which that country was placed, and had no connection with the wishes of any individual, or, indeed, with the wishes of any class.” (Buckle, History of Civilization.) Consider this.
  5. Explain the “plan” of the Wealth of Nations, and consider how far it agrees with the contents of the work.
  6. State and discuss Adam Smith’s doctrine of the Component Parts of Price.
  7. “A man must always live in his work.” Discuss the accuracy of this proposition, and the use made of it by Adam Smith and later economists.
  8. The effect upon English economists in the seventeenth and eighteenth centuries of their observation of the United Netherlands.
  9. “Every candid reader knows that Mr. Malthus laid no stress on his unlucky attempt to give numerical precision to things which do not admit of it, and every person capable of reasoning must see that it is wholly superfluous to his argument.” (Mill). Consider this.
  10. With what justice can socialists claim the authority of Ricardo for their “iron law of wages”?

Source: Harvard University Archives. Harvard University, Mid-year Examinations 1852-1943. Box 3, Bound volume Examination Papers, Mid-Years 1895-96.

*  *  *  *  *  *  *  *  *  *

Final Examination

1895-96.
ECONOMICS 2.

Take any three of the five questions.

  1. State your conclusions regarding the various definitions of Cost of Production. Are wages an element in Cost? Show whether economic cost and commercial (or employers’) cost may vary independently of each other.
  2. State briefly the views of Henry George, Marshall, and Boehm-Bawerk (or any other three writers) regarding the law of Interest. Give also your own conclusions.
  3. Set down carefully your conclusions as to the source and the law of Wages. Examine at least one opposing view.
  4. Explain and examine the Marginal Utility theory of Value. How is it reconciled with the observed connection between value and cost?
  5. Is a high level of wages in a country an obstacle to foreign trade?

Source: Harvard University Archives. Harvard University, Examination papers 1873-1915. Box 4, Bound volume: Examination Papers 1896-97. Papers Set for Final Examinations in Philosophy, History, Government, Economics, Fine Arts, Architecture, and Music in Harvard College, June 1896.

___________________________

1895-96.
The Principles of Sociology.

Course Enrollment for Economics 3.

For Graduates and Undergraduates:—

[Economics] 3. Asst. Professor Edward Cummings. — The Principles of Sociology. — Development of the Modern State, and of its Social Functions. 2 hours.

Total 37: 8 Graduates, 21 Seniors, 6 Juniors, 2 Others.

Source: Harvard University. Report of the President of Harvard College, 1895-96, p. 63.

*  *  *  *  *  *  *  *  *  *

Mid-Year Examination.

1895-96.
ECONOMICS 3.

(Arrange your answers in the order of your questions. Omit one.)

  1. “Hence, in this case we may assert clearly that when the individual is removed the social ceases to be, and that there is absolutely nothing in society which does not exist in a state of subdivision and continual repetition in the living individuals, — or which has not existed in the dead ancestors from whom the living proceed.” Explain carefully. Compare this conception of society with the “social organism” conception, and state clearly your own views.
  2. What do you conceive to have been the habits and characteristics of primitive man in “a state of nature”? Discuss the evidence presented by Westermarck, Spencer, and others.
  3. “In a word, the physiological bond, which of old constituted the main foundation of the small domestic societies, then of the tribes, then of the ancient cities, is still the essential foundation of the great nations of today.” Explain carefully. What according to Spencer, have been the merits and defects of the various forms of family organization? What are the present tendencies?
  4. “Entangled and confused with one another as Ceremonial and Fashion are, they have thus different origins and meanings.” Explain. Trace carefully the significance of these differences, and give examples.
  5. “Class distinctions, then, date back to the beginnings of social life.”
  6. In what order have political institutions evolved? What have been the chief determining factors?
  7. “M. Alfred Fouillée has endeavored to express the truth of both ways of regarding society by saying that the highest form of it must be an ‘organism contractuel,’ — a formula that may perhaps gain more general acceptance than anything expressed in the phraseology of German idealism.” Explain carefully.
  8. Discuss the views of Spencer and of Comte in regard to the scope of sociology and its relation to other sciences

Source: Harvard University Archives. Harvard University, Mid-year Examinations 1852-1943. Box 3, Bound volume Examination Papers, Mid-Years 1895-96.

*  *  *  *  *  *  *  *  *  *

Final Examination

1895-96.
ECONOMICS 3.

Answer the questions in the order in which they stand. Give one hour to each division.

I.

A critical estimate of Giddings’ Principles of Sociology, — contrasted (a) with Spencer, (b) with Tarde.

II.

A critical estimate of Evolution and Effort, — contrasting it with views set forth in Social Evolution.

III.

The bearing of sociological theory upon the practical problems of (a) poverty, (b) pauperism, (c) crime.
Which of the books read during this half-year (and not already discussed) has seemed to you of greatest worth? Why?

Source: Harvard University Archives. Harvard University, Examination papers 1873-1915. Box 4, Bound volume: Examination Papers 1896-97. Papers Set for Final Examinations in Philosophy, History, Government, Economics, Fine Arts, Architecture, and Music in Harvard College, June 1896.

___________________________

1895-96.
The Theory of Statistics.

Course Enrollment for Economics 42.

For Graduates and Undergraduates:—

[Economics] 42. Dr. John Cummings. — The Theory of Statistics. — Applications to Social and Economic Problems. — Studies in movements of Population. Hf. 3 hours. 2d half year.

Total 19: 2 Graduates, 11 Seniors, 4 Juniors, 2 Sophomores.

Source: Harvard University. Report of the President of Harvard College, 1895-96, p. 64.

*  *  *  *  *  *  *  *  *  *

Final Examination

1895-96.
ECONOMICS 4.

(Divide your time equally between A and B.)

A
I and II may be treated as one question.

  1. What do you understand by “movement of population”? What light do Statistics throw upon the law of population as stated by Malthus?
  2. What are some of the “more striking facts and more pregnant results of the vast growth of population in Europe, America, and the British Colonies within the last half century”?

B.
Take five.

  1. In constructing a life table what correction must be made for abnormal age and sex distribution of the population?
  2. Define the following terms: “Mortality,” “Expectation of Life,” “Mean Duration of Life.” How should you calculate the mean duration of life from the census returns for any community?
  3. How should you calculate the economic value of a population?
  4. What are some of the inaccuracies to which censes enumerations are liable?
  5. What is the nature of a statistical law? of what categories of social phenomena may statistical laws be formulated? in what sense are they laws? How do they bear upon freedom of the will in human conduct?
  6. How do the conditions of observation in social sciences differ from conditions of observation in the natural sciences?
  7. What do you understand by the law of criminal saturation?
  8. By what considerations should the Statistician be guided in in making selection of social phenomena for investigation?

Source: Harvard University Archives. Harvard University, Examination papers 1873-1915. Box 4, Bound volume: Examination Papers 1896-97. Papers Set for Final Examinations in Philosophy, History, Government, Economics, Fine Arts, Architecture, and Music in Harvard College, June 1896.

___________________________

1895-96.
Railway Transportation.

Course Enrollment for Economics 51.

For Graduates and Undergraduates:—

[Economics] 51. Professor Taussig. — Railway Transportation. — Lectures and written work. Hf. 3 hours. 1st half year.

Total 43: 6 Graduates, 27 Seniors, 7 Juniors, 3 Law.

Source: Harvard University. Report of the President of Harvard College, 1895-96, p. 64.

 

*  *  *  *  *  *  *  *  *  *

Final Examination

1895-96.
ECONOMICS 5.

  1. The means of transportation in the United States in 1855.
  2. Is there historical warrant for the assertion that in United States the construction and operation of railways have been left mainly to private enterprise?
  3. The resemblances and differences between the legislation of Iowa on maximum rates, and that of England.
  4. Are there good grounds for alarm at the tendency to consolidation and the growth of great systems among railways?
  5. “There was never a more mistaken idea than the idea that rates would be reduced if they were based on cost of service. The principle keeps rates up. If it is strictly applied, it makes it necessary that each item of business should pay its share of fixed charges.” Why? or why not?
  6. “It is not true that when the price falls below cost of production, people always find it for their interest to refuse to produce at a disadvantage. It very often involves worse loss to stop producing than to produce below cost.” Why and how, as to railways?
  7. The provisions of the Interstate Commerce Act which bear on

an agreement to maintain certain rates;
an agreement to divide earnings;
a lower rate for one hundred carloads than for one carload;
a postage-stamp rate;
a higher rate for a shorter than for a longer distance.

  1. Does the history of pooling arrangements in the United States justify the assertion that they tend to remove inequalities in the rates to shippers?
  2. The lessons of public railway management in Italy and in France.
  3. The evidence as to the financial and economic success of public railway management in Prussia.

Source: Harvard University Archives. Harvard University, Mid-year Examinations 1852-1943. Box 3, Bound volume Examination Papers, Mid-Years 1895-96.

Also found in: Harvard University Archives. Harvard University, Examination papers 1873-1915. Box 4, Bound volume: Examination Papers 1896-97. Papers Set for Final Examinations in Philosophy, History, Government, Economics, Fine Arts, Architecture, and Music in Harvard College, June 1896.

___________________________

1895-96.
History of Tariff Legislation in the United States.

Course Enrollment for Economics 61.

For Graduates and Undergraduates:—

[Economics] 61. Professor Taussig. — History of Tariff Legislation in the United States. Hf. 2 hours. 1st half year.

Total 88: 11 Graduates, 40 Seniors, 20 Juniors, 5 Sophomores, 12 Others.

Source: Harvard University. Report of the President of Harvard College, 1895-96, p. 64.

*  *  *  *  *  *  *  *  *  *

Final Examination

1895-96.
ECONOMICS 6.

Arrange your answers strictly in the order of the questions. One question, and one only, may be omitted.]

  1. What earlier legislation affected the provisions of the tariff act of 1789? What light does the earlier legislation throw on the character of this act?
  2. Was the argument for protection to young industries more applicable to cotton goods in 1816 than to silk goods in 1870?
  3. What changes were made, in 1833, in the duties on woollens, cottons, linens, and worsteds? Why the differences in policy?
  4. What were the grounds on which it was maintained, in 1828-32, that a tax on imports was virtually a tax on exports? How far was the assertion true?
  5. Mention points of similarity and points of difference between Webster’s speech of 1824 and Gallatin’s memorial of 1831.
  6. Should you say that the position of the protective system in public opinion was the same in 1870-90 as in 1816-32?
  7. Explain the legislation in regard to the duties on sugar in the acts of 1890 and 1894. Was the at of 1894 more advantageous than its predecessor to the planters? to the refiners? to the public?
  8. What do you believe would now be the effect, on domestic industries, of the free admission of (1) pig iron, (2) woollen goods, (3) linens?
  9. In what mode were the tea and coffee duties dealt with in the period 1840-60? in the period 1865-95? What explanation of the general course of policy can you give in either case?
  10. In what cases, if in any, are duties on imports a charge on the foreign producer?
  11. The significance of the events of 1860 for the tariff history of France and of England.
  12. Is there ground for saying that the drift since 1870 toward protective duties, in the United States and on the Continent of Europe, rests on the same general causes?

Source: Harvard University Archives. Harvard University, Mid-year Examinations 1852-1943. Box 3, Bound volume Examination Papers, Mid-Years 1895-96.

Also found in: Harvard University Archives. Harvard University, Examination papers 1873-1915. Box 4, Bound volume: Examination Papers 1896-97. Papers Set for Final Examinations in Philosophy, History, Government, Economics, Fine Arts, Architecture, and Music in Harvard College, June 1896.

___________________________

1895-96.
Financial Administration and Public Debts.

Course Enrollment for Economics 71.

For Graduates and Undergraduates:—

[Economics] 71. Dr. John Cummings. — Financial Administration and Public Debts. Hf. 3 hours. 1st half year.

Total 27: 1 Graduate, 8 Seniors, 12 Juniors, 1 Sophomore, 5 Others.

Source: Harvard University. Report of the President of Harvard College, 1895-96, p. 64.

*  *  *  *  *  *  *  *  *  *

Final Examination

1895-96.
ECONOMICS 7.

Divide your time equally between A and B. I and II may be treated as one question.

A.

  1. Give an account of the sinking fund provisions enacted by Congress 1790-1820; and of the management and refunding of the debt during this period.
  2. Examine and criticise the following account of the evolution of public credit, with a view to determining whether a government is ever justified in pledging the State to any definite policy of debt payment:—

“In this evolution, as in all others, there are transition stages: we have debts of long term, but secured by the pledging of public property or of income from taxes. Then we have a long period of redemption without such a pledge. The plan of discharging the debt simply on the ground of financial expediency, to which the debtor state has accustomed itself, presently takes the place of redemption simply at the instance of impatient creditors. Finally the question of redemption comes by mutual consent to be left entirely undetermined.”

B.

(Take any five of the questions following.)

  1. What effect upon the present worth of a security has lengthening the term for which it is to run?
  2. Give an account of the payment of the war indemnity to Germany.
  3. Discuss the “use and disuse of ‘relishes,’ gambling risks which are added in order to commend a public loan to the taste of creditors,” as a feature in the development of public credit.
  4. Compare the development of public credit in Prussia with that of Great Britain, at the beginning of this century.
  5. Examine and criticise the following selection:—

“As regards the relation of public control to the public credit, there is obviously a lone step taken in advance when the public control comes to he so employed as to not discriminate in its own favor.”

  1. [sic] Define the following terms, and illustrate: “budget,” “conversion,” “rolling annuity.”
  1. [sic] What influence has our Secretary of the Treasury over financial legislation, as compared with the influence of the English Chancellor of the Exchequer? Compare the manner of making up the estimates of public income and expenditure in England and in the United States; of appropriating funds out of the Treasury.

Source: Harvard University Archives. Harvard University, Mid-year Examinations 1852-1943. Box 3, Bound volume Examination Papers, Mid-Years 1895-96.

Also found in: Harvard University Archives. Harvard University, Examination papers 1873-1915. Box 4, Bound volume: Examination Papers 1896-97. Papers Set for Final Examinations in Philosophy, History, Government, Economics, Fine Arts, Architecture, and Music in Harvard College, June 1896.

___________________________

1895-96.
History of Financial Legislation in the United States.

Course Enrollment for Economics 82.

For Graduates and Undergraduates:—

[Economics] 82. Dr. J.A. Hill. — History of Financial Legislation in the United States. Hf. 2 hours. 2d half year.

Total 64: 5 Graduates, 22 Seniors, 18 Juniors, 6 Sophomores, 13 Others.

Source: Harvard University. Report of the President of Harvard College, 1895-96, p. 64.

 

*  *  *  *  *  *  *  *  *  *

Final Examination

1895-96.
ECONOMICS 8.

(N. B. — Omit one question under each of the five main divisions of the paper.)

I.

  1. Is the Independent Treasury System preferable to the use of banks as public depositories? Present the arguments on each side of the question, using any illustrations from the History of the United States that may occur to you.
  2. What illustrations does our financial history afford of difficulties that may arise from an exclusive reliance upon import duties as a source of revenue?

II.

  1. Describe the scheme which was adopted in 1790 for settling the accounts between the United States and the individual States. How did the assumption of the State debts affect the account?
  2. In what respects did the financial policy which the country pursued during the War of 1812 deviate from that which Gallatin had advocated in anticipation of war?
  3. What descriptions of treasury notes were issued during the War of 1812, and how did the successive issues indicate that the country was drifting towards a government paper currency?

III.

  1. The following extract is from a speech which Webster delivered in Congress on Jan. 2, 1815. The bank bill to which it refers was substantially the same as Dallas’ plan for a bank:—
    “What sort of an institution, sir, is this? It looks less like a bank than a department of the Government. It will be properly the paper money department. Its capital is Government debts; the amount of its issues will depend on Government necessities; Government, in effect, absolves itself from its own debts to the bank, and by way of compensation absolves the bank from its own contracts with others.”
    What features of the proposed bank did Webster refer to in his criticisms? What sort of a bank did he favor? What was the outcome of the movement for a bank at this session of Congress?
  2. What causes produced the surplus of 1836? When was there a somewhat similar situation in the later history of the country?
  3. State briefly where the public moneys of the United States have been kept at different periods since 1789.

IV.

  1. How did Secretary Chase execute the authority conferred upon him by the loan Acts of July 17 and Aug. 5, 1861, and in what respect was the course which he pursued open to criticism?
  2. The Legal Tender Act of March 3, 1863, contains the following clauses:—

And so much of the Act to authorize the issue of United States notes, and for other purposes, approved Feb. 25, 1862, and of the act to authorize an additional issue of United States notes, and for other purposes, approved July 11, 1862, as restricts the negotiation of bonds to market value, is hereby repealed. And the holders of United States notes, issued under and by virtue of said acts, shall present the same for the purpose of exchanging the same for bonds, as therein provided, on or before the first day of July, 1863, and thereafter the right so to exchange the same shall cease and determine.
Explain the meaning, object and effect of these provisions.

  1. How much assistance did the Government derive from the Direct Tax during the Civil War? Why is it probable that this form of taxation will never be resorted to again?

V.

  1. Give the main provisions of the Resumption Act of 1875? Why was it doubtful whether this Act would actually secure the resumption of special payments?
  2. State in general terms the changes effected in the form of the national debt (1) while McCulloch was Secretary of the Treasury, (2) under the Refunding Act of 1870, (3) by Secretary Windom in 1881.
  3. Give an account of the discussion which arose in 1867-68 on the question of paying the principal of the War debt in legal tender notes.

Source: Harvard University Archives. Harvard University, Examination papers 1873-1915. Box 4, Bound volume: Examination Papers 1896-97. Papers Set for Final Examinations in Philosophy, History, Government, Economics, Fine Arts, Architecture, and Music in Harvard College, June 1896.

___________________________

1895-96.
The Social and Economic Condition of Workingmen in the United States and in other countries.

Course Enrollment for Economics 9.

For Graduates and Undergraduates:—

[Economics] 9. Asst. Professor Edward Cummings. — The Social and Economic Condition of Workingmen in the United States and in other countries. 3 hours.

Total 67: 4 Graduates, 25 Seniors, 27 Juniors, 6 Sophomores, 5 Others.

Source: Harvard University. Report of the President of Harvard College, 1895-96, p. 64.

*  *  *  *  *  *  *  *  *  *

Mid-Year Examination.

1895-96.
ECONOMICS 9

(Arrange your answers in the order in which the questions stand. So far as possible illustrate your discussions by a comparison of different countries. Omit one question.)

  1. Contrast the structure of industry before machinery with the structure of modern industry.
  2. In what sense can there be said to be a law of invention? and how is this illustrated historically by the appearance and sequence of the great industrial inventions?
  3. How does machinery affect the demand for labor? the quality of labor? the family of the laborer? his real wage?
  4. Trade unionism vs. trades unionism; the old unionism vs. the new unionism. Explain the differences, and show how and when these phases have from time to time recurred during this century.
  5. How is Chartism related to other phases of the labor movement in England?
  6. The merits and the demerits of such trade-union organizations as you have thus far become acquainted with.
  7. Arbitration and Conciliation: (a) In what industries and in what forms have they succeeded best? (b) The present status and the prospects of industrial arbitration in England and in the United States.
  8. Taking the ordinary factory, how far is it possible or impossible to devise a system of remuneration which reconciles the interests of (a) workmen, (b) foremen, (c) employers, and (d) consumers? Explain carefully the merits and defects of the methods you propose to adopt or reject.
  9. In what respects does labor differ from other commodities? What ethical and economic consequences flow from these differences?
  10. How far, from time to time, has economic theory — Smith, Ricardo, Malthus, Mill, etc., — seemed to justify, and how far to suggest remedies for the industrial evils affecting wage-earners?
  11. The relation of cooperation to trade-unions, to profit-sharing, to socialism.

Source: Harvard University Archives. Harvard University, Mid-year Examinations 1852-1943. Box 3, Bound volume Examination Papers, Mid-Years 1895-96.

*  *  *  *  *  *  *  *  *  *

Final Examination

1895-96.
ECONOMICS 9.

(Arrange your answers in the order in which the questions stand. So far as possible illustrate your discussions by a comparison of different countries. Take the first six questions and one other.)

  1. Describe carefully the German system of compulsory insurance:

(a) To whom and to what proportion of the population it applies;
(b) The method of organization and of assessment in each case;
(c) The relation of the system to employer’s liability, to poor laws, friendly societies, etc.
(d) Arguments for and against the system.

  1. How far and with what modifications have such schemes been adopted or seriously proposed elsewhere?

(a) Contrast the plan in each case with the German plan;
(b) What circumstances seem to you to favor and what to hinder such action by the government?

  1. How far have voluntary organizations solved or failed to solve the problem of workingmen’s insurance, (a) in England? (b) in the United States?
  2. What light does the experience of France and of England during this century throw upon the good or the bad effect of attempts on the part of the government either to repress or to foster, (a) labor organizations; (b) coöperation; (c) friendly societies?
  3. In what other countries have you found instructive examples of such interference?
  4. Compare the experience and the legislation of the United States in regard to immigration, with the experience and legislation of other countries in which immigration problems have arisen.
  5. In what countries and in what ways have labor organization tended to drift into politics, and seek political remedies for industrial evils?

(a) Compare the experience of France, Belgium, Germany and English-speaking countries.
(b) What conclusion do you draw from such experience?

  1. What evidence do statistics of family income and expenditure furnish (a) in regard to the social condition of labor in staple industries of the United States and of competing countries? (b) in regard to cost of labor?
  2. What attempts have been made to perpetuate or reestablish certain aspects of the guild organization in European countries?
  3. Discuss the schemes adopted by governments, municipalities, etc., for meeting the “out-of-work problem.”
    What is the origin of that problem in the United States?

Source: Harvard University Archives. Harvard University, Examination papers 1873-1915. Box 4, Bound volume: Examination Papers 1896-97. Papers Set for Final Examinations in Philosophy, History, Government, Economics, Fine Arts, Architecture, and Music in Harvard College, June 1896.

___________________________

1895-96.
The Mediaeval Economic History of Europe.

Course Enrollment for Economics 10.

For Graduates and Undergraduates:—

[Economics] 10. Professor Ashley. — The Mediaeval Economic History of Europe. 2 hours.

Total 14: 7 Graduates, 5 Seniors, 2 Juniors.

Source: Harvard University. Report of the President of Harvard College, 1895-96, p. 63.

*  *  *  *  *  *  *  *  *  *

Mid-Year Examination.

1895-96.
ECONOMICS 10.

I.

To be first attempted by all.

Translate, and comment, on the following passages:

  1. Totius terrae descriptio diligens facta est, tam in nemoribus quam in pascuis et pratis, nec non in agriculturis, et verbis communibus annotata in librum redacta est.
  2. In Tineguella . . . sunt iiii hidae et dimidia ad geldum Regis. Et de istis tenent xx homines xx virgas terrae. Et xiii homines tenent vi virgas et dimidiam.
  3. Sicut traditum habemus a patribus, in primitivo regni statu post conquisitionem, regibus de fundis suis non auri vel argenti pondera sed sola victualia solvebantur.
  4. Plerique, cum aut aere alieno aut magnitudine tributorum aut injuria potentiorum premuntur, sese in servitutem dicunt nobilibus, quibus in hos eadem omnia sunt jura quae dominis in servos.
  5. Ceteris servis non in nostrum morem, descriptis per familiam ministeriis, utuntur. Suam quisque sedem, suos penates regit.

II.

Write on four only of the following subjects.

  1. The importance of the yardland in the rural economy of the Middle Ages.
  2. A history of the mark theory, from its first promulgation to its general acceptance.
  3. A comparison of the life of a medieval English village with that of a New England village of today.
  4. The Roman colonate.
  5. An account and criticism of Mr. Seebohm’s “Tribal System in Wales.”

Source: Harvard University Archives. Harvard University, Mid-year Examinations 1852-1943. Box 3, Bound volume Examination Papers, Mid-Years 1895-96.

*  *  *  *  *  *  *  *  *  *

Final Examination

1895-96.
ECONOMICS 10.

I.

To be first attempted by all.

Comment on the following passages, and translate those in Latin and French:—

  1. If a man agree for a yard of land, or more, at a fixed rent, and plough it; if the lord desire to raise the land to him to service and to rent, he need not take it upon him, if the lord do not give him a dwelling.
  2. Ego Eadward . . . rex . . . dedi X manentes in illo loco qui dicitur aet Stoce be Hysseburnam, cum omnibus hominibus qui in illa terra erant qando Ælfred rex viam universeæ carnis adiit.
  3. Magnates regni et alii minores domini qui tenentes habebant perdonarunt redditum de redditu ne tenentes abirent prae defectu servorum et caristia rerum.
  4. Whan Adam dalf and Eve span,
    Wo was thanne a gentilman?
  5. Nul ne deit rien achater a revendre en la vile meyme, fors yl sera Gildeyn.
  6. Cives Londoniae debent LX marcas pro Gilda telaria delenda ita ut de cetero non suscitetur.
  7. No one of the trade of Spurriers shall work longer than from the beginning of the day until curfew rings out at the church of St. Sepulchre.

II.

Write on four only of the following subjects:

  1. The economic and constitutional questions involved in recent discussions as to the beginnings of town life in mediaeval Europe.
  2. A comparison of a mediaeval merchant gild with a modern “trust,” and of a craft gild with a modern trade union.
  3. The extent and character of the public regulation of prices and wages in the later middle ages.
  4. The cause of the Peasant Revolt in 1381.
  5. The relation of the English Reformation to the origin of the Poor Laws.
  6. A criticism of Cunningham and McArthur’s Outlines of English Industrial History.

Source: Harvard University Archives. Harvard University, Examination papers 1873-1915. Box 4, Bound volume: Examination Papers 1896-97. Papers Set for Final Examinations in Philosophy, History, Government, Economics, Fine Arts, Architecture, and Music in Harvard College, June 1896.

___________________________

1895-96.
Banking and the History of the leading Banking Systems.

Course Enrollment for Economics 122.

For Graduates and Undergraduates:—

[Economics] 122. Professor Taussig. — Banking and the History of the leading Banking Systems. Hf. 3 hours. 2d half year.

Total 70: 10 Graduates, 30 Seniors, 19 Juniors, 4 Sophomores, 7 Others.

Source: Harvard University. Report of the President of Harvard College, 1895-96, p. 64.

*  *  *  *  *  *  *  *  *  *

Final Examination

1895-96.
ECONOMICS 12.

[Arrange your answers strictly in the order of the questions. Give some answer, however brief, to each question.]

  1. What was Bagehot’s opinion as to the advantage of a “many reserve” system as compared with a “single reserve” system? What light does American experience give?
  2. What important proposal made by Bagehot in Lombard Street has been adopted?
  3. What was the theory of the act of 1844? How far was that theory followed in the legislation on the Reichsbank of Germany?
  4. (a) Arrange in their proper order the following items, in which the figures stand for millions of marks.
Capital 150 Loans 800
Specie 800 Securities 50
Notes 1150 Other Assets 50
Deposits 350 Other Liabilities 50

(b) Consider what would be the significance of the statement if it were for the Reichsbank of Germany; assuming the limit of uncovered issue to be 300 millions of marks.

(c) Rearrange the items as they would appear if the statement were one of the condition of the Bank of England; assuming the limit of notes not required to be covered by specie to be 16 millions sterling = 400 million marks, and assuming that securities of any sort may be held against the uncovered issue. Consider then how the statement, thus rearranged, differs from a probable statement of the actual condition of the Bank of England in recent times.

  1. Does the Bank of France supply an elastic currency? Do the National Banks of the United States?
  2. “Redemption by the Treasury under the national bank legislation has been a convenient method of disposing of worn and soiled notes, and in case of accumulations of currency at special points has facilitated its rapid exchange for legal tender and specie. But nobody would say that this system has compelled any bank to face its notes in the same sense in which it has to face its liability for checks drawn against deposits.” Explain.
  3. Consider the effects on bank-note circulation and redemption of (1) exchange of notes among banks; (2) legislative prohibition of payment by a bank of notes other than its own; and give historical examples of the use of one or the other method.
  4. Does the United States Treasury now carry on a banking business? Did the Comptoir d’Escompte in 1848? The Prussian government in 1866?
  5. Does a banker lend his own money? the money of others?
  6. To what extent, and for what reasons, should the operations of savings-banks, private bankers, and trust companies, be excluded from consideration in this course?

Source: Harvard University Archives. Harvard University, Examination papers 1873-1915. Box 4, Bound volume: Examination Papers 1896-97. Papers Set for Final Examinations in Philosophy, History, Government, Economics, Fine Arts, Architecture, and Music in Harvard College, June 1896.

___________________________

1895-96.
Scope and Method in Economic Theory and Investigation.

Course Enrollment for Economics 132.

For Graduates and Undergraduates:—

[Economics] 132. Professor Taussig. — Scope and Method in Economic Theory and Investigation. Hf. 2 hours. 2d half-year.

Total 14: 11 Graduates, 3 Seniors.

Source: Harvard University. Report of the President of Harvard College, 1895-96, p. 63.

*  *  *  *  *  *  *  *  *  *

Final Examination

1895-96.
ECONOMICS 13.

  1. Compare Wagner’s enumeration of the problems within the scope of economic science with Keynes’s; and consider what doubts or objections there may be in regard to any of the problems mentioned by either writer.
  2. Explain and examine critically one of the following passages in Wagner:
    Section 63 (pp. 158-163).
    Section 70 (pp. 180-182).
  3. Illustrate the mode in which use is advantageously made of the deductive and the inductive method in regard to two of the following topics:

the causes which determine the general range of prices;
the prospects of socialism;
the prospects of coöperation.

  1. What peculiarities and difficulties appear for economic science if the choice of terminology and in definition? Illustrate.
  2. Is there ground for saying that the economic history of very recent times is of greater value for economic theory than the economic history of remote periods?
  3. What do you conceive to be the position in regard to method in economies of Ricardo? J.S. Mill? Roscher? Schmoller?

Source: Harvard University Archives. Harvard University, Examination papers 1873-1915. Box 4, Bound volume: Examination Papers 1896-97. Papers Set for Final Examinations in Philosophy, History, Government, Economics, Fine Arts, Architecture, and Music in Harvard College, June 1896.

___________________________

1895-96.
Communism and Socialism.

Course Enrollment for Economics 141.

For Graduates and Undergraduates:—

[Economics] 141. Asst. Professor Edward Cummings. — Communism and Socialism. — Utopias, ancient and modern. Hf. 2 hours. 1st half-year.

Total 15: 1 Graduate, 10 Seniors, 2 Juniors, 2 Sophomores.

Source: Harvard University. Report of the President of Harvard College, 1895-96, p. 63.

*  *  *  *  *  *  *  *  *  *

Final Examination

1895-96.
ECONOMICS 14.

(Arrange your answers in the order of the questions. Omit one.)

  1. The different senses in which the word Socialism is used. Where do you intend to draw the line between Socialism proper, and familiar forms of government interference and control — such as factory legislation, municipal water works, and government postal, telegraph or railroad services?
  2. “National communism has been confused with the common ownership of the family; tenure in common has been confused with ownership in common; agrarian communism with village commons.” Discuss the evidence.
  3. “Just as Plato had his Republic, Campanella his City of the Sun, and Sir Thomas More his Utopia, so Baboeuf had his Charter of Equality, Cabet his Icaria, St. Simon his Industrial System, and Fourier his ideal Phalanstery. . . . But the common criticism of Socialism has not yet noted the change, and continues to deal with the obsolete Utopias of the pre-evolutionary age.” What do you conceive to be the character of the change referred to? How far did the earlier Utopias anticipate the ideals of the modern social democracy?
  4. What indication of Socialistic tendency are to be found in the discipline of the Christian church? Explain the triple contract and its bearing on the doctrine of usury.
  5. “The Communistic scheme, instead of being peculiarly open to the objection drawn from danger of over-population, has the recommendation of tending in an especial degree to the prevention of that evil.” Explain Mill’s argument. Do you agree?
  6. To what extent are the theories of Karl Marx indebted to earlier writers in the 19th century?
  7. How far are the economic theories of (a) Lasalle, (b) Marx related to the theories of the so-called orthodox Economists? Explain critically.
  8. How far do you trace the influence of historical conditions in the social philosophies of Plato, More, Bacon, Rousseau, St. Simon, Karl Marx?
  9. What connection do you see between the teachings of Rousseau and (a) modern Socialism. (b) modern Anarchism?
  10. What, according to Hertzka, is the economic defect of the existing social and industrial system, and what is the remedy? Contrast “Freeland” with “Looking Backward.”

Source: Harvard University Archives. Harvard University, Mid-year Examinations 1852-1943. Box 3, Bound volume Examination Papers, Mid-Years 1895-96.

Also reprinted in. Harvard University, Examination papers 1873-1915. Box 4, Bound volume: Examination Papers 1896-97. Papers Set for Final Examinations in Philosophy, History, Government, Economics, Fine Arts, Architecture, and Music in Harvard College, June 1896.

 

 

 

Categories
Exam Questions Harvard

Harvard. Mid-year and Year-End Final Exams in Economics and Social Ethics, 1894-1895

 

 

With this post Economics in the Rear-view Mirror adds yet another annual collection of final examination questions for the economics courses offered at Harvard together with the questions from Professor Peabody’s “Ethics of the Social Questions” that covered issues such as poverty, labor relations, and socialism (as opposed to doctrines of individualism). In 1894-95 Frank Taussig was on sabbatical leave in Italy which accounts for his whereabouts that academic year.  Today I learned that “doctrine” was understood as a synonym for “theory” during the gay nineties, see Economics 2 (Economic Theory from Adam Smith to the present time) below.

Exams for one course taught were not included in the published collection of exams. It was Edward Cummings course Economics 14 (Philosophy and Political Economy.—Utopian Literature from Plato’s Republic to the present time). Exams for Economics 14 given in other years have been transcribed and posted.

__________________

1894-95.
PHILOSOPHY 5.

Course Title, Staffing, and Enrollment

[PHILOSOPHY] 5. Professor [Francis G.] Peabody. — The Ethics of the Social Questions. — The questions of Charity, the Family, Temperance, and the various phases of the Labor Question, as problems of practical Ethics. — Lectures, essays, and practical observations. 2 hours.

Total 84: 1 Graduate, 40 Seniors, 15 Juniors, 3 Sophomores, 25 Others.

Source: Harvard University. Report of the President of Harvard College, 1894-1895, p. 59.

 

PHILOSOPHY 5.
THE ETHICS OF THE SOCIAL QUESTIONS
Mid-Year Examination. 1895.

[Omit one question.]

  1. The Ethical Idealism of Plato, of Aristotle, and of Kant, compared with the modern doctrine of duty.
  2. Professor Sumner’s doctrine of the Social Fulcrum vs. the philosophy of scientific charity.
  3. Indicate, very briefly, the place in the History of Philanthropy of:

Frédéric Le Play,
Dorothea Dix,
Pastor von Bodelschwingh,
Charles L. Brace,
Samuel G. Howe.

  1. The Elberfeld System — its organization, officials, relation tomunicipal government, and practical working.
  2. The Liverpool System of Collection.
  3. Mr. Charles Booth’s eight classes of East London,— their definition, dimensions, traits, and proportion. (Labor and Life of the People, I. pp. 37-62.) Mr. Booth’s view of the children of Class E (p. 160).
  4. Compare Mr. Booth’s method and results in East London with his method and results in all London.
  5. Compare the principle as to direct relief of the London Charity Organization Society with that of the Boston Associated Charities. (Loch, Charity Organization, pp. 59, 82.) Which is the sounder principle? Why?
  6. The Belgian Labor Colonies,— their scope and method of classification. Compare their aims with those of the colonies of Holland and Germany.
  7. The Christian doctrine of the Social Order — its principles and its peril.

Source: Harvard University Archives. Harvard University, Mid-year examinations, 1852-1943. Box 3. Bound volume Examination Papers, Mid-Year 1894-95.

 

PHILOSOPHY 5.
THE ETHICS OF THE SOCIAL QUESTIONS.
Year-end Examination. 1895.

  1. Explain the theory of ethics which makes the basis of this course of study; and the way in which this theory is practically illustrated by phases of the modern labor question.
  2. In what respect do the social ideals of Carlyle and Ruskin seem identical, and in what respect do they appear to be inconsistent with each other?
  3. The authorship and the significance of the following phrases:

“There is no wealth but Life…. A strange political economy; the only one, nevertheless, that ever was or can be.”

“I am for permanence in all things. Blessed is he that continueth where he is.”

“The gospel of dilettantism.”

“Roots of honour.”

“Ricardo is the parent of Socialism.”

“The value of a thing is independent of opinion and of quantity.”

“The reformation was the work of a monk; the revolution must be the work of a philosopher.”

“The people are the Rock on which the Church of the future must be built.”

  1. The practical programme proposed by Scientific Socialism; the chief advantages claimed for it by its adherents; and the criticisms on it which appear to you most serious. Utilize here your reading of Naquet and The Social Horizon.
  2. Socialism and Religion. The apparent grounds for sympathy and the practical reasons for antagonism. The teachings concerning socialism in the Encyclical of 1891.
  3. The philosophy of history which encourages the Socialist, and the “Opportunist’s” view of this “Law” of social evolution.
  4. The growth of Trades Unionism in Great Britain, and its contribution to moral education.
  5. Federalism and Individualism in English Coöperation. The issue involved, and the advantages of each scheme of expansion.
  6. Compare the characteristics of the forms of Liquor Legislation in force in Massachusetts and in Pennsylvania. (Fanshawe, XI, XII.) How are licenses granted under the Brooks Law? What is the function of probation-officers in Massachusetts?
  7. How far do physiological considerations go to determine one’s duty as to drink?

Source: Harvard University Archives. Harvard University, Examination Papers, 1873-1915. Box 4. Bound volume: Examination Papers, 1893-95. “Papers Set for Final Examinations in Philosophy, History, Government and Law, Economics Fine Arts, Architecture, and Music in Harvard College, June, 1895,” pp. 6-7.

__________________

1894-95.
ECONOMICS 1.

Course Title, Staffing, and Enrollment

[ECONOMICS] 1. Professor [William] Ashley, Asst. Professor [Edward] Cummings, Dr. [John] Cummings, and Mr. [Frederick Redman] Clow. — Outlines of Economics. — Mill’s Principles of Political Economy. — Lectures on Economic Development, Distribution, Social Questions, and Financial Legislation. 3 hours.

Total 277: 2 Graduates, 39 Seniors, 18 Juniors, 159 Sophomores, 9 Freshmen, 50 Others.

Source: Harvard University. Report of the President of Harvard College, 1894-1895, p. 62.

 

ECONOMICS 1.
Mid-Year Examination. 1895.

[Arrange your answers strictly in the order of the question. One question may be omitted.]

  1. “All members of the community are not laborers, but all are consumers, and consume either unproductively or productively.” Explain and illustrate by examples. Suppose everybody resolved to consume “productively” only, what would be the result?
  2. “The distinction, then, between capital and not-capital, does not lie in the kind of commodities, but in the mind of the capitalist — in his will to employ them for one purpose rather than another.” Discuss this statement, using the following illustrations:—

Bread.
A knitting machine.
A steam engine.
A carriage.

  1. Where does true economic rent appear in the following cases:—

(a) The cultivation of a farm by its owner.
(b) The rental of a farm under a long lease by a tenant who has made permanent improvements on the land.

  1. What is the effect on values of a general fall of profits? Of a general fall of wages?
  2. What is the effect on rents of (1) an improvement in the methods of agriculture, (2) an improvement in transportation?
  3. “The price of land, mines, and all other fixed sources of income, depends on the rate of interest.” Explain.
  4. According to Mill, “Every addition to capital gives to labor either additional employment, or additional remuneration.” Why? What is the effect of an increase of labor-saving machinery on employment and on remuneration? Illustrate carefully.
  5. “Money cannot in itself perform any part of the office of capital, since it can afford no assistance to production.” Do you agree or disagree? Why? Is money capital? Is credit money? Is credit capital?
  6. What does Mill mean by “stationary state”? And what changes would bring about a progressive state?
  7. What would be the effect on prices of (1) adding to a gold and silver currency a small issue of inconvertible paper money, (2) the discovery of very rich gold fields?
  8. What do you understand by the Domestic system? By Competition? By Labor?

Source: Harvard University Archives. Harvard University, Mid-year examinations, 1852-1943. Box 3. Bound volume Examination Papers, Mid-Year 1894-95.

 

ECONOMICS 1.
Year-end Examination. 1895.

(Arrange your answers strictly in the order of the question. Omit three of the even numbers: answer all others.)

  1. “We must suppose the entire savings of the community to be annually invested in really productive employment within the country itself; and no new channels opened by industrial inventions, or by more extensive substitution of the best known processes for inferior ones.” How would profits be affected supposing population (a) to remain stationary; (b) to increase in proportion to the increase in capital?
  2. The operations, therefore, of speculative dealers, are useful to the public whenever profitable to themselves; and although they are sometimes injurious to the public, by heightening the fluctuations which their more usual office is to alleviate, yet, whenever this happens the speculators are the greatest losers. Explain Mill’s reasoning.
  3. Mill says of the stationary state, “I am inclined to believe that it would be, on the whole, a very considerable improvement on our present condition.” Why? Explain carefully.
  4. Is there a necessary hostility of interests between consumers organized in co-operative associations and producers organized in trade unions?
  5. Describe the different results obtained in co-operation by distributing profits in the form of dividend (a) on capital, (b) on labor (in proportion to wages), (c) on purchases. Illustrate by the experience of co-operation in France and England.
  6. How do you distinguish between what Mill calls the necessary and the optional functions of government?
  7. “We have had an example of a tax on exports, that is, on foreigners, falling in part on ourselves. We shall therefore not be surprised if we find a tax on imports, that is, on ourselves, partly falling on foreigners.” Explain carefully each case, tracing the possible effects upon prices and international trade of taxes (a) upon exports; (b) upon imports.
  8. “Equality of taxation, therefore, as a maxim of polities, means equality of sacrifice.” Apply this maxim to a tax on incomes.
  9. Suppose a tax of a fixed sum per bushel to be laid upon corn; what would be the effect (a) upon prices; (b) upon population; (c) upon profits; (d) upon rents?
    How would the results differ if instead of a fixed sum per bushel the tax were…

(i) …a fixed proportion of the produce;
(ii) …proportioned to the rent of the land;
(iii) …a fixed sum of so much per cultivated acre? Explain carefully each case.

  1. Describe the kinds of currency used in the United States, indicating briefly the conditions of issue in each case.
  2. Explain the causes and effects of (a) combined reserves, (b) a suspension of the Bank Charter Act in England.
  3. What are the provisions of the law in regard to the issue of bank notes at the present day in England? In Germany?

Source: Harvard University Archives. Harvard University, Examination Papers, 1873-1915. Box 4. Bound volume: Examination Papers, 1893-95. “Papers Set for Final Examinations in Philosophy, History, Government and Law, Economics Fine Arts, Architecture, and Music in Harvard College, June, 1895,” pp. 33-34.

__________________

1894-95.
ECONOMICS 2.

Course Title, Staffing, and Enrollment

[ECONOMICS] 2. Professors Ashley and [Silus Marcus] Macvane. — Economic Theory from Adam Smith to the present time. — Selections from Adam Smith and Ricardo. — Modern Writers. — Lectures. 3 hours.

Total 34: 9 Graduates, 14 Seniors, 6 Juniors, 1 Sophomores, 4 Others.

Source: Harvard University. Report of the President of Harvard College, 1894-1895, p. 62.

 

ECONOMICS 2.
Mid-Year Examination. 1895.

N.B.—Not more than seven questions must be attempted.

  1. “The study which lately in England has been called Political Economy is, in reality, nothing more than the investigation of some accidental phenomena of modern commercial operations, nor has it been true in its investigation even of these. It has no connection whatever with political economy, as understood and treated of by the great thinkers of past ages; and as long as its unscholarly and undefined statements are allowed to pass under the same name, every word written on the subject by those thinkers—and chiefly the words of Plato, Xenophon, Cicero, and Bacon—must be nearly useless to mankind” (Ruskin, Munera Pulveris). Consider some or all of these assertions.
  2. Give a brief account of the Physiocrat doctrine, and state to what extent it was “corrected” by Adam Smith.
  3. Explain the origin and content of Adam Smith’s conception of “Nature.”
  4. “A diamond has scarcely any value in use.” Consider this statement in its relation to the discussion since Adam Smith’s time of the doctrine of Value.
  5. How does the doctrine of Rent expounded by Adam Smith agree with, and differ from, that of Ricardo?
  6. Compare Adam Smith’s “natural rate of wages” with Ricardo’s “natural price of labour.”
  7. “Population tends to outstrip the means of subsistence.” Distinguish the various meanings assignable to this phrase, and indicate which was meant by Malthus.
  8. What does Adam Smith understand by “Capital”? Compare his conception with that of John Stuart Mill.
  9. Present a critical estimate—based upon your own study—of one of the following:

1. Ingram, History of Political Economy.
2. Price, Political Economy in England.
3. Cossa, Introduction to the Study of Political Economy.

Source: Harvard University Archives. Harvard University, Mid-year examinations, 1852-1943. Box 3. Bound volume Examination Papers, Mid-Year 1894-95.

 

ECONOMICS 2.
Year-end Examination, 1895

Answer at least four, but not more than six, of the following questions:

  1. What is the economic source of Interest? Examine the proposition that “interest is the price paid for the use of capital.”
  2. State briefly your conclusions as to the law of general wages.
  3. Apply the Austrian theory of wages to the following case:
    Number of laborers 1,000,000; total subsistence fund $600,000,000; scale of increase of productiveness of labor as the “productive period” is lengthened from one year to seven years: $350, $450, $530, $580, $620, $650, and $670.
  4. How, in your opinion, are the profits of employers determined? What is your conclusion as to the function, in distribution, of the so-called “no profits employers.”
  5. Discuss the following passages:
    “This National Dividend is at once the aggregate Net product of, and the sole source of payment for, all the agents of production within the country: it is divided up into Earnings of labour, Interest of capital, and lastly the Producer’s Surplus, or Rent, of land and of other differential advantages for production. It constitutes the whole of them and the whole of it is distributed among them.”
    “The proposal to put rent aside while we are considering how earnings and interest are determined, has been found to suggest that rent is determined first and then takes part in determining earnings and interest; and this is, of course, the opposite of what really occurs.”
  6. It has been said that Mill expresses his meaning badly when he said that demand for commodities is not a demand for labor. Does the proposition seem to you to need revision!
  7. Does increase of saving tend to make the supply of goods outrun the demand for goods?
  8. Examine the doctrine that the exchange value of commodities is determined by marginal utility.
  9. Past and present relations between gold and silver.

Source: Harvard University Archives. Harvard University, Examination Papers, 1873-1915. Box 4. Bound volume: Examination Papers, 1893-95. “Papers Set for Final Examinations in Philosophy, History, Government and Law, Economics Fine Arts, Architecture, and Music in Harvard College, June, 1895,” p. 35.

__________________

1894-95.
ECONOMICS 3.

Course Title, Staffing, and Enrollment

[ECONOMICS] 3. Asst. Professor Cummings. — The Principles of Sociology. — Development of the Modern State, and of its Social Functions. 2 hours.

Total 52: 10 Graduates, 30 Seniors, 4 Juniors, 3 Sophomores, 5 Others.

Source: Harvard University. Report of the President of Harvard College, 1894-1895, p. 62.

 

ECONOMICS 3.
Mid-Year Examination. 1895.

Answer the questions in the order in which they stand. Omit three questions.

  1. State accurately the reading you have done in this course to date.
  2. “But now let us drop the alleged parallelism between individual organizations and social organizations. I have used the analogies elaborated but as a scaffolding to help in building up a coherent body of sociological inductions. Let us take away the scaffolding: the inductions will stand by themselves.” What are these inductions?
  3. “The family relinquishes one provisional and temporary function after another; its only purpose being to fill gaps in social offices, it made way for independent institutions … as soon as these institutions arose.” Explain and illustrate. How far would Spencer assent to this doctrine?
  4. “Most anthropologists who have written on prehistoric customs believe, indeed, that man lived originally in a state of promiscuity or ‘communal marriage’; but we have found this hypothesis is essentially unscientific.” Discuss the evidence.
  5. “The status of children, in common with that of women, rises in proportion as the compulsory coöperation characterizing militant societies, becomes qualified by the voluntary coöperation characterising industrial societies.” Why? Trace the rise, and illustrate.
  6. “These three distinct states of mind, all of which, in point of fact, are admitted to exist together at the present time, and perhaps to have always done so to a greater or less extent, Comte declares to have undergone a regular progressive movement in the history of society. There have been three successive epochs, during which these philosophic principles, each in its turn, preponderated over both the others and controlled the current of human events.” Explain.
  7. “So that as law differentiates from personal commands, and as morality differentiates from religious injunctions, so politeness differentiates from ceremonial observance. To which I may add, so does rational usage differentiate from fashion.” Explain and illustrate.
  8. How does Spencer account for the diverse types of political organization; and what influences determine the order in which they arise? Illustrate.
  9. “From the Evolution-standpoint we are thus enabled to discern the relative beneficence of institutions which, considered absolutely, are not beneficent; and are taught to approve as temporary that which, as permanent, we abhor.” Explain and illustrate. Does our idea of progress then include all social changes?
  10. “In all ways, then, we are shown that with this relative decrease of militancy and relative increase of industrialism, there has been a change from a social order in which individuals exist for the State, to a social order in which the State exists for individuals?” Explain and illustrate.
  11. According to Spencer, what are likely to be the future forms of political organization and action in societies that are favorably circumstanced for carrying social evolution to its highest stage?
  12. “At bottom this is a physical explanation, and Spencerian sociology in general, whether formulated by Mr. Spencer or by other writers under the influence of his thought, is essentially a physical philosophy of society, notwithstanding its liberal use of biological and psychological data.” Do you agree or disagree? Why?

Source: Harvard University Archives. Harvard University, Mid-year examinations, 1852-1943. Box 3. Bound volume Examination Papers, Mid-Year 1894-95.

 

ECONOMICS 3.
Year-end Examination

[Answer the questions in the order in which they stand. Omit one question.]

  1. State accurately the reading you have done in this course to date.
  2. What has been the function of religion in social evolution? (Compare Spencer and Kidd.) Do you find reasons for thinking society will become more religious?
  3. “The only conclusion to which we are brought by this prolonged examination of authorities is that community of land has not yet been historically proved.” Discuss the evidence.
  4. “And as of old, Society and State tend to coincide, political questions to become identical with social questions.” Discuss the historical changes and tendencies in question. Distinguish carefully between Society, the State, the Government, the Nation.
  5. “It is becoming clear that, when people speak of natural rights of liberty, property, etc., they really mean, not rights which once existed and have been lost, but rights which they believe ought to exist, and which would be produced by a condition of society and an ordering of the State such as they think desirable.” Explain. How far do changes in the theory and practice of penal legislation substantiate this view?
  6. “The gulf between the state of society towards which it is the tendency of the process of evolution now in progress to carry us, and socialism, is wide and deep.”
    “The Individualism of the past is buried, and the immediate future is unmistakably with a progressive Socialism, the full extent of which no man can get see.” Discuss carefully the facts and theories upon which these opposing views are based.
  7. “The philanthropic and experimental forms of socialism, which played a conspicuous rôle before 1848, perished then in the wreck of the Revolution, and have never risen to life again.” What were the characteristics of these earlier forms; and what was their relation to the movements which preceded them and followed them?

Source: Harvard University Archives. Harvard University, Examination Papers, 1873-1915. Box 4. Bound volume: Examination Papers, 1893-95. “Papers Set for Final Examinations in Philosophy, History, Government and Law, Economics Fine Arts, Architecture, and Music in Harvard College, June, 1895,” pp. 35-36.

__________________

1894-95.
ECONOMICS 5.

Course Title, Staffing, and Enrollment

[ECONOMICS] 52. Mr. George Ole Virtue. — Railway Transportation. — Lectures and written work. 3 hours. 2d half-year.

Total 21: 2 Graduates, 10 Seniors, 6 Juniors, 1 Sophomores, 2 Others.

Source: Harvard University. Report of the President of Harvard College, 1894-1895, p. 62.

 

ECONOMICS 5.
Year-end Examination. 1895.

  1. Sketch the railroad history of France.
  2. “The [Reilly] bill now before Congress proposes to extend the debt for another fifty years and a grand opportunity will thus be let slip for trying, under the most favorable circumstances, an experiment whose possibilities no man can measure.”
  3. What legislation can you suggest for improving the relations between the different classes of owners of railway capital? For the protection of the interests of investors in railway capital generally?
  4. State briefly the significance in railway history of the following cases: Munn v. Illinois; Wabash, etc. Ry. Co. v. Illinois; Ames v. U. P. Ry. Co.; Budd v. New York; In re Louisville & Nashville; The Denaby Main Colliery Case.
  5. Choose one:

(a) The bearing upon the making of rates, of the “cost of service”; “value of service”; “charging what the traffic will bear”; “joint cost.”
(b) “Group rates,” “equal mileage rates,” “the blanket rate,” “the postage rate,” “Wagen-raum tarif,” “differentials.”
(c) A “reasonable rates.”

  1. Recount the experience which has led the Interstate Commerce Commission to recomment an amendment to the Act to Regulate Commerce: (a) Construing the meaning of “the word ‘line’ when used in the act to be a physical line and not a business arrangement”; (b) relieving “shippers and individuals not connected with railway employment from liability to fine and imprisonment under Section 10,” with certain exceptions.
  2. What would be the probable effect of giving the Commission power to prescribe minimum as well as maximum rates? Would it obviate the necessity now claimed for pooling?
  3. “When the first bill to regulate commerce was passed the great and powerful wedge of State socialism, so far as government control of railroads is concerned, was driven one-quarter of its length into the timber of conservative government. … The pending bill, [the pooling which passed the House at the last session is referred to] the moment it becomes a law, will drive the wedge three-quarters of its length into the timber.”
    Give your reasons for agreeing or disagreeing with each of the above statements.
  4. What conclusions on the question of public management can you draw from the experience of the states in the internal improvement movement?
  5. Why is it peculiarly true in railway business that “competition must end in combination”?
  6. The success of the State Railroad Commissions and suggestions for increasing their efficiency.

Source: Harvard University Archives. Harvard University, Examination Papers, 1873-1915. Box 4. Bound volume: Examination Papers, 1893-95. “Papers Set for Final Examinations in Philosophy, History, Government and Law, Economics Fine Arts, Architecture, and Music in Harvard College, June, 1895,” pp. 36-37.

__________________

1894-95.
ECONOMICS 71.

Course Title, Staffing, and Enrollment

[ECONOMICS] 71. Professor [Charles F.] Dunbar. — The Theory and Methods of Taxation, with special reference to local taxation in the United States. 3 hours. 1st half-year.

Total 28: 6 Graduates, 11 Seniors, 9 Juniors, 2 Sophomores.

Source: Harvard University. Report of the President of Harvard College, 1894-1895, p. 62.

 

ECONOMICS 71.
Mid-year Examination. 1895.

  1. What is the “Benefit Theory” of taxation? What is the “Faculty Theory”? Define “Faculty” as used in this expression.
  2. What are the leading points of difference between the English, Prussian and American income tax systems?
  3. What reasons are there for having income tax levied by national authority rather than local? To what extent, if at all, do these reasons apply also to a general property tax?
  4. In levying a general property tax, should the debts of the taxpayer be deducted from the property held by him?
  5. By what reasoning is it maintained that,—
    “When the local real estate tax is levied according to rental value and assessed in the first instance on the occupier, as is the case in England, the main burden of the tax will rest ultimately on the occupier, not the owner of the premises.”
    Will the same reasoning apply to the income tax on rent, assessed under Schedule A., and collected from the occupier?
  6. What are the leading points of difference between the German method of taxing distilled liquors and the method practised in England and the United States?
  7. The theories of Canard, Thiers and Stein are,—
    “That every tax is shifted on everybody — that every consumer will again shift the tax on a third party, and that this third party who is again a consumer will shift it to someone else — and so ad infinitum. And since everyone is a consumer, everyone will bear a portion of the taxes that everybody else pays.”
    Professor Seligman’s comment is that “the error of this doctrine lies in the failure to distinguish between productive and unproductive consumption.” Is this answer complete? If not, wherein does it fail?
  8. In a statement of the circumstances under which a tax may or may not be capitalized, it is said,—
    “The principle would not apply to special taxes on property or profits if the capital value of this class of commodities should for any other reason fluctuate in price. For example, if a special tax were levied on government securities it might nevertheless happen that if some reason confidence in government bonds, as over against general securities, might decrease to such an extent as to counterbalance the decreased returns from the investment. In such a case there would be no capitalization of the tax.”
    What criticism have you to make on this reasoning?
  9. Can the theory of progressive taxation be satisfied by a gradually decreasing rate of progression [“degressively progressive taxation”]. or does it require a rate which shall cut off all income or accumulation above a certain level?
  10. What practical difficulties does the taxation of real estate offer in shaping a system of progressive taxation?

Source: Harvard University Archives. Harvard University, Examination Papers, 1873-1915. Box 4. Bound volume: Examination Papers, 1893-95. “Papers Set for Final Examinations in Philosophy, History, Government and Law, Economics Fine Arts, Architecture, and Music in Harvard College, June, 1895,” pp. 37-38.

__________________

1894-95.
ECONOMICS 72.

Course Title, Staffing, and Enrollment

[ECONOMICS] 72. Professor Dunbar. — Financial Administration and Public Debts. 3 hours. 2d half-year.

Total 28: 7 Graduates, 11 Seniors, 9 Juniors, 1 Sophomores.

Source: Harvard University. Report of the President of Harvard College, 1894-1895, p. 62.

 

ECONOMICS 72.
Year-end Examination

[Spend an hour on A, and the remainder of the time on B.]

A.

  1. Give an account of the management of the English debt in the decade 1880-90.
  2. Do “sound rules of finance” demand that the principal of the debt or the rate of interest shall be determined by the government? that securities shall never be issued below par? that a government shall not buy in its securities at a premium?

B.

  1. How far, if at all, is the government justified in pledging itself to any fixed policy of debt payment?
    How may the policy of conversation conflict with the policy of debt payment?
  2. Give an account of the United States refunding operations in the decade 1865-75.
  3. Discuss the respective powers of the Secretary of the Treasury of the United States, the Chancellor of the Exchequer in Great Britain, the minister of finance in France.
    In each case where does the responsibility for the financial policy of the government rest?
  4. Give an account of the creation of Pit’s sinking fund and of the successive modifications made in the sinking fund provisions down to 1803.
  5. Discuss the various methods of placing government securities in the market, and the conditions of contract which make one form of security more attractive to buyers than another.
  6. The United States 4 per cent. 30-year bonds are quoted at about 123¼; how is the present worth of these securities determined?
    What determines the present worth of a terminable annuity? of a perpetual annuity? of a life annuity?
  7. Discuss the manner of making up the estimates of public income and expenditure in Great Britain and in France; the manner of providing for any deficits which may occur in any department during the year; the manner of providing for carrying on the government where the enactment of the budget is delayed until after the beginning of the year; and the disposal of balances unexpended at the end of the year.

Source: Harvard University Archives. Harvard University, Examination Papers, 1873-1915. Box 4. Bound volume: Examination Papers, 1893-95. “Papers Set for Final Examinations in Philosophy, History, Government and Law, Economics Fine Arts, Architecture, and Music in Harvard College, June, 1895,” p. 39.

__________________

1894-95.
ECONOMICS 8.

Course Title, Staffing, and Enrollment

[ECONOMICS] 81. Professor Dunbar. — History of Financial Legislation in the United States. 2 hours.

Total 52: 5 Graduates, 22 Seniors, 22 Juniors, 3 Others.

Source: Harvard University. Report of the President of Harvard College, 1894-1895, p. 62.

 

ECONOMICS 8.
Mid-Year Examination. 1895.

  1. Hamilton is sometimes said to have favored the policy of perpetual debt, and Gallatin, on the other hand, to have established the policy of debt-payment. How far are these statements confirmed by the measures of Hamilton and Gallatin respectively?
  2. How far should you say that Hamilton was justified in his expectation (stated in the Report on Public Credit), (1) That the public debt, if properly funded, would answer most of the purposes of money, and (2) that it would increase the amount of capital for use in trade and lower the interest of money?
  3. When were the several classes of obligations in which the revolutionary debt was funded finally paid off?
  4. Was it fortunate or unfortunate that Congress did not adopt Madison’s policy as to a United States Bank in January, 1815? Why?
  5. Give a list, with dates, of the cases in which bills for establishing a United States Bank have been vetoed.
  6. Give as complete a chronology as you can of the events connected with the Bank, from President Jackson’s first attack upon it down to its final failure.
  7. The removal of the deposits is sometimes spoken of as a fatal blow to the United States Bank. What do you gather from your reading as to its importance as regards the business position or credit of the Bank?
  8. What was the Specie Circular of 1836, and what serious financial results did it produce?
  9. What led to the adoption of the National Bank system in 1863?
  10. How would it have eased the financial difficulty in 1861, if the Secretary of the Treasury had made more free use of his authority, under the act of August 5, for suspending some of the provisions of the Independent Treasury act?
  11. The earlier legal-tender acts provided for funding the notes, at the pleasure of the holder, in United States bonds. When and why was this privilege of funding withdrawn? What would probably have been the effect if it had been retained until the close of the war?
  12. What were the steps by which the legal tender issues came to be treated as the practically permanent element in our paper currency and to be fixed in amount?

Source: Harvard University Archives. Harvard University, Mid-year examinations, 1852-1943. Box 3. Bound volume Examination Papers, Mid-Year 1894-95.

__________________

1894-95.
ECONOMICS 9.

Course Title, Staffing, and Enrollment

[ECONOMICS] 9. Asst. Professor Cummings. — The Social and Economic Condition of Workingmen in the United States and in other countries. 3 hours.

Total 79: 3 Graduates, 34 Seniors, 31 Juniors, 5 Sophomores, 6 Others.

Source: Harvard University. Report of the President of Harvard College, 1894-1895, p. 62.

 

ECONOMICS 9.
Mid-Year Examination, 1895.

(Arrange your answers, in the order in which the questions stand. So far as possible illustrate your discussions by a comparison of the experience of different countries.)

  1. State accurately the reading you have done in this course to date.
  2. “The interests of the working classes are identical in all lands governed by capitalist methods of production. The extension of the world’s commerce and production for the world’s markets, make the position of the workman in any one country daily more dependent upon that of the workmen in other countries.” Why? Explain how in the history of trade unions this community of interest among workmen, not only of the same trade and the same country but of different trades and different countries, has actually manifested itself. Illustrate.
  3. Precisely what answer to the “lump of labor” theory is to be drawn from that version of the wage-fund doctrine adopted by Mill, by Walker, by yourself?
  4. How far has the theory and the practice of coöperation offered a complete remedy for the evils of the existing industrial organization? and at precisely what points has the theory and the practice broken down? Illustrate carefully.
  5. “The struggle of the working classes against capitalist exploitation must of necessity be a political struggle.” How far does the history of trade unions and of coöperation show a tendency in this direction? Illustrate carefully.
  6. “But above all things, observe that all types of piece wage, whether single or progressive, and whether individual or collective, possess this most marked superiority over Profit-sharing.” … “At the same time, it is right to remark that there are many cases, in which the method of Profit-sharing surpasses in important respects any form of the ordinary wage-system.” Explain carefully the grounds of the alleged inferiority and superiority in each case.
  7. “Before, therefore, the trade union can realize its policy of ‘collective bargaining,’ it must solve the two-fold problem – how to bind its own constituents, and how to obtain the recognition of employers.” By what methods have trade unions endeavored to solve this problem? Illustrate.
  8. Trace the successive stages of the so-called “industrial revolution” during the last hundred and fifty years.

Source: Harvard University Archives. Harvard University, Mid-year examinations, 1852-1943. Box 3. Bound volume Examination Papers, Mid-Year 1894-95.

 

ECONOMICS 9.
Year-end Examination. 1895.

[Arrange your answers in the order in which the questions stand. So far as possible illustrate your discussions by statistical and descriptive matter showing the relative condition of working people in the United States and in other countries.]

I.

State accurately the reading you have done in this course since the mid-year examinations.

II.

Devote three hours to a careful discussion of the merits and defects of the German system of compulsory insurance, under the following general heads:

  1. An accurate account of the origin, scope, organization, administration of the system in Germany, — stating approximately the numbers insured, the cost of insurance to all parties concerned, the benefits provided, the methods of collection, distribution, etc.;
  2. Difficulties, opposition, and criticisms thus far encountered;
  3. Progress of similar movements towards compulsory insurance in other countries;
  4. Facts bearing upon the adequacy of existing provisions for sickness, accident, old age in England and the United States;
  5. A biographical sketch showing at what age and in what respects the State already interferes to prescribe conditions of employment, education, etc., for operatives reared from childhood to old age in the factory system of Massachusetts: showing also the additional interference which would be involved in the adoption of the German system of compulsory insurance;
  6. Conclusion.

Source: Harvard University Archives. Harvard University, Examination Papers, 1873-1915. Box 4. Bound volume: Examination Papers, 1893-95. “Papers Set for Final Examinations in Philosophy, History, Government and Law, Economics Fine Arts, Architecture, and Music in Harvard College, June, 1895,” p. 40.

__________________

1894-95.
ECONOMICS 10.

Course Title, Staffing, and Enrollment

[ECONOMICS] 10. Professor Ashley. — The Elements of Economic History from the Middle Ages to Modern Times. 2 hours.

Total 61: 9 Graduates, 20 Seniors, 21 Juniors, 10 Sophomores, 1 Other.

Source: Harvard University. Report of the President of Harvard College, 1894-1895, p. 62.

 

ECONOMICS 10.
Mid-Year Examination. 1895.

I. To be first attempted by all.

Translate, and comment on, the following passages:

  1. Quomodo vocatur mansio; quis tenuit eam T. R. E.; quis mod tenet; quot hidae; quot carrucae in dominio; quot hominum; quot villani; quot cotarii; quot servi; quot liberi homines; quot sochemanni.
  2. De virgis operantur ii diebus in ebdomada.
  3. Rex. . . destinavit per regnum quos ad id prudentiores.. . . cognoverat, qui circumeuntes et oculata fide fundos singulos perlustrantes, habita aestimatione victualium quae de hiis solvebantur, redegerunt in summam denariorum.
  4. Interiors plerique frumenta non serunt, sed lacte et carne vivunt, pellibusque sunt vestiti.
  5. Ideo rogamus, sacratissime imperator, subvenias. . . . ademptum sit jus etiam procuratoribus, nedum conductori, adversus colonos ampliandi partes agrarias.
  6. Arva per annos mutant et superest ager.
  7. Ego Eddi episcopus terram quae dicitur Lantocal tres cassatos Heglisco abbati libenter largior.
  8. Rex misit in singulos comitatus quod messores et alii operarii non plus caperent quam capere solebant.
  9. Noveritis nos concessisse omnibus tenentibus nostris . . . . quod omnia praedicta terrae et tenementa de cetero sint libera, et liberae conditionis.

II. Write on two only of the following subjects.

  1. The reasons for believing in the survival in Britain of the Roman agrarian organisation.
  2. A comparison, from the economic point of view, of the open-field system with modern methods of farming.
  3. The condition of the tillers of the soil in England in A.D. 1381 as compared with A.D. 1066.

Source: Harvard University Archives. Harvard University, Mid-year examinations, 1852-1943. Box 3. Bound volume Examination Papers, Mid-Year 1894-95.

 

ECONOMICS 10.
Year-end Examination. 1895.

I.
[To be first attempted by all.]

TRANSLATE, and comment on, the following passages:—

  1. In Kateringes sunt x. hidae ad geldum Regis. Et de istis
  2. hidis tenent xl. villani xl. virgas terrae. … Et omnes isti homines operantur iiibus diebus in ebdomada.
  3. Agriculturae non student; majorque pars eorum victus in lacte caseo carne consistit; neque quisquam agri modum certum aut fines habet proprios; sed magistratus ac principes in annos singulos gentibus cognationibusque hominum qui una coierunt, quantum et quo loco visum est agri attribuunt atque anno post alio transire cogunt.
  4. Nul ne deit rien acheter a revendre en la vile meyme, fors yl serra Gildeyn.
  5. Cives Londoniae debent xl marcas pro Gilda Telaria delenda; ita ut de cetero non suscitetur.
  6. Johannes Hore mortuus est, qui tenuit de domino dimidiam acram terrae cujus heriettum unus vitulus precii iiii d. Et Johanna soror dicti Johannis est proximus heres, quae venit et gersummavit dictam terram tenendam sibi et suis in villenagio ad voluntatem per servicia et consuetudines.

II.
[Write on four only of the following subjects.]

  1. “I contend that from 1563 to 1824, a conspiracy, concocted by the law and carried out by parties interested in its success, was entered into, to cheat the English workman of his wages, to tie him to the soil, to deprive him of hope, and to degrade him into irremediable poverty.” Consider this.
  2. Discuss the question whether the statute of 5 Eliz. c. 4, displays any distinct economic policy.
  3. Explain the causes, nature and consequences of the change in commercial routes in the sixteenth century.
  4. What is meant by a national economy, as contrasted with a town economy? Illustrate from European conditions in the 15th and 16th centuries.
  5. What has been the economic gain to England from immigration?
  6. Mention briefly those respects in which the economic development of England has resembled that of Western Europe, and those respects in which it has been peculiar.

Source: Harvard University Archives. Harvard University, Examination Papers, 1873-1915. Box 4. Bound volume: Examination Papers, 1893-95. “Papers Set for Final Examinations in Philosophy, History, Government and Law, Economics Fine Arts, Architecture, and Music in Harvard College, June, 1895,” pp. 40-41.

 

 

Categories
Exam Questions Harvard

Harvard. Semester exams for all economics and one social ethics course, 1893-1894

 

With this post Economics in the Rear-view Mirror adds yet another annual slice of final examinations from Harvard. Over twenty pages of exam questions (with course enrollment figures) for the 1893-94 academic year have been transcribed and are now available to the internet community of historians of economics.  For other years visitors can simply scan or search the chronological catalogue of artifacts. Alternatively using Google search constrained to Economics in the Rear-view Mirror, “harvard economics exams site:irwincollier.com“, will get you links to plenty of Harvard examination postings through the years.

______________________

Enrollment for Philosophy 5.
The Ethics of the Social Questions.
1893-94.

Enrollment.

[Philosophy] 5. Professor Peabody. — The Ethics of the Social Questions. — The questions of Charity, Divorce, the Indians, Temperance, and the various phases of the Labor Question, as problems of practical Ethics. — Lectures, essays, and practical observations. 2 hours.

Total 118: 6 Gr., 56 Se., 23 Ju., 2 So., 12 Others, 19 Divinity.

Source: Harvard University. Report of the President of Harvard College, 1893-94, p. 58.

1893-94.
PHILOSOPHY 5.
THE ETHICS OF THE SOCIAL QUESTIONS.
Mid-Year Examination.

  1. “Political Economy ought to combine with the old question: ‘Will it pay?’ another and higher query: ‘Is it right?’” (C. D. Wright, Political Economy and the Labor Question, p. 17.) The place and value of this view of Political Economy.
  2. Spencer’s formula for conduct, explained and criticized (Data of Ethics, p. 14.)
  3. The Socialist’s view of Charity and the argument which sustains it. Mr. Spencer’s view of Charity and his practical advice. (Principles of Ethics, II. p. 376, ff.)
  4. What does Mr. Charles Booth regard as the “crux” of the Social Problem in East London? (Labour and Life of the People, I. pp. 596 and 162.) Why? The practical remedy proposed by him.
  5. The causes of poverty in East London, as analyzed by Mr. Booth, (I. 147); in their order of importance and the proportion of cases involved.
  6. The Labor Colonies of Germany compared with those of Holland, in method and intention. How far, and under what principle, is such an enterprise applicable to the condition of this country?
  7. Liberalitas” and “Caritas,” — the aim, the service, and the peril of each.
  8. The historical development and the practical rules of the English Poor-Law System.
  9. The Relation of Charity Organization in England to Poor-Law Relief. (Loch, p. 37, ff.); and the objections to Charity Organization. (Loch, p. 97, ff.)
  10. The growth of Charity Organization in the United States, its present extent and its two types (Report, pp. 1-8.) Which type is represented by the London Charity Organization Society? (Loch p. 54.) Which is the sounder principle for this country? Why? Which is the more generally accepted principle? (Appendix of Report, p. 34.)

Source: Harvard University Archives. Harvard University, Mid-year examinations, 1852-1943. Box 3, Volume: Examination Papers, Mid-Year, 1893-94

PHILOSOPHY 5.
THE ETHICS OF THE SOCIAL QUESTIONS.
Final Examination

[Omit one question.]

  1. The authorship and the historical importance of the following phrases:—
    “The value of a thing is independent of opinion and of quantity. To be valuable is to avail towards life.”
    “All commodities are only masses of congealed labor-time.”
    “The high road to a stable sufficiency and comfort among the people is through the medium of their character.”
    “Cash-payment never was or could, except for a few years, be the union-bond of man to man.”
    “Aristocracy of talent.”
    “It is easier to determine what a man ought to have for his work, than what his necessities will compel him to take for it.”
    “Ill-th.”
  2. Compare Carlyle and Ruskin in their attitudes toward the growth of democracy and in their doctrine of social progress.
  3. Compare the view of the “Social Horizon” with that of Naquet as to the effect of collectivism on enterprise and invention. (Social Horizon, pp. 112-151; Naquet, pp. 92-126.)
  4. The Anarchist’s criticism of the Socialist, the Socialist’s criticism of the Anarchist, and the Communist as he is criticised by both.
  5. Is thrift a virtue? Who doubts it? Why?
    Is competition an evil? Who doubts it? Why?
  6. Christian Socialism and its difficulties. The logical and the practical relation of Socialism to Religion.
  7. In the four ideals which are possible to Socialism and Individualism, “the normal relation would be that of cross-correspondence.” (Bosanquet. The Civilization of Christendom, p. 136.) Explain and comment on this statement.
  8. Enumerate and classify the arguments presented in the Course on the ethical aspects of Socialism, with your judgment of the weight of these suggestions.
  9. Compare the plan of profit-sharing in the Paris and Orleans Railway (Sedley Taylor, pp. 77-86) with that adopted by the Chicago, Burlington and Quincy Railroad.
  10. How far are we carried in the argument for abstinence from intoxicating drink by considerations drawn from the “risks of life.” Why?

Source: Harvard University Archives. Harvard University, Examination papers, 1853-2001. Box 2, Volume: Papers Set for Final Examinations in Philosophy, History, Government and Law, Economics, Fine Arts, and Music in Harvard College, June 1894, p. 7.

______________________

Economics 1.
Outlines of Economics.
1893-94.

Enrollment.

[Economics] 1. Professors Taussig and Ashley, Asst. Professor Cummings and Mr. Clow. — Outlines of Economics. — Mill’s Principles of Political Economy. — Lectures on Economic Development, Distribution, Social Questions, and Financial Legislation. 3 hours.

Total 340: 1 Gr., 35 Se., 111 Ju., 136 So., 7 Fr., 50 Others.

Source: Harvard University. Report of the President of Harvard College, 1893-94, p. 61.

1893-94.
ECONOMICS 1.
Mid-Year Examination.

[Arrange your answers strictly in the order of the questions.
One question may be omitted.]

  1. “Let us consider whether, and in what cases, the property of those who live on the interest of what they possess, without being personally engaged in production, can be regarded as capital.” Illustrate by example.
  2. “Capital, though saved, and the result of saving, is nevertheless consumed. The word saving does not imply that what is saved is not consumed, nor even necessarily that its consumption is deferred.” Explain. Who is the consumer? and is the consumption usually deferred?
  3. Are wages likely to be low or high in different occupations because of (1) attractiveness, (2) unpleasantness, of the work? Why?
  4. “This equalizing process, commonly described as the transfer of capital from one employment to another, is not necessarily the slow, onerous, and almost impracticable process which it is often represented to be.” What is the equalizing process? and why is it or is it not slow and onerous?
  5. “Even if there were never any land taken into cultivation for which rent was not paid, it would be true, nevertheless, that there is always some agricultural capital which pays no rent.” Explain, and give the reasons for the statement.
  6. What are the laws of value applicable to: silver bullion, cotton-cloth, raw hides, wheat-bread, telephones?
  7. Explain what is meant by a fall in the value of money; an appreciation of gold; a depreciation of inconvertible paper; a stable standard of value.
  8. Wherein does the play of demand and supply, in determining the value of money, differ from its operation in determining the value of commodities in general? Wherein does cost of production determine the value of money and of commodities differently?
  9. What is the effect of general high wages on prices? on values? on profits? Why?
  10. “So far as rents, profits, wages, prices. are determined by competition, laws may be assigned for them. Assume competition to be their exclusive regulator, and principles of broad generality and scientific precision may be laid down, according to which they will be regulated.” Trace the historical origin of the conditions here assumed.
  11. What seems to you to be the value of economic history in relation to the study of economic theory?

Source: Harvard University Archives. Harvard University, Mid-year examinations, 1852-1943. Box 3, Volume: Examination Papers, Mid-Year, 1893-94.

1893-94.
ECONOMICS 1.
Final Examination.

[Arrange your answers strictly in the order of the questions.]

I.
[One question in this group may be omitted.]

  1. Explain the connection between the law of diminishing returns the pressure of population on subsistence; the tendency of profits to a minimum.
  2. What is the nature of the remuneration received by the holder of a government bond; the holder of a railway bond; the landlord of a building let for business purposes; the landlord of land let for agricultural purposes; a manufacturer carrying on business with borrowed capital; the holder of a patent receiving a royalty for its use?
  3. How does cost of production influence tire value of (1) silver bullion, (2) oats, (3) coffee, (4) bicycles?
  4. What seems to you to be the value of economic history in relation to the study of economic theory?

II.
[One question in this group may be omitted.]

  1. In 1851, very rich deposits of gold were found in Australia. What would you expect the result to be in Australia on wages, prices, imports and exports?
  2. Is the gain from international trade to be found in the import or in the exports? Why and how?
  3. It is said that when the quantity of money is increased, prices rise precisely in proportion to the increase. What exceptions or qualifications would you make to this statement?
  4. Is the exportation of specie from a country disadvantageous?

III.
[Answer all in this group.]

  1. What sorts of advantages, in regard to wages, do Trade-unions and Coöperative Societies offer to workingmen?
  2. “Deposits are currency.” What is meant?
  3. What is the most important objection to the use of inconvertible paper money? What illustrations of its force do you find in the experience of the United States since 1860?
  4. Compare the policy followed in times of panic by the Bank of England, the Reichsbank of Germany, and the National Banks of the United States.

Source: Harvard University Archives. Harvard University, Examination papers, 1853-2001. Box 2, Volume: Papers Set for Final Examinations in Philosophy, History, Government and Law, Economics, Fine Arts, and Music in Harvard College, June 1894, pp. 34-35.

______________________

Economics 2.
Economic Theory from Adam Smith
to the Present Time.
1893-94.

Enrollment

For Graduates and Undergraduates:—

[Economics] 2. Professor Taussig. — Economic Theory from Adam Smith to the present time. — Examination of selections from leading writers. 3 hours.

Total 43: 12 Graduates, 16 Seniors, 10 Juniors, 1 Sophomore, 4 Others.

Source:   Harvard University, Annual Reports of the President and Treasurer of Harvard College, 1893-94, p. 61.

1893-94
ECONOMICS 2.
Mid-Year Examination.

[Arrange your answers strictly in the order of the questions.
Write with deliberation, but answer all the questions.]

  1. “It is no doubt true that a portion of capital is always remuneratory and not auxiliary in its nature; that is, does not consist of instruments that make labour more efficient, but of finished products, destined for the consumption of labourers and others. This part of capital continually becomes real wages (as well as real profits, interest, and rent), being purchased by the labourer with the money wages he receives from time to time. But it does not seem to me therefore correct to regard the real wages as capital ‘advanced’ by the employer to the labourer. The transaction between the two is essentially a purchase, not a loan. The employer purchases the results of a week’s labour, which thereby becomes part of his capital, and may be conceived — if we omit for simplicity’s sake the medium of exchange — to give the labourer in return some of the finished products of his industry.”
    Consider whether and how remuneratory capital continually becomes real interest and rent, as well as real wages; and give your opinion as to the closing analysis of the relation between employers and laborers.
  2. Suppose (1) that profit-sharing were universally adopted; (2) that laborers habitually saved a very large part of their income, — and consider whether any modification must be made in the reasoning of those who would maintain a Wages-Fund doctrine.
  3. It has been said that while the capital of the employing class is the immediate source from which wages are paid, the ultimate and important source is the income of the consumers who buy the goods made by the laborers for the capitalists. Consider this doctrine.
  4. Compare critically the treatment by Walker, Sidgwick, and Ricardo, of the relation between the profits of the individual capitalist and the amount of capital owned by him.
  5. State carefully Ricardo’s criticism of Adam Smith’s doctrine on labor as the measure of value.
  6. Compare Adam Smith’s reasoning with Ricardo’s as to the manner in which the progress of society in wealth affects profits.
  7. “We have seen that in the early stages of society both the landlord’s and the labourer’s share of the value of the produce of the earth would be but small; and that it would increase in proportion to the progress of wealth and the difficulty of procuring food. We have known, too, that although the value of the labourer’s portion will be increased by the high value of food, his real share will be diminished; while that of the landlord will not only be raised in value, but will also be increased in quantity.”
    Explain the reasoning by which Ricardo reached the several conclusions here summarized, and give your opinion as to the soundness of the conclusions.

Source: Harvard University Archives. Prof. F. W. Taussig, Examination Papers in Economics 1882-1935 (Scrapbook). Also: Harvard University Archives. Harvard University, Mid-year examinations, 1852-1943. Box 3, Volume: Examination Papers, Mid-Year, 1893-94.

1893-94
ECONOMICS 2.
[Final Examination.]

  1. “Perhaps the most striking conflict of the Wages-Fund-theory with facts, is found in the periodical influctions and depressions of trade. After a commercial crisis, when the shock is over and the necessary liquidation has taken place, we generally find that there is a period during which there is a glut of capital, and yet wages are low. The abundance of capital is shown by the low rate of interest and the difficulty of obtaining remunerative investments.” — Nicholson, Political Economy
    How far is the theory in conflict with the facts here adduced?
  2. How is the significance of the doctrine of consumer’s rent affected by the fact that the money incomes of different purchasers vary widely?
  3. Explain Marshall’s doctrine as to the influence on wages of the standard of living among laborers; and consider how far it differs from Richard’s teaching as to the connection between wages and the price of food.
  4. Explain Marshall’s doctrine of the quasi-rent of labor; compare it with his conclusions as to the rent of business ability; and point out how far he finds in either case something analogous to economic rent as defined by the classic writers.
  5. “It is not true that the spinning of yarn in a factory, after allowance has been made for the wear-and-tear of the machinery, is the product of the labour of the operatives. It is the product of their labour (together with that of the employer and subordinate managers) and of the capital; and that capital itself is the product of labour and waiting; and therefore the spinning is the product of labour (of many kinds) and of waiting. If we admit that it is the product of labour alone, and not of labour and waiting, we can no doubt be compelled by inexorable logic to admit that there is no justification for interest, the reward of waiting.”
    How far would you accept this reasoning?
  6. “Barter, though earlier historically than buying and selling, is really a mere complex transaction, and the theory of it is rather curious than important.” — Marshall.
    “The attribute of normal or usual value implies systematic and continuous production.” — Cairnes.
    “Where commodities are made for sale, the sellers’ subjective valuations fall out altogether, and price is determined by the valuation of the last buyer.” — Böhm-Bawerk.
    Explain these statements, separately or in connection with each other.
  7. What does Böhm-Bawerk mean by the general subsistence market, or the total of advances for subsistence; and how far do the “advances” differ from the wages-fund of the classic economists?
  8. Explain Böhm-Bawerk’s views as to the connection between the prolongation of the period of production, and the increase in the productiveness of labor; and consider how far his conclusions as to interest would need to be modified, if those views were changed.
  9. Explain briefly, by definition or example, the sense in which Böhm-Bawerk uses the terms, —

social capital;
private capital;
subjective value;
marginal pairs;
technical superiority of present goods.

Source: Harvard University Archives. Prof. F. W. Taussig, Examination Papers in Economics 1882-1935 (Scrapbook). Also: Harvard University Archives. Harvard University, Examination papers, 1853-2001. Box 2, Volume: Papers Set for Final Examinations in Philosophy, History, Government and Law, Economics, Fine Arts, and Music in Harvard College, June 1894, pp. 35-36.

___________________________

Economics 3.
Principles of Sociology.
1893-94.

Enrollment.

[Economics] 3. Asst. Professor Cummings. — The Principles of Sociology. — Development of the Modern State, and of its Social Functions. 3 hours.

Total 47: 17 Gr., 19 Se., 5 Ju., 6 Others.

Source: Harvard University. Report of the President of Harvard College, 1893-94, p. 61.

 

1893-94.
ECONOMICS 3.
Mid-Year Examination

(Arrange your answers in the order of your questions. Omit two.)

  1. “In fact, the conception of society as an organism seems to admit of more easy application to just those very views about the State which Mr. Spencer most dislikes: and, though the conception or organism has its value in helping political thinking out of the confusions of individualism, if it be taken as a final key to all mysteries, it leads to new confusions of its own, for which it would be absurd to blame Mr. Spencer.” Explain and criticise.
  2. How does Spencer account for the diverse types of political organization; and what influences determine the order in which they arise? Illustrate.
  3. What evidence of political evolution is there in the sequence of the various forms of political organization in Greek, Roman, and Medieval society? Trace the steps.
  4. According to Burke, “Society is indeed a contract. … It is a partnership in all science; a partnership in all art; a partnership in every virtue and in all perfection. As the ends of such a partnership cannot be obtained in many generations, it becomes a partnership not only between those who are living, but between those who are living, those who are dead, and those who are to be born.” Explain. How does this differ from earlier conceptions of the social contract? From the conception of society as an organism?
  5. Upon what grounds does Spencer base his preference for the industrial rather than the militant type of society?
  6. According to Jevons, “the first step must be to rid our minds of the idea that there are any such things in social matters as abstract rights, absolute principles, indefeasible laws, inalterable rules, or anything whatever of an eternal and inflexible nature.” According to another view, “the state presupposes rights and the rights of individuals.” What is your own opinion? Why? Are there “Natural Rights”? Illustrate.
  7. “The State is after all the least of the powers that govern us.” How far is this true at different stages of social development?
  8. What is involved in the conception of Sovereignty? In whom is it rested? On what does it rest? For example, England and the United States.
  9. What is the bearing of Comte’s maxim, “Voir pour prevour,” upon the doctrine of social evolution?
  10. “The environment in our problem must, therefore, not only include psychical as well as physical factors, but the former are immeasurably the more important factors, and as civilization advances their relative importance steadily increases.”
  11. What do you mean by State Interference? By Individual liberty?

Source: Harvard University Archives. Harvard University, Mid-year examinations, 1852-1943. Box 3, Volume: Examination Papers, Mid-Year, 1893-94.

1893-94.
ECONOMICS 3.
Final Examination

[Questions are in all cases to be discussed with direct reference to facts and theories presented in this course. Arrange your answers in the order in which the questions stand. Take either the first question or six others.]

  1. Devote three hours to a discussion of “Social Evolution”;— expounding Mr. Kidd’s views, discussing his opinions and conclusions in the light of facts and theories presented in this course, and stating carefully your own reasons for agreeing or disagreeing.
  2. What, according to Mr. Kidd, are the necessary “Conditions of Human Progress”? Do you agree or disagree? Why?
  3. What are the points of resemblance and of difference between the “Scientific Socialism” of today and earlier forms of so-called socialistic propaganda which have appeared within this century?
  4. “Step by step the community has absorbed them, wholly or partially, and the area of private exploitation has been lessened. Parallel with this progressive nationalization or municipalization of industry, there has gone on, outside, the elimination of the purely personal element in business management.” Indicate briefly the character, extent and probable significance of “nationalization and municipalization” in the United States and in European Countries.
  5. What inferences may and what may not safely be drawn from American experience in municipal ownership or control of gas, of water, and of electric light plants? Discuss carefully the extent and character of the evidence.
  6. “According to them, the tribe or horde is the primary social unit of the human race, and the family only a secondary unit, developed in later times. Indeed, this assumption has been treated by many writers, not as a more or less probable hypothesis, but as a demonstrated truth. Yet the idea that a man’s children belong to the tribe, has no foundation in fact.” Indicate briefly the present state of this controversy. What significance do you attach to it?
  7. “The central fact with which we are confronted in our progressive societies is, therefore, that the interests of the social organism and those of the individuals comprising it at any time are actually antagonistic; they can never be reconciled, they are inherently and essentially irreconcilable.” State carefully the arguments for and against this position.
  8. “True Socialism of the German type must be recognized to be, ultimately, as individualistic and as anti-social as individualism in its advanced forms.” By what line of reasoning is this conclusion reached? State carefully your reasons for agreeing or disagreeing.

Source: Harvard University Archives. Harvard University, Examination papers, 1853-2001. Box 2, Volume: Papers Set for Final Examinations in Philosophy, History, Government and Law, Economics, Fine Arts, and Music in Harvard College, June 1894, pp. 36-37.

___________________________

Economics 5 (First Semester).
Railway Transportation.
1893-994.

Enrollment.

[Economics] 51. Professor Taussig. — Railway Transportation. — Lectures and written work. 3 hours. 1st half-year.

Total 39: 3 Gr., 24 Se., 9 Ju., 1 So., 2 Others.

Source: Harvard University. Report of the President of Harvard College, 1893-94, p. 61.

1893-94.
ECONOMICS 5.
Final [Mid-Year] Examination.

[Arrange your answers strictly in the order of the questions.]

  1. State what important general lessons are to be learned from the early experiments of Pennsylvania and Michigan in constructing and managing transportation routes.
  2. Why the change in the attitude of the public towards the Pacific railways after 1870? And what were some consequences of the change?
  3. What was the effect of the land-grant system on the welfare of the community, and on railway profits?
  4. “These conditions [leading to financial losses] may fairly enough be described as the Interstate Commerce Commission describes them, — parallel railroad construction and wars of rates. But when the Commission goes on to say that they cannot with any justice be claimed to have resulted from the act or from its administration, they make an unwarranted assertion.” What were the conditions here referred to (give dates)? And was the assertion unwarranted?
  5. Consider the probable results of the repeal of the section of the Interstate Commerce act which prohibits pooling.
  6. “High rates on some articles are not to be regarded as a tax which could be removed if low rates on others were abandoned.” Why not?
  7. “The enormous fixed capital and the consequent impossibility of retiring from the enterprise if it becomes unprofitable; the greater or less degree of monopoly; the wide gulf between railway managers and investors, sometimes leading to consequences of its own,” consider in what manner and extent these circumstances have affected railway rates in the United States.
  8. What do you believe to be the significance and importance of the following figures (for the United States in 1891):
Revenue per passenger mile 2.142 cents
Average cost of carrying a passenger one mile 1.910 cents
Revenue per ton mile 0.895 cents
Average cost of carrying a ton one mile 0.583 cents
Revenue per freight train mile $1.63
Average cost of running a freight train one mile $1.06
  1. Compare the course of railway policy in France, Prussia, and Italy, in 1880-85.
  2. Compare the principles which underlie the natural (car-space) system of freight rates and the zone system of passenger rates.

Source: Harvard University Archives. Harvard University, Examination papers, 1853-2001. Box 2, Volume: Papers Set for Final Examinations in Philosophy, History, Government and Law, Economics, Fine Arts, and Music in Harvard College, June 1894, pp. 37-38.

___________________________

Economics 6 (Second Semester)
History of Tariff Legislation
in the United States.
1893-94.

Enrollment.

[Economics] 62. Professor Taussig. — History of Tariff Legislation in the United States. 3 hours. 2d half-year.

Total 97: 11 Gr., 33 Se., 36 Ju., 2 So., 1 Fr., 14 Others.

Source: Harvard University. Report of the President of Harvard College, 1893-94, p. 62.

1893-94.
ECONOMICS 6.
Final Examination

[Arrange your answers strictly in the order of the questions.
Answer all the questions.]

  1. Is it to be inferred from Hamilton’s Report on Manufactures that if he were now living, he would not be an advocate of protection?
  2. What grounds are there for saying that the act of 1789 was a protective measure?
  3. State the important provisions of the act of 1816, and consider whether it differs in any essentials from the act of 1824.
  4. Was Clay right in affirming, or Webster in denying, that the protective system of 1824 was “American”?
  5. How would you ascertain what were the duties, in 1840, on (1) woollen goods, (2) cotton goods, (3) silk goods, (4) bar iron?
  6. Suppose the present specific duties on woollen manufactures to be removed; the ad valorem duties to remain unchanged; wool to be admitted free; and consider how far there would ensue a change in the effective protection given on finer woollen cloths, on cheaper woollen cloths, and on carpets.
  7. Mention briefly what were the duties on tea and coffee in the successive stages of tariff legislation from 1789 to 1890; noting the significance of the changes made from time to time.
  8. Why do the effects, in recent times, of the duties on flax and hemp, and on glassware, “reduce themselves in the last analysis to illustrations of the doctrine of comparative costs”?
  9. Wherein is there resemblance, wherein difference, between the general course of tariff history in the United States after the civil war, and in France after the Napoleonic wars?
  10. What would be the probable effects of the removal of the present duties on cotton goods?

Source: Harvard University Archives. Harvard University, Examination papers, 1853-2001. Box 2, Volume: Papers Set for Final Examinations in Philosophy, History, Government and Law, Economics, Fine Arts, and Music in Harvard College, June 1894, pp. 38-39

___________________________

Economics 8 (First Semester)
History of Financial Legislation
in the United States.
1893-94.

Enrollment.

[Economics] 81. Professor Dunbar. — History of Financial Legislation in the United States. 2 hours. 1st half-year.

Total 63: 9 Gr., 26 Se., 23 Ju., 1 So., 4 Others.

Source: Harvard University. Report of the President of Harvard College, 1893-94, p. 62.

1893-94.
ECONOMICS 8.
Mid-Year Examination.

Instead of answering the starred questions in this paper you may substitute, if you prefer, an essay on the subject marked A, printed at the close.

  1. *“It is sometimes said that Mr. Hamilton believed in a perpetual debt, and when one notices the form into which he threw the obligations of the United States, the only escape from this conclusion is to say that he was ignorant of the true meaning of the contracts which he created.” — [H. C. ADAMs, Public Debts, p. 161.]
    How far is the above remark confirmed by the provisions as to the payment of the debt funded by the Act of 1790?
  2. How far should you say that Gallatin, although an anti-Federalist, finally adopted Federalist measures or methods in financial matters?
  3. Give a general statement of the agreement between the banks and the Treasury for the resumption of specie payment in 1817, and show the way in which it was intended to operate.
  4. Inasmuch as Jackson’s general prepossessions were unfavorable to all banks, how are we to explain his resort to the plan of depositing Government funds in State banks after the removal of the deposits in 1833?
  5. *How serious a blow did Jackson really strike when he removed the deposits from the United States bank in 1833?
  6. What expedients were suggested for supplying the needs of the government in 1861-62 without resorting to the issue of legal-tender notes?
  7. *The “Gold Bill” of June 17, 1864, and its fate.
  8. What was the process by which the bonds issued during the war were refunded under the act of 1870 and when did the refunding take place?
  9. What signs of change in the policy of Congress as to the resumption of specie payments are to be found in the legislation between 1865 and 1876?
  10. State the provisions of the Resumption Act of 1875 as to the redemption of legal-tender notes, and show whether the act did or did not provide for the possible eventual disappearance of all the notes. What has made the amount of outstanding legal-tender notes stationary at $346,681,016?
  11. *A recent writer, discussing the question of a paper currency issued by government, says:—
    “In the United States there were twenty issues of treasury notes before the late war. Those issues were receivable in the revenues the government, and were always preferred to gold.”
    What criticism is to be made on this statement?
  12. *Describe the different kinds of paper currency now in use in the United States, stating as to each the cases in which it can be tendered for private debt, and those which it. can he received or paid out by the government.

*  *  *  *  *  *  *  *  *  *
A.

The change which has taken place since 1846 in the conditions affecting the Independent Treasury, and the justification of Secretary Carlisle’s statement, in the Finance Report for 1893, that “the laws have imposed upon the Treasury Department all the duties and responsibilities of a bank of issue, and to a certain extent the functions of bank of deposit.”

Source: Harvard University Archives. Harvard University, Mid-year examinations, 1852-1943. Box 3, Volume: Examination Papers, Mid-Year, 1893-94.

___________________________

Economics 9.
The Social and Economic Condition of Workingmen in the United States and in other countries.
1893-94.

 Enrollment.

[Economics] 9. Asst. Professor Cummings. — The Social and Economic Condition of Workingmen in the United States and in other countries. 3 hours.

Total 43: 7 Graduates, 16 Seniors, 11 Juniors, 3 Sophomores, 1 Freshman, 5 Other.

Source: Harvard University. Annual Report of the President of Harvard College, 1893-94, p. 61.

Mid-year Examination.
ECONOMICS 9.
1893-94.

(Arrange your answers in the order in which the questions stand. So far as possible illustrate your discussions by a comparison of the experience of different countries. Omit two questions.)

  1. “It becomes my duty, therefore, in undertaking to interpret the social movement of our own times, to disclose, first, those changes in industrial methods by which harmony in industries has been disturbed, and then to trace the influence of such changes into the structure of society.” State carefully what these changes have been; and trace their influence.
    [Henry C. Adams. “An Interpretation of the Social Movements of our Time”, International Journal of Ethics, Vol II, October, 1891), p. 33]
  2. Discuss the effect upon wages of machinery, — (a) as a substitute for labor (b) as auxiliary to labor; (c) as affecting division of labor; (d) as concentrating labor and capital; (e) as affecting the nobility[sic, “mobility”] of labor and capital.
  3. “In my opinion, combination among workingmen is a necessary step in the re-crystallization of industrial rights and duties.” State fully your reasons for agreeing or disagreeing with this opinion. What forms of combination do you include?
    [Henry C. Adams. “An Interpretation of the Social Movements of our Time”, International Journal of Ethics, Vol II, October, 1891), p. 45]
  4. “Trade-unions have been stronger in England than on the Continent, and in America….” In what respects stronger? Why? Contrast briefly the history and present tendencies of the trade-union movement in the United States, England, France, Germany, and Italy.
    [Alfred Marshall, Elements of Economics of Industry: being the First Volume of Elements of Economics (London: Macmillan, 1892), Book VI, Ch. XIII. §18, p. 404]
  5. “Trade-unions have been stronger in England than on the Continent, and in America; and wages have been higher in England than on the Continent, but lower than in America.” “Again, those occupations in which wages have risen most in England happen to be those in which there are no unions.” How far do such facts impeach the effectiveness of trade-unions as a means of raising wages and improving the condition of workingmen? What do you conceive to be the economic limits and the proper sphere of trade-union action?
    [Alfred Marshall, Elements of Economics of Industry: being the First Volume of Elements of Economics (London: Macmillan, 1892), Book VI, Ch. §18, pp. 404-405.]
  6. “We saw at the beginning that in comparatively recent years the difficulties of keeping up a purely offensive and defensive organization had brought many of the unions back nearer their old allies, the friendly societies, and emphasized the friendly benefits in proportion as the expenditure for trade disputes seemed less important.” Explain carefully this earlier and later relation of trade-unions and Friendly Societies in England.
    [Edward Cummings, The English Trades-Unions, Quarterly Journal of Economics, Vol. III (July, 1889), p. 432.]
  7. “This spirit of independent self-help has its advantages and its disadvantages. We have already had occasion to remark how slow in these Friendly Societies has been the progress of reform, and we must repeat that up to the present day it still exhibits defects.” Explain and illustrate the progress of the reform and the nature of existing defects. Does English self-help experience suggest the desirability or undesirability of imitating German methods of compulsory insurance?
  8. “Countless[sic, “Doubtless” in original] boards of arbitration and conciliation, the establishment of certain rules of procedure, agreements covering definite periods of time, may aid somewhat in averting causes of dispute or in adjusting disputes as they arise; but if we have these alone to look to, strife will be the rule rather than the exception.” Explain the various methods adopted and the results obtained. What have you to say of “compulsory arbitration?”
    [Francis A. Walker. “What Shall We Tell the Working Classes?” Scribner’s Magazine, Vol. 2, 1887.  Reprinted in Discussions in Economics and Statistics, edited by Davis R. Dewey. Vol. II315-316.]
  9. “The conclusion of the whole matter seems to be, that what is desirable is not so much to put a stop to sub-contracting as to put a stop to ‘sweating,’ whether the man who treats the workman in the oppressive manner which the word ‘sweating’ denotes be a sub-contractor, a piece-master, or a contractor.” Indicate briefly some of the principal forms of industrial remuneration, — giving the special merits and defects of each.
    [David F. Schloss. Methods of Industrial Remuneration (London: Williams and Norgate, 1892), p. 140.]
  10. “Now that I am on piece-work, I am making about double what I used to make when on day-work. I know I am doing wrong. I am taking away the work of another man.” State and criticize the theory involved in this view of production.
    [David F. Schloss. Methods of Industrial Remuneration (London: Williams and Norgate, 1892), p. 43-44.]

Source: Harvard University Archives. Harvard University, Mid-year examinations, 1852-1943. Box 3, Volume: Examination Papers, Mid-Year, 1893-94. Transcribed and posted earlier at Economics in the Rear-view Mirror.

Year-End Examination
ECONOMICS 9.
1893-94.

(Arrange your answers in the order in which the questions stand. So far as possible illustrate your discussions by a comparison of the experience of different countries. Take the first three questions and four others.)

  1. “As soon, however, as the factory system was established, the inequality of women and children in their struggle with employers attracted the attention of even the most careless observers; and, attention once drawn to this circumstance, it was not long before the inequality of adult men was also brought into prominence.” How far is this true (a) of England, (b) of the United States? Trace briefly the legislative consequences for children and for adults in the two countries.
    [Arnold Toynbee. Lectures on the Industrial Revolution of the 18th Century in England (The Humboldt Library of Popular Science Literature, Vol. 13. New York: Humboldt Publishing Co.), p. 17.]
  2. “It will be necessary, in the first place, to distinguish clearly between the failure of Industrial Coöperation and the failure of the coöperative method—a method, as we have seen, adopted, even partially, by only a very small fraction of Industrial Coöperation.” Explain carefully, discussing especially the evidence furnished by France and England.
  3. “These four concerns—the Maison Leclaire, the Godin Foundry, the Coöperative Paper Works of Angoulême and the Bon Marché—are virtually coöperative; certainly they secure to the employers and stockholders the substantial benefits of purely coöperative productive enterprises, while they are still, logically, profit-sharing establishments.” State your reasons for agreeing or disagreeing. Indicate briefly the characteristic features of each enterprise.
  4. “What inferences are we to draw from the foregoing statistics? Unmistakably this, that the higher daily wages in America do not mean a correspondingly enhanced labor cost to the manufacturer. But why so?” Discuss the character of available evidence in regard to the United States, Great Britain and the continent of Europe.
    [E. R. L. Gould. The Social Condition of Labor (Baltimore: Johns Hopkins Press, January 1893), pp. 41-2.]
  5. “The juxtaposition of figures portraying the social-economic status of workmen of different nationalities in the country of their birth and the land of their adoption furnishes lessons of even higher interest. From this we are able to learn the social effect of economic betterment.” Explain. How do the facts in question affect your attitude toward recent changes in the character and volume of our immigration?
    [E. R. L. Gould. The Social Condition of Labor (Baltimore: Johns Hopkins Press, January 1893), pp. 35-6.]
  6. “The Senate Finance Committee issued some time ago a comparative exhibit of prices and wages for fifty-two years, from which the conclusion is generally drawn that the condition of the wage earner is better to-day than it was thirty or forty years ago. A conclusion of this kind reveals the weakness of even the best statistics. No one can doubt that the work of the Finance Committee is work of high excellence, but for comparing the economic condition of workers it is of little value.” Do you agree or disagree? Why? Indicate briefly the character of the evidence.
  7. What are the principle organizations which may be said to represent the “Labor Movement” in the United States at the present time? How far are they helpful and how far hostile to one another?
  8. “In a preceding chapter I have said that as a moral force and as a system the factory system of industry is superior to the domestic system, which it supplanted.” State your reasons for agreeing or disagreeing.
    [Carroll D. Wright. Factory Legislation from Vol. II, Tenth Census of the United States, reprinted inFirst Annual Report of the Factory Inspectors of the State of New York (Albany, 1887), p. 41.]
  9. Contrast the English and the German policy in regard to Government Workingmen’s Insurance.
  10. “Gladly turning to more constructive work, I next consider some industrial changes and reforms which would tend to correct the present bias towards individualism.” What are they?
  11. Give an imaginary family budget for American, English and German operatives in one of the following industries, — coal, iron, steel, cotton, wool, glass, indicating roughly characteristic differences in such items as throw most light on the social condition of labor.

Source: Harvard University Archives. Harvard University, Examination papers, 1873-1915. Box 4, Volume: Examination Papers, 1893-95. pp. 39-41. Transcribed and posted earlier at Economics in the Rear-view Mirror.

______________________

Economics 10.
The Elements of Economic History from the Middle Ages to Modern Times.
1893-94.

[Economics] 10. Professor Ashley. — The Elements of Economic History from the Middle Ages to Modern Times. 3 hours.

Total 51: 6 Gr., 17 Se., 20 Ju., 4 So., 1 Fr., 3 Others.

Source: Harvard University. Annual Report of the President of Harvard College, 1893-94, p. 61.

1893-94.
ECONOMICS 10.
Mid-Year Examination.

 

  1. A modern writer has insisted upon the difference between the point of view of economic history and the point of view of constitutional history. Consider this in relation to the growth of mediaeval towns.
  2. Distinguish briefly between the various processes known as “Enclosure,” and explain their relation to the open-field husbandry.
  3. What light does the history of the English woollen industry throw upon the question as to the relation between the gild and the domestic workshop?
  4. “Only one who is unacquainted with social conditions under Henry VIll. and Edward VI. can maintain that the Reformation was not responsible for English pauperism.” Discuss this.

Source: Harvard University Archives. Harvard University, Mid-year examinations, 1852-1943. Box 3, Volume: Examination Papers, Mid-Year, 1893-94.

1893-94.
ECONOMICS 10.
Final Examination.

[Candidates are requested to answer only six questions, of which the first should be one.]

  1. Translate and comment upon:
    1. Omnes isti sochemanni habent viii carrucas, et arant iii vicibus per annum. Et quisquis eorum metit in Augusto de blado domini dimidiam acram et ii vicibus in Augusto precationem.
    2. Sciatis me concessisse … civibus meis in Oxenforde omnes libertates et consuetudines et leges et quietantias quas habuerunt tempore regis Henrici avi mei, nominatim gildam suam mercatoriam cum omnibus libertatibus et consuetudinibus in terris et in silvis pasturis et aliis pertinentiis, ita quod aliquis qui non sit de gildhalls aliquam mercaturam non faciet in civitate vel suburbiis.
  2. Give some account of the changes in trade-routes during the sixteenth century.
  3. Describe the organization of industry in the middle of the reign of Elizabeth.
  4. Compare the Enclosures of the eighteenth century with those of the sixteenth.
  5. What was the condition of the mercantile marine of New England in the eighteenth century? What connection was there between this condition and the Navigation Acts?
  6. Institute a comparison between the reforms of Stein and Hardenberg and recent agrarian legislation in Ireland, or any other country with which you are familiar.
  7. What light is cast upon the teaching of (1) Adam Smith, (2) Malthus, (3) Ricardo, by contemporary economic conditions.
  8. Estimate the importance of Arthur Young in the economic history of England.
  9. What seem to you the most characteristic features of the economic development of the United States during the present century as contrasted with England.

Source: Harvard University Archives. Harvard University, Examination papers, 1853-2001. Box 2, Volume: Papers Set for Final Examinations in Philosophy, History, Government and Law, Economics, Fine Arts, and Music in Harvard College, June 1894, pp. 42-43.

___________________________

Economics 12 (First Semester).
Banking and the History
of the leading Banking Systems
1893-94.

 Enrollment.

[Economics] 121. Professor Dunbar. — Banking and the History of the leading Banking Systems. 3 hours. 1st half-year.

Total 50: 10 Gr., 24 Se., 15 Ju., 1 Other.

Source: Harvard University. Report of the President of Harvard College, 1893-94, p. 62.

1893-94.
ECONOMICS 12[1].
Mid-Year Examination

  1. Which system of banks appears to present the greatest advantages, — (a) one with a powerful central bank as in England and Germany: (b) an aristocracy of strong banks as in Scotland; or (c) a democracy of banks as in this country?
  2. In any period of financial pressure, would the Bank of England he under any obligation, legal or moral, to act for the relief of the public, if such action involved risk or loss to its stockholders? What would be the source of such obligation, if any exists?
  3. The German bank act requires every bank to hold cash, (a) for all notes issued by it above its limit of uncovered issue: (b) and amounting to at least one third of all the notes issued Why is it that notes of other banks can be reckoned as cash in one of these cases, but not in the other?
  4. What is to be said as to the proposition frequently maintained. that “note issue is in reality a function of the State as much as coinage, and should not be delegated to corporations or to private hands?”
  5. If we hold that all note issues need to be kept under national control, in order to secure uniformity of value, what ground is there for denying that all deposit banking needs the same control for the same reason?
  6. Supposing the securities required for deposit under the national banking system to be abundant and fairly attractive as investments, — would that system afford an elastic currency?
  7. To the plan of securing notes by a safety fund (as practiced formerly in New York and now in Canada), it has been objected that it would be unjust to require well-managed banks to pay for losses incurred by weak or imprudent ones, and that a premium would be offered for bad management. How much weight is there in this objection?
  8. To the plan of making the notes of a bank a first lien on its assets it has been objected,—
    “It deprives the bank of the fund which is the basis of its credit in asking for deposits Without the deposit the banks cannot do a profitable business. It is difficult to believe that, the capital being subjected to a first lien for the amount of the notes, and there being always the possibility of an over-issue of such notes, the credit of the bank in its discount and deposit business would not be impaired. is calling upon the capital to do a double work when it is already loaded with the single task of inspiring confidence in the people who have to make deposits.”
    What is the answer to this objection?
  9. Discuss the following extract from the Commercial and Financial Chronicle of May 14th, 1892:—
    “Every prerogative and attribute even of our bank notes, and still more of our silver certificates, tends to draw them away from the interior, even when the issuer is resident in a Southern or Western State, and lodge them in an Eastern city. [The semi legal-tender quality of the national bank circulation and its redemption at the Treasury help to make its movements unnatural, artificial, and impart to it a roaming character helping to force it away from the issuer, away from the country districts where it is needed, and consequently to induce its accumulation when out of active commercial employment in the great financial centres, and while there to foster and become more or less fixed in speculative ventures — that is unresponsive to commercial influences when needed for commercial work?”

Source: Harvard University Archives. Harvard University, Mid-year examinations, 1852-1943. Box 3, Volume: Examination Papers, Mid-Year, 1893-94.

___________________________

Economics 12 (Second Semester).
International Payments and the Flow of the Precious Metals.
1893-94.

Enrollment.

[Economics] 122. Professor Dunbar.—International Payments and the Flow of the Precious Metals. 3 hours. 2d half-year.

Total 38: 12 Graduates, 18 Seniors, 7 Juniors, 1 Other.

Source: Harvard University. Report of the President of Harvard College, 1893-94, p. 62.

1893-94
ECONOMICS 122.
Final Examination.

  1. Mr. Goschen says that while a gold currency existed on both sides of the Atlantic the actual par of exchange between New York and London was about 109. What is the explanation of this method of stating the point of equilibrium?
  2. Is Clare justified in making the general statement that “the gold-points mark the highest level to which an exchange may rise, and the lowest to which it may fall?”
  3. What effect would the current rate of interest (as e.g. in a tight money market, either in the drawing or in the accepting country,) have on the rates for sixty-day bills as compared with cash bills?
  4. Clare makes the remark that “as the rate of exchange between two countries…must be fixed by the one who draws and negotiates the bill, it follows that the exchanges between England and most other countries are controlled from the other side, and that we in London have scarcely part or say in the matter.” Is the rate then a matter of indifference to those in London?
  5. Why is it that in certain trades bills are drawn chiefly, or even exclusively, in one direction, as e.g. by New York on London and not vice versa; and how is this practice made to answer the purpose of settling payments, which have to be made in one direction as well as the other?
  6. Mr. Goschen says that the primary cause which makes England the great banking centre of the world is “the stupendous and never-ceasing exports of England, which have for their effect that every country I the world, being in constant receipt of English manufactures, is under the necessity of making remittances to pay for them, either in bullion, in produce, or in bills.”
    Compare this statement with the fact that for ten years past the imports of merchandise into England have averaged about £400,000,000 annually, and the exports from England have averaged a little under £300,000,000.
  7. Suppose the exportation of specie from the United States to be prohibited (or, as has sometimes been suggested, to be slightly hindered,) what would be the effect on rates of exchange, and on prices of goods, either domestic or foreign? Would the country be a loser or not? [See Ricardo (McCulloch’s ed.) p. 139.]
  8. State Mr. Cairnes’s general doctrine as to the movement of prices which determines the normal flow of new supplies of gold from one country to another in the process of distribution over the commercial world.
  9. Cairnes argues that, as the effect of the cheapening of gold, “each country will endure a loss;” but that in particular cases “the primary loss may…be compensated, or even converted into a positive gain.” State and discuss the reasoning on which this proposition rests.
  10. Say, in his Report on the Indemnity, says:—
    La France a, en réalité, (1) fait passer à l’étranger le plus de capitaux possible, en prenant tous les changes qu’elle pouvait acquérir sur quelque pays que ce fût, et (2) a ensuite dirigé sur l’Allemagne tout ce qu’elle avait approvisionné ailleurs.

    1. What reason was there why France should prefer the course described in (1) rather than a direct transfer to Germany?
    2. What movements of trade or capital, of any sort, made the course described in (1) possible or easy?
    3. What movements of the same nature made (2) possible, or enable Germany to absorb the capital thus turned towards her?

*  *  *  *  *  *  *

  1. On either of the following topics, give an orderly and concise statement, as complete as you can make it in thirty minutes:—
    1. Sidgwick’s criticisms on Mill’s doctrine of international trade and their validity.
    2. The supply and distribution of the new gold from the United States and Australia, 1858-70.
    3. The action of the new gold in the banking countries.
    4. The absorption of new gold by the currency of France and the foreign trade of that country.
    5. The reasons for the varying ability of India to absorb silver?

Source:  Harvard University Archives. Final examinations, 1853-2001. Box 2, Papers set for Final Examinations in Philosophy, History, Government and Law, Economics, Fine Arts, and Music in Harvard College, June 1894, pp. 44-46. Transcribed and posted earlier in Economics in the Rear-view Mirror.

____________________

1893-94
Enrollment for Economics 13.
The Development of Land Tenures and of Agrarian Conditions in Europe.

Enrollment.

[Economics] 13. Professor Ashley. – The Development of Land Tenures and of Agrarian Conditions in Europe. 1 hour.

Total 2: 1 Graduate, 1 Senior.

Source: Harvard University. Report of the President of Harvard College, 1893-94, p. 61.

Note: No printed final examination in the collection of Harvard semester examinations.

____________________

Economics 14.
Ideal Social Reconstructions
from Plato to the Present.
1893-94.

Enrollment.

[Economics] 14. Asst. Professor Cummings. — Ideal Social Reconstructions, from Plato’s Republic to the present time. 1 hour.

Total 22: 7 Graduates, 8 Seniors, 5 Juniors, 2 Sophomores.

Source: Harvard University. Report of the President of Harvard College, 1893-94, p. 61.

ECONOMICS 14.
Mid-year examination, 1893-94.

(Arrange your answers in the order of the questions. Omit one.)

  1. What is a Utopia? and what significance do you attached to the recurrence of such literature at certain historical ethics?
  2. “For judging of the importance of any thinker in the history of Economics, no matter is more important to us than the view he takes of the laboring population.” Judge Plato, More and Bacon by this standard.
  3. “Moreover, it is hardly too much to say that Plato never got to the point of having a theory of the State at all.” In the Republic “man is treated as a micropolis, and the city is the citizen writ large.” Explain and criticize.
  4. “In More’s Utopia we have a revival of the Platonic Republic with additions which make the scheme entirely modern.… The economical element in the social body receives for the first time its proper rank as of the highest moment for public welfare.” Explain. To what extent have the ideals of Utopia been realized?
  5. “Then we may say that democracy, like oligarchy, is destroyed by its insatiable craving for the object which defines to be supremely good?” What, according to the Republic are the peculiar merits and defects of the several forms of political organization? and how are these forms related in point of origin and sequence?
  6. “Sir Thomas More has been called the father of Modern Communism.” How does he compare in this respect with Plato? How far do you trace the influence of historical conditions in each case?
  7. “But in your case, it is we that have begotten you for the State as well as for yourselves, to be like leaders and kings of the hive,– better and more perfectly trained than the rest, and more capable of playing a part in both modes of life.” Criticise the method and purpose of the educational system of the Republic. How far does Plato’s argument as to the duty of public service apply to the educated man to-day?
  8. “The religious ferment produced by the Reformation movement had begun to show signs of abatement, when another movement closely connected with it made its appearance almost at the same time in England and Italy, namely, the rise of a new philosophy.” How was this new philosophy embodied in the social ideals of Bacon and of Campanella? and what is the distinguishing characteristic of it?
  9. What essential contrast between pagan and Christian ideals have you found in schemes for social regeneration?
  10. Is there any recognition of “Social Evolution” in the Utopian philosophies thus far considered?
  11. What in a word, do you regard as the chief defect of the social reconstruction suggested in turn by Plato, Lycurgus, More, Bacon and Campanella? To what main problems suggested by them have we still to seek an answer?

Source: Harvard University Archives. Harvard University, Mid-year examinations, 1852-1943. Box 3, Volume: Examination Papers, Mid-Year, 1893-94. Previously transcribed and posted in Economics in the Rear-view Mirror.

 

ECONOMICS 14.
Final examination, 1893-94.

(Arrange your answers in the order of the questions.)

  1. [“]The essential unity and continuity of the vital process which has been in progress in our civilization from the beginning is almost lost sight of. Many of the writers on social subjects at the present day are like the old school of geologists: they seem to think that progress has consisted of a series of cataclysms.” How far is this criticism true? Is the characteristic in question more or less conspicuous in earlier writers?
  2. “At the outset underneath all socialist ideals yawns the problem of population…. Under the Utopias of Socialism, one of two things must happen. Either this increase must be restricted or not. If it be not restricted, and selection is allowed to continue, then the whole foundations of such a fabric as Mr. Bellamy has constructed are bodily removed.” State carefully your reasons for agreeing or disagreeing. In which of the schemes for social reconstruction, ancient or modern, do you find any adequate recognition of the part which selection plays in progress?
  3. “If it is possible for the community to provide the capital for production without thereby doing injury to either the principle of perfect individual freedom or to that of justice, if interest can be dispensed with without introducing communistic control in its stead, then there no longer stands any positive obstacle in the way of the free social order.” Discuss the provisions by which Hertzka hopes to guaranteed this “perfect individual freedom.” Contrast him with Bellamy in this respect.
  4. “I perceive that capitalism stops the growth of wealth, not – as Marx has it – by stimulating ‘production for the market,’ but by preventing the consumption of the surplus produce; and that interest, though not unjust, will nevertheless in a condition of economic justice becomes superfluous and objectless.” Explain Hertzka’s reasoning and criticise the economic theory involved.”
  5. What is the gist of “News from Nowhere”?
  6. The condition which the social mind has reached may be tentatively described as one of realization, more or less unconscious, that religion has a definite function to perform in society, and that it is a factor of some kind in the social evolution which is in progress.” How far have you found a recognition of this factor in theories of social reconstruction?

Source: Harvard University Archives. Final Examinations, 1853-2001. (HUC 7000.28). Box 2, Papers Set for Final Examinations in Philosophy, History, Government and Law, Economics, Fine Arts, and Music in Harvard College, June 1894.

Also: Harvard University Archives. Harvard University, Examination papers, 1853-2001. Box 2, Volume: Papers Set for Final Examinations in Philosophy, History, Government and Law, Economics, Fine Arts, and Music in Harvard College, June 1894, pp. 46-47. Previously transcribed and posted in Economics in the Rear-view Mirror.

 

Source: Left-to-right: Dunbar, Taussig, Ashley. From University and their Sons. History, Influence and Characteristics of American Universities with Biographical Sketches and Portraits of Alumni and Recipients of Honorary Degrees. Editor-in-chief, General Joshua L. Chamberlain, LL.D. Vol II (1899), pp. 159 [Dunbar], 595 [Ashley].   Vol. III (1899), p. 99 [Taussig]